Re: [obm-l] Novo na lista

2005-11-02 Por tôpico Claudio Buffara
Po, amigo! A demonstracao estah essencialmente completa. Basta notar que
10^6 == 1 (mod 7) e, portanto, a coisa toda se repete com periodo 6 no
expoente de 10. Aquele E por ai vai... soh precisa ser substituido por uma
inducao formal, mas pra bom entenddor 99% de palavra deveria bastar.

[]s,
Claudio.

on 02.11.05 15:38, Adélman de Barros Villa Neto at [EMAIL PROTECTED]
wrote:

 ninguem ainda?
 
 
 On Mon, 31 Oct 2005 23:14:38 -0200, Adélman de Barros Villa Neto
 [EMAIL PROTECTED] escreveu:
 
 De: Adélman de Barros Villa Neto [EMAIL PROTECTED]
 Data: Mon, 31 Oct 2005 23:14:38 -0200
 Para: obm-l@mat.puc-rio.br
 Assunto: [obm-l] Novo na lista
 
 
 Olá,estou procurando de um arquivo da lista onde é demonstrado um critério de
 divisibilidade por 7.Alguem pode me ajudar?Encontrei essas mensagens mas em
 nem uma o autor completa a demonstração.
 Grato.
 
 Mod 7:
 1 == 1
 10 == 3
 100 == 2 == 
 (abc) = 100a + 10b + c == 2a + 3b + c (mod 7)
 
 Logo, 7 divide (abc) == 7 divide 2a + 3b + c
 
 1000 == -1
 1 == -3
 10 == -2 ==
 (abcdef) = 10a + 1b + 1000c + 100d + 10e + f ==
 -2a -3b -c + 2d + 3e + f == -(2a+3b+c) + (2d+3e+f) (mod 7)
 
 Logo, 7 divide (abcdef) == 7 divide -(2a+3b+c) + (2d+3e+f)
 
 E por ai vai
 
 Ficou claro?
 
 Entao farelo pra voce tambem.
 
 []s,
 Claudio.
 


=
Instruções para entrar na lista, sair da lista e usar a lista em
http://www.mat.puc-rio.br/~nicolau/olimp/obm-l.html
=


Re: [obm-l] m^x + x (off-topic)

2005-11-01 Por tôpico claudio\.buffara





De:
[EMAIL PROTECTED]




Para:
obm-l@mat.puc-rio.br




Cópia:





Data:
Mon, 31 Oct 2005 23:07:36 +0100




Assunto:
Re: [obm-l] m^x + x (off-topic)
 Só uma idéia (nem testei ainda) m^x tem período que divide phi(n) (é
 isso mesmo?), 

Acho que sim. Certamente quando m e n são primos entre si.

enquanto x tem período n. Agora, eu acho que phi(n) e n
 s~ao primos entre si. 

Não.Por exemplo, Phi(2^k) = 2^(k-1), Phi(6) = 2, Phi(12) = 4 e dados primos p, q onde p divide q-1, teremos que Phi(pq) = (p-1)(q-1) = múltiplo de p. Exemplos: Phi(21) = 12, Phi(55) = 40, etc..

Se for, acho que acabou.

Em geral, o período divide n*Phi(n), mas acho que isso não afeta a conclusão.

 
 Abraços
 --
 Bernardo Freitas Paulo da Costa
 
 
 On 10/31/05, claudio.buffara <[EMAIL PROTECTED]>wrote:
  Desculpem o off-topic mas alguém sabe provar que a função f: N - Z_n dada
  por f(x) = m^x + x é sobrejetiva, quaisquer que sejam m, n naturais?
  (N = {1,2,3,...})
 
  []s,
  Claudio.
 
 
 =
 Instruções para entrar na lista, sair da lista e usar a lista em
 http://www.mat.puc-rio.br/~nicolau/olimp/obm-l.html
 =
 


Re:[obm-l] trigonometria (de novo)

2005-11-01 Por tôpico claudio\.buffara
Sem dúvida. Falha minha...

[]s,
Claudio.





De:
[EMAIL PROTECTED]




Para:
obm-l@mat.puc-rio.br




Cópia:





Data:
Mon, 31 Oct 2005 19:24:18 -0300 (ART)




Assunto:
Re:[obm-l] trigonometria (de novo)
 
 
 No "Por outro lado" o resultaod não é
 
 (3t - t^3)/(1 - 3 t^2) ?
 
 
 --- "claudio.buffara" <[EMAIL PROTECTED]>
 escreveu:
 
  A identidade pode não ser óbvia, mas é fácil de
  provar:
  
  Pondo t = tg(x), teremos:
  
  Por um lado,
  tg(3x) =
  tg(2x + x) =
  (tg(2x) + t)/(1 - tg(2x)*t) =
  (2t/(1 - t^2) + t)/(1 - 2t^2/(1 - t^2)) =
  (3t - t^2)/(1 - 3t^2).
  
  Por outro lado,
  tg(x)*tg(60 - x)*tg(60 + x) =
  t*(tg(60)-t)*(tg(60)+t)/( (1-t*tg(60))*(1+t*tg(60))
  ) =
  t*(3 - t^2)/(1 - 3t^2) =
  (3t - t^2)/(1 - 3t^2)
  
  []s,
  Claudio.
  
  De:[EMAIL PROTECTED]
  
  Para:"obm-l" obm-l@mat.puc-rio.br
  
  Cópia:
  
  Data:Mon, 31 Oct 2005 13:27:46 -0300
  
  Assunto:Re:[obm-l] trigonometria (de novo)
  
   Faça x = 10.
  
   No entanto, será que essa solução é única (a menos
  de múltiplos do período de tg(x)tg(5x)tg(7x))?
  Aliás, quanto vale P?
   E você também precisa provar a tal identidade, que
  não me parece óbvia.
  
   []s,
   Claudio.
  
   De:[EMAIL PROTECTED]
  
   Para:obm-l@mat.puc-rio.br
  
   Cópia:
  
   Data:Mon, 31 Oct 2005 11:15:33 -0200
  
   Assunto:[obm-l] trigonometria (de novo)
  
pessoal, eu não consegui resolver essa questão:
   
(tgx)*(tg5x)*(tg7x) = sqrt(3)/3
   
ate me deram a dica de usar essa identidade:
   
tg3x = (tgx)*[tg(60-x)]*[tg(60+x)]
   
mas ainda assim, eu não achei a resposta...
   
alguem pode me ajudar a resolver?
   
abraços
   
   
 
 _
MSN Messenger: converse online com seus amigos .
http://messenger.msn.com.br
   
   
 
 =
Instruções para entrar na lista, sair da lista e
  usar a lista em
   
  http://www.mat.puc-rio.br/~nicolau/olimp/obm-l.html
   
 
 =
   
  
 
 
 
 
 
 
 
 
 
 ___ 
 Promoção Yahoo! Acesso Grátis: a cada hora navegada você
 acumula cupons e concorre a mais de 500 prêmios! Participe!
 http://yahoo.fbiz.com.br/
 =
 Instruções para entrar na lista, sair da lista e usar a lista em
 http://www.mat.puc-rio.br/~nicolau/olimp/obm-l.html
 =
 


Re: [obm-l] m^x + x (off-topic)

2005-11-01 Por tôpico claudio\.buffara
O problema geral por trás disso parece ser o seguinte:

Dado um conjunto finito A euma função periódica e sobrejetiva f: N - Z_n (n arbitrário mas fixo), que condições uma função g: N -Z_n deve satisfazer para que f + g seja sobrejetiva?






De:
[EMAIL PROTECTED]




Para:
obm-l@mat.puc-rio.br




Cópia:





Data:
Mon, 31 Oct 2005 23:07:36 +0100




Assunto:
Re: [obm-l] m^x + x (off-topic)
 Só uma idéia (nem testei ainda) m^x tem período que divide phi(n) (é
 isso mesmo?), enquanto x tem período n. Agora, eu acho que phi(n) e n
 s~ao primos entre si. Se for, acho que acabou
 
 Abraços
 --
 Bernardo Freitas Paulo da Costa
 
 
 On 10/31/05, claudio.buffara <[EMAIL PROTECTED]>wrote:
  Desculpem o off-topic mas alguém sabe provar que a função f: N - Z_n dada
  por f(x) = m^x + x é sobrejetiva, quaisquer que sejam m, n naturais?
  (N = {1,2,3,...})
 
  []s,
  Claudio.
 
 


[obm-l] Re:[obm-l] CÍRCULO QUADRADO!

2005-11-01 Por tôpico claudio\.buffara





De:
[EMAIL PROTECTED]




Para:
obm-l@mat.puc-rio.br




Cópia:





Data:
Tue, 01 Nov 2005 11:38:55 +




Assunto:
[obm-l] CÍRCULO QUADRADO!
 Ok! Eduardo, pois a tal quadratura do círculo me faz lembrar a exposição 
 interativa de experimentos de Física, instalada no pavilhão da UNIJUÍ onde 
 as novidades na área da mecânica foram a "roda quadrada", "os ciclóides", 
 etc...
 
Esse é legal. Se uma bicicleta tem ambas as rodas quadradas e iguais, qual deve ser o formato do chão para que o ciclista ande em linha reta sem sacudir?

Dê uma olhada em:
http://www.sciencenews.org/articles/20040403/mathtrek.asp

[]s,
Claudio.



Re: [obm-l] m^x + x (off-topic)

2005-11-01 Por tôpico claudio\.buffara
Então eu acerteiao dizer queera off-topic, pois problemas de olimoíada são o que menos têm aparecido nessa lista...

[]s,
Claudio.





De:
[EMAIL PROTECTED]




Para:
obm-l@mat.puc-rio.br




Cópia:





Data:
Tue, 1 Nov 2005 14:14:39 -0200




Assunto:
Re: [obm-l] m^x + x (off-topic)Esse é essencialmente o problema 6 da terceira fase do terceiro nível da OBM desse ano, escrito de uma forma diferente.
 Em 01/11/05, claudio.buffara [EMAIL PROTECTED] escreveu:

 O problema geral por trás disso parece ser o seguinte:
 
 Dada um função periódica e sobrejetiva f: N - Z_n (n arbitrário mas fixo), que condições uma função g: N -Z_n deve satisfazer para que f + g seja sobrejetiva?
 


[obm-l] Re:[obm-l] Re: [obm-l] Número Co mplexo

2005-11-01 Por tôpico claudio\.buffara
Outra forma de resolver o problema é observar que, no plano complexo,o lugar geométrico dos complexos z tais que:
|(z-i)/(z-1)| = 1 == |z-i| = |z-1| e z  1
é a mediatriz do segmento cujas extremidades são os complexos 1 e i, ou seja, a reta Re(z) = Im(z), bissetriz dos quadrantes ímpares.

[]s,
Claudio.





De:
[EMAIL PROTECTED]




Para:
obm-l@mat.puc-rio.br




Cópia:





Data:
Tue, 1 Nov 2005 13:42:04 -0200




Assunto:
[obm-l] Re: [obm-l] Número Complexo
 Bom, resolvendo aqui também encontrei a=b.
 Logo, qualquer a e b satisfazem a equacao, inclusive a = b = 0.
 
 Abraço,
 Marcelo

- Original Message - 
From: gustavo 
To: obm-l@mat.puc-rio.br 
Sent: Monday, October 31, 2005 6:45 PM
Subject: [obm-l] Número Complexo
 
 Sendo Z = a + bi, eI (Z - i)/ ( Z - 1) I = 1 , ou sejao módulo deste quocienteé igual 1.Encontrei apenas que a =bporémo gabarito que eu tenho informa que a =b = 1/2 ,porém testei a=b =3 , deu certo !! em tempo: não seria a = b e diferenre de zero ,desde já agradeçoqualquer ajuda !!


[obm-l] tg(3x) = tg(x)*tg(60-x)*tg(60+x)

2005-11-01 Por tôpico claudio\.buffara

Pondo t = tg(x), teremos:

Por um lado,
tg(3x) =
tg(2x + x) =
(tg(2x) + t)/(1 - tg(2x)*t) =
(2t/(1 - t^2) + t)/(1 - 2t^2/(1 - t^2)) =
(3t - t^3)/(1 - 3t^2).

Por outro lado,
tg(x)*tg(60 - x)*tg(60 + x) =
t*(tg(60)-t)*(tg(60)+t)/((1-t*tg(60))*(1+t*tg(60))) =
t*(3 - t^2)/(1 - 3t^2) =
(3t - t^3)/(1 - 3t^2)

Agora foi...

[]s,
Claudio.



Re:[obm-l] trigonometria (de novo)

2005-10-31 Por tôpico claudio\.buffara
Faça x = 10.

No entanto, será que essa solução é única (a menos de múltiplos do período de tg(x)tg(5x)tg(7x))? Aliás, quanto vale P?
E você também precisa provar a tal identidade, que não me parece óbvia.

[]s,
Claudio.





De:
[EMAIL PROTECTED]




Para:
obm-l@mat.puc-rio.br




Cópia:





Data:
Mon, 31 Oct 2005 11:15:33 -0200




Assunto:
[obm-l] trigonometria (de novo)
 pessoal, eu não consegui resolver essa questão:
 
 (tgx)*(tg5x)*(tg7x) = sqrt(3)/3
 
 ate me deram a dica de usar essa identidade:
 
 tg3x = (tgx)*[tg(60-x)]*[tg(60+x)]
 
 mas ainda assim, eu não achei a resposta...
 
 alguem pode me ajudar a resolver?
 
 abraços
 
 _
 MSN Messenger: converse online com seus amigos . 
 http://messenger.msn.com.br
 
 =
 Instruções para entrar na lista, sair da lista e usar a lista em
 http://www.mat.puc-rio.br/~nicolau/olimp/obm-l.html
 =
 


Re:[obm-l] trigonometria (de novo)

2005-10-31 Por tôpico claudio\.buffara
A identidade pode não ser óbvia, mas é fácil de provar:

Pondo t = tg(x), teremos:

Por um lado, 
tg(3x) = 
tg(2x + x) = 
(tg(2x) + t)/(1 - tg(2x)*t) =
(2t/(1 - t^2) + t)/(1 - 2t^2/(1 - t^2)) = 
(3t - t^2)/(1 - 3t^2).

Por outro lado, 
tg(x)*tg(60 - x)*tg(60 + x) = 
t*(tg(60)-t)*(tg(60)+t)/( (1-t*tg(60))*(1+t*tg(60)) ) =
t*(3 - t^2)/(1 - 3t^2) =
(3t - t^2)/(1 - 3t^2)

[]s,
Claudio.





De:
[EMAIL PROTECTED]




Para:
"obm-l" obm-l@mat.puc-rio.br




Cópia:





Data:
Mon, 31 Oct 2005 13:27:46 -0300




Assunto:
Re:[obm-l] trigonometria (de novo)
 Faça x = 10.
 
 No entanto, será que essa solução é única (a menos de múltiplos do período de tg(x)tg(5x)tg(7x))? Aliás, quanto vale P?
 E você também precisa provar a tal identidade, que não me parece óbvia.
 
 []s,
 Claudio.
 




De:
[EMAIL PROTECTED]




Para:
obm-l@mat.puc-rio.br




Cópia:





Data:
Mon, 31 Oct 2005 11:15:33 -0200




Assunto:
[obm-l] trigonometria (de novo)
  pessoal, eu não consegui resolver essa questão:
  
  (tgx)*(tg5x)*(tg7x) = sqrt(3)/3
  
  ate me deram a dica de usar essa identidade:
  
  tg3x = (tgx)*[tg(60-x)]*[tg(60+x)]
  
  mas ainda assim, eu não achei a resposta...
  
  alguem pode me ajudar a resolver?
  
  abraços
  
  _
  MSN Messenger: converse online com seus amigos . 
  http://messenger.msn.com.br
  
  =
  Instruções para entrar na lista, sair da lista e usar a lista em
  http://www.mat.puc-rio.br/~nicolau/olimp/obm-l.html
  =
  


[obm-l] m^x + x (off-topic)

2005-10-31 Por tôpico claudio\.buffara
Desculpem o off-topic mas alguém sabe provar que a função f: N - Z_n dada por f(x) = m^x +xé sobrejetiva, quaisquer que sejam m, n naturais?
(N = {1,2,3,...})

[]s,
Claudio.



Re: [obm-l] CABEÇA À PRÊMIO!

2005-10-28 Por tôpico Claudio Buffara

on 28.10.05 12:48, Chicao Valadares at [EMAIL PROTECTED] wrote:

 pondo alcool congelado na sua provocacao, lembre-se
 que vc uma vez ja postou aqui uma msg perguntando como
 fazer uma moeda ter a prob da aresta igual a 1/3 e nao
 respondeu a mesma deixando para outros...
 
 
 --- [EMAIL PROTECTED] escreveu:
 
 Prof. Nicolau e participantes,
 posso também postar textos como esse? Meu pai tem os
 17 volumes daquela
 famosa enciclopedia Tesouro da Juventude
 Se sim, na proxima mensagem não percam:
 
 -De que é feito o Sol?
 -Conhece-se alguma especie de materia que não se
 encontre na terra?
 -O que é o vacuo?
 -Poderemos transportar-nos um dia a outro planeta?
 
 
 Jorge Luis Rodrigues e Silva Luis wrote:
 Pessoal! Juro que não queria mais tocar nesse
 assunto, pois além de
 fugir um pouco do escopo da lista, estou correndo
 sério risco de ser
 deletado por estar brincando com fogo. Mas,
 atendendo à um pedido
 muito especial, me vejo na obrigação de jogar
 água na fogueira.
 Acredito que o mais racional seria procurarmos os
 departamentos de
 física das nossas respectivas universidades para
 ratificarmos os tais
 supostos absurdos.
 
 Para que a barcaça consiga passar por baixo da
 ponte é necessário
 remover o cascalho que está dentro dela e jogá-lo
 na água, pois assim o
 nível da água, juntamente com a barcaça, irão
 abaixar numa proporção n
 vezes maior que o calado da embarcação.
 
 Quanto ao cubo de gelo que derretesse sòmente após
 ser jogado dentro da
 piscina, PASMEM! O nível da água abaixará devido o
 cubo de gelo deslocar
 mais volume d'água quando se encontra dentro do
 barco. Vale salientar
 que o nível da água permaneceria o mesmo caso o
 gelo estivesse boiando
 na água antes de derreter, o que não é o nosso
 caso.
 
 Com relação à altura máxima na qual a água poderia
 ser bebida através de
 um canudo, não importa quão fortes seus pulmões
 possam ser, ou qual o
 dispositivo que você usa para produzir vácuo
 dentro do canudo, ao nível
 do mar a água não poderia ser empurrada pela
 atmosfera mais alto do que
 10,3m. E aí, vem a inevitável pergunta. Como
 retirar água de um poço
 artesiano à 90 metros de profundidade para encher
 uma caixa d'água de um
 prédio de 100 andares.
 
 Vocês sabiam...que, o álcool congelado pega fogo.
 Inacreditável, não!
 Experimentem!
 
 E após tanta água, o melhor a fazer é refrescar a
 cuca e darmos um
 gelo nesse assunto tão ardiloso... Abraços!
 
 
 
 _
 MSN Messenger: converse online com seus amigos .
 http://messenger.msn.com.br
 
 
 
 =
 Instruções para entrar na lista, sair da lista e
 usar a lista em
 
 http://www.mat.puc-rio.br/~nicolau/olimp/obm-l.html
 
 
 =
 
 
 
 
 -- 
 Niski - http://www.linux.ime.usp.br/~niski
 
 sin^2(X) is odious to me, even thoug Laplace made
 use of it; shoud it
 be feared that sin^2(x) might become ambiguous,
 which would perhaps
 never occur ... well then, let us write (sin(x))^2,
 but not sin^2(X), which
 by analogy should signify sin(sin(x))
 
 Carl Friedrich Gauss
 
 =
 Instruções para entrar na lista, sair da lista e
 usar a lista em
 http://www.mat.puc-rio.br/~nicolau/olimp/obm-l.html
 
 =
 
 
 
 O Binômio de Newton é tão belo como a Vênus de Milo.
 O que há é pouca gente para dar por isso... 
 Fernando Pessoa - Poesias de Alvaro Campos
 
 _
 As informações existentes nessa mensagem e no(s) arquivo(s) anexado(s)
 são
 para uso restrito, sendo seu sigilo protegido por lei. Caso não seja
 destinatário, saiba que leitura, divulgação ou cópia são proibidas.
 Favor
 apagar as informações e notificar o remetente. O uso impróprio será
 tratado
 conforme as normas da empresa e a legislação em vigor. Agradecemos sua
 colaboração.
 
 
 The information mentioned in this message and in the archives attached
 are
 of restricted use, and its privacy is protected by law. If you are not
 the
 addressee, be aware that reading, disclosure or copy are forbidden.
 Please
 delete this information and notify the sender. Inappropriate use will
 be
 tracted according to company's rules and valid laws. Thank you for your
 cooperation.
 
 
 
 
 
 
 
 
 ___
 Promoção Yahoo! Acesso Grátis: a cada hora navegada você
 acumula cupons e concorre a mais de 500 prêmios! Participe!
 http://yahoo.fbiz.com.br/
 =
 Instruções para entrar na lista, sair da lista e usar a lista em
 http://www.mat.puc-rio.br/~nicolau/olimp/obm-l.html
 =
 



Re: [obm-l] CABEÇA À PRÊMIO!

2005-10-28 Por tôpico Claudio Buffara
Deixa eu entender: Sempre que eu mandar uma msg propondo um problema pra
lista, vou ter que mandar tambem uma msg com a solucao? E se eu nao conhecer
a solucao? 

No mais, esse problema da moeda foi respondido pelo Nicolau e, se nao me
engano, por outros participantes tambem. E trata-se de um problema 100%
matematico, ou seja, 100% on-topic.

[]s,
Claudio.

on 28.10.05 12:48, Chicao Valadares at [EMAIL PROTECTED] wrote:

 pondo alcool congelado na sua provocacao, lembre-se
 que vc uma vez ja postou aqui uma msg perguntando como
 fazer uma moeda ter a prob da aresta igual a 1/3 e nao
 respondeu a mesma deixando para outros...
 
 

=
Instruções para entrar na lista, sair da lista e usar a lista em
http://www.mat.puc-rio.br/~nicolau/olimp/obm-l.html
=


Re: [obm-l] Re:[obm-l] Aparições

2005-10-28 Por tôpico Claudio Buffara
Title: Re: [obm-l] Re:[obm-l] Aparições



Falando nisso, qual a relacao entre:

1) a razao entre o comprimento e o diametro de uma circunferencia;

2) SOMA(n=1...infinito) 1/n^(2k) com k natural;

3) INTEGRAL(-infinito...+infinito) exp(-x^2)dx;

4) PRODUTO(n=1...infinito) (4n^2/(4n^2-1)) ?

Ou seja, por que aparece sempre a mesma constante nessas situacoes aparentemente tao diversas?

Tentar responder a essa pergunta foi o que me fez voltar a estudar matematica. 

[]s,
Claudio.

on 28.10.05 19:39, Osvaldo Mello Sponquiado at [EMAIL PROTECTED] wrote:

http://www.expoente.com.br/professores/kalinke/projeto/aurea.htm - razao aurea
 
Quem tal usar o google??
 
 
 
 Olá pessoal boa tarde. 
 
 Gostaria de saber sobre outras aparições da razão áurea...e do número de Euler, bem como algum site que fale desta incrível matemática do universo e as constantes que se repetem, entre elas a sequencia de Fibonacci por exemplo. 
 
 Um abraço, Marcelo. 
 
 No iBest, suas horas navegadas valem pontos que podem ser trocados por prêmios. Sem sorteio! Inscreva-se já! www.navegueeganhe.com.br 
 





Re: [obm-l] Fatoração?

2005-10-27 Por tôpico Claudio Buffara
Title: Re: [obm-l] Fatoração?



Por que?

on 27.10.05 18:38, Iuri at [EMAIL PROTECTED] wrote:

a^3 + b^3 + c^3 - 3abc = (a+b+c)(a^2 + b^2 + c^2 -ab -ac -bc)
Usando essa identidade, ta provado.


Em 27/10/05, Raul Ribeiro [EMAIL PROTECTED] escreveu:
 Boa tarde!

 Essa é da Opm-02 (Alguém sabe onde encontrar os gabaritos das opm's?)

 Prove que a equação abaixo tem infinitas soluções inteiras positivas?

 x^3 + 2y^3 + 4z^3 - 6xyz = 1

= 
Instruções para entrar na lista, sair da lista e usar a lista em
http://www.mat.puc-rio.br/~nicolau/olimp/obm-l.html
= 








Re:[obm-l] equacao

2005-10-26 Por tôpico claudio\.buffara
Seja d = mdc(x,y). Então x = dz e y = dw, com mdc(z,w) = 1.

A equação fica (z + w)k = dzw.

k não pode dividir z pois z = km ==
(km + w)k = dkmw ==
km + w = dmw ==
w = m(dw - k) ==
m divide w ==
contradição, pois z (e portanto m) é primo com w

Da mesma forma, vemos que k não pode dividir w.

Logo, k divide d == 
d = kn ==
(z + w)k = knzw ==
z + w = nzw ==
1/w + 1/z = n = inteiro positivo

Como z ew são inteiros positivos, 1/z + 1/w = 2.

Se z =w = 1, então x = y = d == 2dk = d^2 == d = 2k ==
uma solução é (2k,2k).

Se z  1 ou w  1, então 1/z + 1/w = n = 1 ==
z = w = 2 e d = k ==
de novo obtemos a solução (2k,2k).

Logo, a única solução é (2k,2k).






De:
[EMAIL PROTECTED]




Para:
obm-l@mat.puc-rio.br




Cópia:





Data:
Wed, 26 Oct 2005 11:28:09 + (GMT)




Assunto:
[obm-l] equacao
 Determine o conjunto solucao d (x+y)k = xy sendo x e y inteiros positivose k um numero primo


Promoção Yahoo! Acesso Grátis: a cada hora navegada vocêacumula cupons e concorre a mais de 500 prêmios! Participe!


Re:RES: RES: [obm-l] inversa = derivada

2005-10-26 Por tôpico claudio\.buffara
Eu supuz f(x) da forma ax^b, com a e b positivos.

Nesse caso, f^(-1)(x) = (x/a)^(1/b) e f'(x) = abx^(b-1).

Igualando coeficientes e expoentes, eu achei:
1/a^(1/b) = ab e 1/b = b-1 ==
a = 1/b^(1/(1+1/b))= 1/b^(1/b) e b^2 - b - 1 = 0

Como b  0, só pode ser b = (1+raiz(5))/2.

Assim, f:(0,+inf) - (0,+inf) dada por: 
f(x) = ax^b, com b = (1+raiz(5))/2 e a = 1/b^(1/b) é tal que:
f^(-1)(x) = f'(x) para cada x em (0,+inf).

Mais uma aparição (inusitada ?)da razão áurea...

***

O problema geral, de resolver a equação diferencial f'(x) = f^(-1)(x) em (0,+inf), me parece mais complicado.

[]s,
Claudio.





De:
[EMAIL PROTECTED]




Para:
obm-l@mat.puc-rio.br




Cópia:





Data:
Wed, 26 Oct 2005 10:29:51 -0200




Assunto:
RES: RES: [obm-l] inversa = derivada

 Assim, talvez exista(m) esta(s) funcao(oes). Jah vimos que, se existir, esta f, da forma como definida abaixo, tem que ser estritamente crescente. Isto implica que f^(-1) = f' tambem seja estritamente crescente e positiva. Dado que f' existe e nunca se anula, segue-se que a derivada de f^(-1) tambem existe em todo o intervalo (0, +inf). E como f^(-1) = f', concluimos que f' eh diferenciavelem (0, +inf), ou seja, f'' existe em (0, +inf).Assim, uma condicao adicional para a existencia desta funcao eh que sua derivada segunda exista em (0, + inf).Nao que isso ajude muito..
 
 Artur
 
 

-Mensagem original-De: [EMAIL PROTECTED] [mailto:[EMAIL PROTECTED]Em nome de claudio.buffaraEnviada em: terça-feira, 25 de outubro de 2005 19:02Para: obm-lAssunto: Re:RES: [obm-l] inversa = derivada
 Mudemos o enunciado:
 
 Dê um exemplo de uma bijeção diferenciável f:(0,+inf) - (0,+inf) tal que:
 f'(x) = f^(-1)(x) para todo x em (0,+inf).
 
 É possível achar todas as f com esta propriedade?
 
 []s,
 Claudio.


Re:[obm-l] equacao

2005-10-26 Por tôpico claudio\.buffara
Eu supuz que k é um primo fixo dado.





De:
[EMAIL PROTECTED]




Para:
"obm-l" obm-l@mat.puc-rio.br




Cópia:





Data:
Wed, 26 Oct 2005 12:20:17 -0200 (BRST)




Assunto:
Re:[obm-l] equacao
 
 Na verdare, por tentativa (e muitos erros)
 e' possivel tambem outras solucoes:
 
 zk - zw = -wk
 = z = -wk/(k-w)
 Logo, se k = (w+1) entao z = -w(w+1)
 
 Por simetria, se k = (z+1) entao w = -z(z+1)
 
 Abraco,
 sergio
 
 On Wed, 26 Oct 2005, claudio.buffara wrote:
 
  Seja d = mdc(x,y). Então x = dz e y = dw, com mdc(z,w) = 1.
  
  A equação fica (z + w)k = dzw.
  
  k não pode dividir z pois z = km ==
  (km + w)k = dkmw ==
  km + w = dmw ==
  w = m(dw - k) ==
  m divide w ==
  contradição, pois z (e portanto m) é primo com w
  
  Da mesma forma, vemos que k não pode dividir w.
  
  Logo, k divide d ==
  d = kn ==
  (z + w)k = knzw ==
  z + w = nzw ==
  1/w + 1/z = n = inteiro positivo
  
  Como z e w são inteiros positivos, 1/z + 1/w = 2.
  
  Se z = w = 1, então x = y = d == 2dk = d^2 == d = 2k ==
  uma solução é (2k,2k).
  
  Se z  1 ou w  1, então 1/z + 1/w = n = 1 ==
  z = w = 2 e d = k ==
  de novo obtemos a solução (2k,2k).
  
  Logo, a única solução é (2k,2k).
  
  
  De:[EMAIL PROTECTED]
  
  Para:obm-l@mat.puc-rio.br
  
  Cópia:
  
  Data:Wed, 26 Oct 2005 11:28:09 + (GMT)
  
  Assunto:[obm-l] equacao
  
   Determine o conjunto solucao d (x+y)k = xy sendo x e y inteiros positivos e k um numero primo
  
  
  Promoção Yahoo! Acesso Grátis: a cada hora navegada você
  acumula cupons e concorre a mais de 500 prêmios! Participe!
  
 
 =
 Instruções para entrar na lista, sair da lista e usar a lista em
 http://www.mat.puc-rio.br/~nicolau/olimp/obm-l.html
 =
 


Re: Re:[obm-l] equacao

2005-10-26 Por tôpico claudio\.buffara
É isso aí. Mancada minha!

O melhor jeito é olhar e ver que a equação equivale a 1/x + 1/y = 1/k, que sendo x e y positivos, devemos ter x  k == x = k + m, com m inteiro positivo e, portanto, 1/y = 1/k - 1/(k+m) = m/(k(k+m)) == 
y = k(k+m)/m == m divide k^2 e, como k é primo, m = 1, k ou k^2, o que dá origem as três soluções: (2k,2k)e mais as duas que você mencionou.

Fica como um novo problema determinar em que ponto da minha pseudo-resolução abaixo eu "perdi" as outras duas soluções.

[]s,
Claudio.





De:
[EMAIL PROTECTED]




Para:
obm-l@mat.puc-rio.br




Cópia:





Data:
Wed, 26 Oct 2005 14:12:05 -0200




Assunto:
Re: Re:[obm-l] equacao



  Mesmo assim, ainda temos as soluções: (k^2+k, k+1) e (k-k^2, k-1) e suas simétricas.

- Original Message - 
From: claudio.buffara 
To: obm-l 
Sent: Wednesday, October 26, 2005 1:14 PM
Subject: Re:[obm-l] equacao
 
 Eu supuz que k é um primo fixo dado.
 




De:
[EMAIL PROTECTED]




Para:
"obm-l" obm-l@mat.puc-rio.br




Cópia:





Data:
Wed, 26 Oct 2005 12:20:17 -0200 (BRST)




Assunto:
Re:[obm-l] equacao
  
  Na verdare, por tentativa (e muitos erros)
  e' possivel tambem outras solucoes:
  
  zk - zw = -wk
  = z = -wk/(k-w)
  Logo, se k = (w+1) entao z = -w(w+1)
  
  Por simetria, se k = (z+1) entao w = -z(z+1)
  
  Abraco,
  sergio
  
  On Wed, 26 Oct 2005, claudio.buffara wrote:
  
   Seja d = mdc(x,y). Então x = dz e y = dw, com mdc(z,w) = 1.
   
   A equação fica (z + w)k = dzw.
   
   k não pode dividir z pois z = km ==
   (km + w)k = dkmw ==
   km + w = dmw ==
   w = m(dw - k) ==
   m divide w ==
   contradição, pois z (e portanto m) é primo com w
   
   Da mesma forma, vemos que k não pode dividir w.
   
   Logo, k divide d ==
   d = kn ==
   (z + w)k = knzw ==
   z + w = nzw ==
   1/w + 1/z = n = inteiro positivo
   
   Como z e w são inteiros positivos, 1/z + 1/w = 2.
   
   Se z = w = 1, então x = y = d == 2dk = d^2 == d = 2k ==
   uma solução é (2k,2k).
   
   Se z  1 ou w  1, então 1/z + 1/w = n = 1 ==
   z = w = 2 e d = k ==
   de novo obtemos a solução (2k,2k).
   
   Logo, a única solução é (2k,2k).
   
   
   De:[EMAIL PROTECTED]
   
   Para:obm-l@mat.puc-rio.br
   
   Cópia:
   
   Data:Wed, 26 Oct 2005 11:28:09 + (GMT)
   
   Assunto:[obm-l] equacao
   
Determine o conjunto solucao d (x+y)k = xy sendo x e y inteiros positivos e k um numero primo
   
   
   Promoção Yahoo! Acesso Grátis: a cada hora navegada você
   acumula cupons e concorre a mais de 500 prêmios! Participe!
   
  


Re:RES: [obm-l] inversa = derivada

2005-10-25 Por tôpico claudio\.buffara
Mudemos o enunciado:

Dê um exemplo de uma bijeção diferenciável f:(0,+inf) - (0,+inf) tal que:
f'(x) = f^(-1)(x) para todo x em (0,+inf).

É possível achar todas as f com esta propriedade?

[]s,
Claudio.





De:
[EMAIL PROTECTED]




Para:
obm-l@mat.puc-rio.br




Cópia:





Data:
Tue, 25 Oct 2005 16:39:45 -0200




Assunto:
RES: [obm-l] inversa = derivada

 De fato, o colega se equivocou.
 
 Definida em todo o R, nao existe tal funcao. Supondo-se que esta f exista e seja definida em todo o R, temos que, por possuir uma inversa, f eh estritamente monotonica em R. Suponhamos que f seja monotonicamente crescente. Entao, f'(u) = 0 para todo real u (1). Como f' eh a inversa de f, temos para todo x de R que f'(f(x)) = x (2). Mas, em virtude de (1), f'(f(x) = 0 para todo real x, de modo que (2) nao pode ser atendida para x0.
 
 Se f for monotonicamente decrescente, entao por um raciocinio similar vemos que (2) nao pode ser atendida para x0.
 
 Se esta funcao existir, entao a condicao pedida so podera ser atendida ou para valores de x positivos ou valores de x negativos.
 
 Artur
 



[obm-l] Funcoes e Aplicacoes

2005-10-20 Por tôpico Claudio Buffara
Title: Funcoes e Aplicacoes



A historia, de fundo matematico, eh baseada numa aplicacao A: Sal - Ovos e na confusao gerada pelo isomorfismo existente entre Sal e Talco...

O outro resultado vale em R^n e nao apenas na reta.

[]s,
Claudio.

on 19.10.05 12:16, Artur Costa Steiner at [EMAIL PROTECTED] wrote:

A história daquela Sra e do sal, eu nao entendi nao... Poderia explicar melhor? Vou tentar a da funcao, que parece mais facil.
 
(a) Se c =0, entao g eh constante e f(x) = x + C para alguma constante C. Segue-se automaticamente que f eh bijetora. Supondo-se c em (0, 1), admitamos que em I existam x e y distintos tais que f(x) = f(y). Entao, x + g(x) = y + g(y) = |g(x) - g(y)| = |x- y|. Como g eh Lipschitz, temos que |x - y| = c|x -y|. Como x e y sao distintos, concluimos que c=1, contrariamente aa hipotese. Logo, f eh uma injecao de I sobre f(I). E como todo elemento de f(I) eh, por definicao, imagem de algum x de I, segue-se que f eh uma bijecao enter I e f(I).
 
(b) Se c = 0, entao f(x) = x + C ea conclusao eh trivialmente verificada. Se c 0, jah foi demonstrado aqui, ha pouco tempo, que, como g eh Lipschitz com constante c e diferenciavel em I, entao |g'(x| = c para todo x de I. Como f'(x) = 1 + g'(x) e 0  c  1, temos que f' eh estritamente positiva em I (o que implica que f seja esritamente crecente em I). Por ser bijecao, f tem uma inversa f^(-1) e, como f' nao se anula em I e eh continua, um resultado classico da Analise diz que f^(-1) existe em I. 
 
(c) Suponhamos que I = R. Se c= 0 , entao f(x) = x + C e a conclusao eh imediata. Se c estiver em (0,1) entao, para todo real x, temos que |g(x) - g(0)|  = c|x|. de modo que |g(x| = |g(0| + c|x|. Para x0, temos entao que f(x) = x + g(x) = x -|g(0)| -c|x| = -|g(0)| + (1-c)*x. .Como c esta em (0,1), 1-c 0 e, aumentando x, podemos tornar f arbitrariamente grande. De modo similar, fazendo x - -oo podemos faxer f(x) - -oo. Como f eh continuaem R, pois g que eh Lipscitz e a funcao identidade sao continuas, temos que f(I) = R. 
 
 
Artur 
 
 
 
 -Mensagem original-
De: [EMAIL PROTECTED] [mailto:[EMAIL PROTECTED]Em nome de claudio.buffara
Enviada em: terça-feira, 18 de outubro de 2005 20:15
Para: obm-l
Assunto: Re:RES: [obm-l] Probabilidade

 
Sejam I um intervalo aberto de R, c um real em [0,1) e g: I - R tal que:
|g(x) - g(y)| = c|x - y| para quaisquer x e y em I.
Seja f: I - R dada por f(x) = x + g(x).
 
a) Prove que f é uma bijeção entre I e J = f(I) = intervalo aberto de R.
 
b) Prove que se g é continuamente diferenciável, então f é um difeomorfismo (bijeção diferenciável com inversa diferenciável) entre I e J.
 
c) Prove que se I = R, então J = R.
 
[]s,
Claudio.







Re:[obm-l] teoria dos numeros

2005-10-19 Por tôpico claudio\.buffara
Que tal 7 e 19?
7 = 4*1 + 3 e 19 = 4*4 + 3
mdc(1+1,4+1) = 1.

[]s,
Claudio.





De:
[EMAIL PROTECTED]




Para:
obm-l@mat.puc-rio.br




Cópia:





Data:
Wed, 19 Oct 2005 16:01:03 -0200




Assunto:
[obm-l] teoria dos numeros
 boa tarde a todos, quem me ajuda com esse?
 
 sabemos que existem infinitos numeros primos da forma 4k + 3. dado um 
 inteiro b e sendo S o conjunto de todos os primos da forma p = 4k + 3 , onde 
 p não divide b. a questão é: existem dois primos (4k + 3) e (4q + 3) em S de 
 tal forma que (k + 1) e (q + 1) são primos entre si?
 
 abraços
 
 _
 Chegou o que faltava: MSN Acesso Grátis. Instale Já! 
 http://www.msn.com.br/discador
 
 =
 Instruções para entrar na lista, sair da lista e usar a lista em
 http://www.mat.puc-rio.br/~nicolau/olimp/obm-l.html
 =
 


Re:[obm-l] 2 probleminhas

2005-10-19 Por tôpico claudio\.buffara





De:
[EMAIL PROTECTED]




Para:
obm-l@mat.puc-rio.br




Cópia:





Data:
Sun, 16 Oct 2005 06:36:44 -0300 (ART)




Assunto:
[obm-l] 2 probleminhas
 Ao se dividir o número 400 em parte diretamente
 proporcionais a 1, 2/3 e 5/3, obtem-se
 respectivamente:
 
 120, 80 e 200
 360, 240 e 600
 60, 40 e 100
 40, 80/3 e 200/3
 100,40 e 60
 
1 + 2/3 + 5/3 = 10/3

1/(10/3) = x/400 == x = 120

(2/3)/(10/3) = y/400 == y = 80

(5/3)/(10/3) = z/400 == z = 200

Ou entao, repare que a unica alternativa na qual a soma dos numeros eh 400 eh a primeira...


 Em uma circunferência são escolhidos 12 pontos
 distintos. Ligam-se quatro quaisquer destes pontos, de
 modo a formar um quadrilátero. o número total de de
 diferentes quadriláteros que podem ser formados é:
 
 128
 495
 545
 1.485
 11.880 
 
Cada4 pontos dah origem a 3 quadrilateros, um dos quais ehconvexo e os outros dois tem o formato de uma gravata borboleta.
Assim, temos Binom(12,4)*3 = 495*3 = 1485 quadrilateros.

[]s,
Claudio.

 
 
 
 
 
 
 ___ 
 Promoção Yahoo! Acesso Grátis: a cada hora navegada você acumula cupons e concorre a mais de 500 prêmios! Participe! http://yahoo.fbiz.com.br/
 =
 Instruções para entrar na lista, sair da lista e usar a lista em
 http://www.mat.puc-rio.br/~nicolau/olimp/obm-l.html
 =
 


Re: [obm-l] Probabilidade

2005-10-18 Por tôpico claudio\.buffara
Um argumento que me convenceu foi o seguinte:

Imagine que, ao invés de três, temos um milhão de portas, uma das quais esconde um carro e as 999.999 restantes, um bode cada uma.

Você escolhe uma porta e o apresentador abre 999.998 outras portas, todas com um bode atrás. Restam fechadas apenas a porta que você escolheu e uma outra.

Não querer trocar de porta significa que você acha que escolheu, de primeira,a porta com o carro - um evento com probabilidade de 1 em 10^6. Será que você é tão sortudo assim?

[]s,
Claudio.





De:
[EMAIL PROTECTED]




Para:
obm-l@mat.puc-rio.br




Cópia:





Data:
Tue, 18 Oct 2005 00:23:53 -0200




Assunto:
Re: [obm-l] Probabilidade
 Não vou entrar no mérito da questão, mas entre esses alguns matemáticos que por
 alguma razão acreditaram que não compensava mudar de porta esteve ninguém menos
 que Paul Erdös... E, mesmo após ouvir o argumento contrário, ele disse: "Não
 pode ser!". Portanto, não é vergonha alguma ficar encanado com esse problema,
 pelo menos em um primeiro momento.
 
 Leo
 
 
 Quoting "Nicolau C. Saldanha" <[EMAIL PROTECTED]>:
 
  On Mon, Oct 17, 2005 at 07:39:00PM -0300, cfgauss77 wrote:
   
   Num programa em que são sorteados prêmios tem-se 3 portas: uma com tesouro
  e
   duas com monstros. Você escolhe 1 das portas, mas não a abre. O
  apresentador
   do programa, para ajudá-lo, abre uma das outras portas (sem ser a de sua
   escolha) e desta sai um monstro. Pergunta-se, vale a pena trocar de
  porta???
  
  Este problema já foi muito discutido em muito lugares, nesta lista
  inclusive.
  Em geral é formulado com bodes em vez de monstros e um carro em vez de um
  tesouro. Este problema é baseado em um show americano, o apresentador
  chamava-se Monty Hall. O problema ficou especialmente famoso (infame?)
  depois que Marilyn Vos Savant, uma mulher com um QI supostamente altíssimo,
  respondeu a mesma pergunta que você fez na coluna dela na revista Parade.
  A resposta dela estava perfeitamente correta, mas por alguma razão muita
  gente (incluindo alguns matemáticos profissionais) acharam que estava tudo
  errado e escreveram várias cartas para a revista, algumas muito grosseiras.
  Se você procurar por "Monty Hall" e "Savant" no google você poderá ler
  um monte de coisa sobre este episódio, incluindo os textos originais
  da Marilyn e algumas das respostas. Você também pode querer ler o meu artigo
  na Eureka #1, "Como perder amigos e enganar pessoas":
  http://www.mat.puc-rio.br/~nicolau/publ/papers/bom.pdf
  
  Btw, a resposta correta é SIM, vale a pena trocar. Se você trocar
  a probabilidade de ganhar é 2/3. 
  
  []s, N.
  
  =
  Instruções para entrar na lista, sair da lista e usar a lista em
  http://www.mat.puc-rio.br/~nicolau/olimp/obm-l.html
  =
  
 
 
 =
 Instruções para entrar na lista, sair da lista e usar a lista em
 http://www.mat.puc-rio.br/~nicolau/olimp/obm-l.html
 =
 


Re:RES: [obm-l] Probabilidade

2005-10-18 Por tôpico claudio\.buffara
Este seu problema do salme lembra uma história, quase que certamente fictícia, mas mesmo assim engraçada. Há tempos, no programa do Sílvio Santos, este escolheu uma senhora da platéia e fez a ela uma pergunta valendo mil reais (ou o equivalente em dinheiro da época). A pergunta era: 
"Qual a substância química cuja fórmula é NaCl?" 
Ela não soube responder de cara e o Sílvio deu a seguinte pista: 
"É um pó e é branco."
Ainda assim, nada. 
Como o Sílvio estava com boa vontade naquele dia, ele deu mais uma dica:
"É o que o seu marido coloca nos ovos todos os dias de manhã."
Daí ela não titubeou: 
"Ah, Sílvio! Já sei! É talco!"...

Desculpem o off-topic e a infâmia, mas eu não resisti.

E pra não ficar totalmente fora da lei, aqui vai um de função:

Sejam I um intervalo aberto de R, c um real em [0,1) e g: I - R tal que:
|g(x) - g(y)| = c|x - y| para quaisquer x e y em I.
Sejaf: I - R dada por f(x) = x + g(x).

a) Prove que f é uma bijeção entre I e J = f(I) = intervalo aberto de R.

b) Prove que se g é continuamente diferenciável, então f é um difeomorfismo (bijeção diferenciável com inversa diferenciável) entre I e J.

c) Prove que se I = R, então J = R.

[]s,
Claudio.





De:
[EMAIL PROTECTED]




Para:
obm-l@mat.puc-rio.br




Cópia:





Data:
Tue, 18 Oct 2005 18:24:12 -0200




Assunto:
RES: [obm-l] Probabilidade

 Este seu argumento eh legal. Mas eu de fato estive tentado a dizer que a probabilidadeera 1/2, baseado que a informacao dada pela porta aberta mudou o nivel de conhecimento e o espaco amostral. Eh claro que o Nicolau estah certo, mas eu fui tentado a fazer o raciocinio como a do estudante de quimica que estava fazendo uma prova e numa das perguntas, de multipla escolha, pedia-se a formula do sal (cloreto de sodio) e davam-se 5 opcoes:
 
 a) H2So4
 b) NaCl
 c) H2O
 d) H Cl
 e)HF
 
 O estudante ia muito mal, estava completamente ignorante e ia chutar. Nesta situacao, qualquer das 5 alternativas era para ele tao boa quanto qualquer das outras e ele tinha 1/5 de probabilidade de dar a resposta certa NaCl. Mas de repenteele se lembrou de uma das poucas aulas em que tinha comparecido e lhe ocorreu quena formula do sal tem o cloro Cl. Dado que na lista de opcoes apenas 2 tem formulas contendo Cl,o espaco amostral do estudante reduziu-se ao composto pelas alternativas (b), correta, e (d), errada.Com relacao a estas duas, o estudante continuou completamente ignorante, mas com a reducao do espaco amostral sua probabilidade de acerto elevou-se para 1/2. Passou a interessar a nova situacao com o conhecimento de que na formula do sal entracloro. O fato de que antes do estudante se lembrar a probabilidade era 1/5 deixou de interessar.
 
 Mas de fato nao eh a mesma situacao do probelam dos bodes e do carro.
 Artur
 
 .




Re:[obm-l] Sistema Dificil

2005-10-18 Por tôpico claudio\.buffara
Todas as triplas (x,y,z) que satisfazem me parece difícil, mas uma solução particular é fácil: se w^3 + bw^2 + cw + d = 0, então (w,w,w) é solução.





De:
[EMAIL PROTECTED]




Para:
obm-l@mat.puc-rio.br




Cópia:





Data:
Tue, 18 Oct 2005 16:27:14 -0300 (ART)




Assunto:
[obm-l] Sistema Dificil
 Pessoal , alguem sabe fazer essa?
 
 
 Sejam b, c e d numeros complexos , encontre x , y e z tais que
 
  (3x^2 +2bx+c)y+ bx^2+2cx+3d=0
  (3y^2 +2by+c)z+ by^2+2cy+3d=0
  (3z^2 +2bz+c)x+ bz^2+2cz+3d=0
 
  Abs.


Promoção Yahoo! Acesso Grátis: a cada hora navegada você acumula cupons e concorre a mais de 500 prêmios! Participe!


Re:[obm-l] subconjunto proprio de R

2005-10-17 Por tôpico claudio\.buffara





De:
[EMAIL PROTECTED]




Para:
obm-l@mat.puc-rio.br




Cópia:





Data:
Sun, 16 Oct 2005 15:16:00 -0700 (PDT)




Assunto:
[obm-l] subconjunto proprio de R
 O problema a seguir talvez fosse mais interessante se
 nao tivessa havido esta discussao sobre conjuntos com
 interior vazio e medida positiva. Apos esta discussao,
 a solucao eh bem obvia:
 
 
 Sejam (r_n) uma enumeracao dos racionais, (x_n) uma
 sequencia de termos reais positivos, I_n = (r_n - x_n
 , r_n + x_n) e I = Uniao (n=1, inf) I_n. Entao, I eh
 um aberto denso em R. Mostre que, se Soma(n=1, inf)
 x_n convegir, entao I eh um subconjunto proprio de R.
 

Nesse caso, I tem medida finita = 2*SOMA(n=1...inf) x_n.

[]s,
Claudio.



Re:[obm-l] Dois exercicios sobre teoria dos grupos

2005-10-17 Por tôpico claudio\.buffara





De:
[EMAIL PROTECTED]




Para:
obm-l@mat.puc-rio.br




Cópia:





Data:
Mon, 17 Oct 2005 10:23:48 -0300 (ART)




Assunto:
[obm-l] Dois exercicios sobre teoria dos grupos
 Oi. Sejam x, a, b elementos de um grupo G com a
 operacao .
 
 1) Mostre que a equacao x.a.x=b tem solucao num grupo
 G se e somente se a.b=g^2 para algum g em G
 
Seja m em G tal que mam= b.
Seja g = am.
Então, g^2 = (am)^2 = (am)(am) = a(mam) = ab.

Por outro lado, sejag em G tal que ab = g^2.
Seja n = a^(-1)g.
Então, nan = a^(-1)gaa^(-1)g = a^(-1)g^2 = a^(-1)ab = b.
Ou seja, x = n é solução de xax = b.


 2) Mostre que a equacao x^2.a.x=a^(-1) tem solucao em
 x num grupo G se e somente se a=g^3 para algum g em G
 
Seja m em G tal que m^2am = a^(-1) == mama = m^(-1).
Seja g = am.
Então g^3 = (am)^3 = a(mama)m = am^(-1)m = a.

Agora, seja g em G tal que a = g^3.
Seja n = g^(-2).
Então n^2an = g^(-4)g^3g^(-2) = g^(-3) = a ==
x = g^(-2)é solução de x^2ax = a^(-1).

[]s,
Claudio.




Re: [obm-l] SOMA(n = 1...inf) sen(n)/n

2005-10-17 Por tôpico claudio\.buffara
Excelente! Matou o problema. Muito obrigado.

[]s,
Claudio.





De:
[EMAIL PROTECTED]




Para:
obm-l@mat.puc-rio.br




Cópia:





Data:
Mon, 17 Oct 2005 20:45:30 -0200




Assunto:
Re: [obm-l] SOMA(n = 1...inf) sen(n)/n
 Claudio, espero que este link 
 http://web01.shu.edu/projects/reals/numser/answers/t_alter2.html
 possa ajudar.
 
 Um abraço
 
 claudio.buffara wrote:
  Oi, pessoal:
  
  Estou com a seguinte dúvida:
  A série SOMA(n = 1...inf) sen(n)/n converge?
  
  Mais geralmente, para que complexos z a série:
  SOMA(n = 1...inf) exp(nz)/n é convergente?
  
  []s,
  Claudio.
  
 
 
 -- 
 Niski - http://www.linux.ime.usp.br/~niski
 
 "sin^2(X) is odious to me, even thoug Laplace made use of it; shoud it
 be feared that sin^2(x) might become ambiguous, which would perhaps
 never occur ... well then, let us write (sin(x))^2, but not sin^2(X), which
 by analogy should signify sin(sin(x))"
 
 Carl Friedrich Gauss
 =
 Instruções para entrar na lista, sair da lista e usar a lista em
 http://www.mat.puc-rio.br/~nicolau/olimp/obm-l.html
 =
 


Re:[obm-l] Valor Esperado

2005-10-15 Por tôpico claudio\.buffara
Uma forma eh usar a funcao geratriz dos momentos:

F(t) = E[exp(tX)] = exp(m*t + s^2*t^2/2)

E[X^4] = derivada quarta de F(t) avaliada em t = 0

Eh bracal mas elementar...

Se nao errei nas contas, E[X^4] = m^4 + 6m^2*s^2 + 3*s^4

[]s,
Claudio.





De:
[EMAIL PROTECTED]




Para:
obm-l@mat.puc-rio.br, obm-l@mat.puc-rio.br




Cópia:





Data:
Fri, 14 Oct 2005 22:56:51 -0300




Assunto:
[obm-l] Valor Esperado
Seja X uma variável aleatória com distribuição normal (mi, sigma^2). Alguém pode me ajudar a calcular o valor esperado de X^4. Na verdade, preciso desse resultado para calcular a variância de X^2. Eu tentei usar a função gama mas acho que estou empacando na mudança de variável.
Obrigado.
Fabio



Re:RES: RES: [obm-l] Medida Positiva e Interior Vazio

2005-10-14 Por tôpico claudio\.buffara
OK. E se quisermos medida positiva, interior vazio, fechado e sem pontos isolados? Repare que, no exemplo abaixo, podemos ter dois intervalos abertos da forma (a,b) e (b,c), de modo que b seria um ponto isolado do complementar da união dos intervalos.
Será que dá pra escolher, para cada racional r_n, um intervalo aberto I_n tal que isso nunca ocorra?

[]s,
Claudio.





De:
[EMAIL PROTECTED]




Para:
obm-l@mat.puc-rio.br




Cópia:





Data:
Thu, 13 Oct 2005 17:23:02 -0300




Assunto:
RES: RES: [obm-l] Medida Positiva e Interior Vazio

 basta tomar o complementardaquele exemplo que vc deu.O complementar eh fechado, tem interior vazio e medida infinita
 Artur
 
 

-Mensagem original-De: [EMAIL PROTECTED] [mailto:[EMAIL PROTECTED]Em nome de claudio.buffaraEnviada em: quinta-feira, 13 de outubro de 2005 14:04Para: obm-lAssunto: Re:RES: [obm-l] Medida Positiva e Interior Vazio
 E se, além de medida positiva e interior vazio, exigirmos que o tal conjunto seja fechado?
 
 []s,
 Claudio.
 




De:
[EMAIL PROTECTED]




Para:
obm-l@mat.puc-rio.br




Cópia:





Data:
Thu, 13 Oct 2005 12:13:18 -0300




Assunto:
RES: [obm-l] Medida Positiva e Interior Vazio

  Na realidade, nos demos um exemplo ainda mais marcante: o de um conjunto aberto e denso em R mas com medida arbitrariamente proxima de zero.
  
  Um conjunto com medida infinita e interior vazio eh o dos irrracionais. Se quisermos medida finita e positiva, tomemos os irrracionais em [0, 1], Tem medida 1.
  
  A funcao de Thomae eh um exemplo de funcao continua so nos irracionais, certo? f(x) = 0 se x for irracional, f(x) =1 /n se x = m/n forracional, me n0 primos entre si. Agora, eu quero ver alguem dar um exemplo de funcao continua nos racionais e descontinua nos irracionais.
  
  Considremos agora f(x) =x/2 + (x^2)*(sen(1/x) se x0 e f(x) = 0 se x = 0. Entao f'(0) = lim (x - 0) (x/2 + (x^2)*(sen(1/x)))/x = lim (x - 0) 1/2 + x*sen(1/x) = 1/2  0.
  Temos que2*x*sen(1/x) = 0 quando x= 0 e que, em qualquer intervalo aberto do tipo (0, a),1/2 + cos(1/x) passa infinitas vezes pelos valores -1/2 e 3/2. de modo que, em qualquer intervalo contendo a origem, f tem uma infinidade de maximos e minimos relativos. Logo, f nao eh monotonica em nenhum destes intervalos.
  
  Isto ilustra que f'(a) 0)nao eh condicao suficiente para que a seja ponto de crescimento de f. Dizemos quea ehponto de crescimento de f se existir uma vizinhanca de a na qual f seja crescente.
  
  Artur
  ]-Mensagem original-De: [EMAIL PROTECTED] [mailto:[EMAIL PROTECTED]Em nome de claudio.buffaraEnviada em: quarta-feira, 12 de outubro de 2005 22:53Para: obm-lAssunto: [obm-l] Medida Positiva e Interior Vazio

  Oi, pessoal:
  
  Noutro dia o Artur pediu um exemplo de conjunto denso em R e de medida nula. Isso me lembrou de outro problema parecido:
  
  Dê um exemplo de subconjunto de R com medida positiva e interior vazio.
  
  Outros dois bonitinhos são: 
  Dê um exemplo de função real contínua nos irracionais e descontínua nos racionais.
  e
  Dê um exemplo de uma função real f derivável em todo ponto, tal que f'(0)  0 mas que não seja crescente em nenhum intervalo contendo a origem.
  
  No mais, alguém já descobriu por que um chicote estala quando é usado?
  
  []s,
  Claudio.
  


Re: [obm-l] RECORRENCIA

2005-10-14 Por tôpico claudio\.buffara
Outra forma, chegando diretamente à recorrência, é a seguinte:

Dada uma sequência com n-1 termos, teremos 3 possibilidades:

1. A sequência obedece às condições do enunciado:
Existem a(n-1) tais sequências e o n-ésimo termo pode ser escolhido de 5 maneiras distintas.
Total = 5*a(n-1)

2. A sequência não obedece às condições do enunciado:
2a) A sequência não contém nenhum2 nem nenhum 0:
Existem 3^(n-1) tais sequências e o n-ésimo termo tem que ser 2.
Total = 3^(n-1).

2b) A sequência não contém nenhum 2 mas contém algum 0:
Não importa qual seja o n-ésimo termo, esta sequência não dará origem a uma seqûencia válida.
Total = 0.

Assim, 
a(n-1) = 5*a(n-1) + 3^(n-1) ==

a(n-1) = 5*a(n-2) + 3^(n-2) ==
3^(n-1) = a(n) - 5*a(n-1) = 3*a(n-1) - 15*a(n-2) ==
a(n) - 8*a(n-1) + 15*a(n-2) = 0

Equação característica: t^2 - 8t + 15 = 0 ==
raízes: t = 3 e t = 5 ==
a(n) = P*3^(n-1) + Q*5^(n-1)

Claramente, a(1) = 1e a(2) = 8==
a(1) = P + Q = 1
a(2) = 3P + 5Q = 8 ==
P =-3/2e Q = 5/2 ==

a(n) = (5^n - 3^n)/2

[]s,
Claudio.





De:
[EMAIL PROTECTED]




Para:
obm-l@mat.puc-rio.br




Cópia:





Data:
Thu, 13 Oct 2005 22:49:54 -0300




Assunto:
Re: [obm-l] RECORRENCIA
 vamos faser o principio fundamental da contagem(PFC) separando em n casos.O primeiro eh quando o primeiro 2 aparece logo no 1º digito. Apos ele, podem aparecer todos os outros numeros( 0,1,2,3 ou 4)Logo há 5^(n-1) possibilidades. No segundo eh quando o primeiro 2 aparece no 2º digito. Antes dele so pode aparecer (1, 3 e 4. o 2 nao pra evitar dupla contatem). Apos ele, podem aparecer todos os outros numeros( 0,1,2,3 ou 4)Logo há 3.5^(n-2) e assim por diante...teremos que (A_n)=5^(n-1) + 3.5^(n-2) + 3².5^(n-3) + ... + 5².3^(n-3) + 5.3^(n-2) + 3^(n-1)Note que eh uma PG de primeiro termo 5^(n-1) e razao 3/5Resposta (i): (A_n)=(5^n - 3^n)/2 Se quiser deixar em termo de recorrencia: (A_n)=8(A_n-1) -15(A_n-2)com (A_0)=0 e (A_1)=1
 
 Logo, se quisermos deixar em funcao de (A_n) n e (A_n-1):
 Resposta (ii): (A_n)=8(A_n-1) -15(5^(n-2) - 3^(n-2))/2com (A_0)=0
 
 
 []'z Renato Lira.
 
 
 On 10/13/05, Adroaldo Munhoz [EMAIL PROTECTED] wrote:
Alguém resolveu esta?Abraços,Aldo
 Danilo Nascimento wrote:

 Seja ano numero de sequencias de n elementos, todos pertencentes ao conjunto {0,1,2,3,4} tais que:
 (i) há pelo menos um 2 na sequencia
 (ii) se houver um 0 na sequencia, deve haver pelo menos um 2 antes dele.
 Determine
 a) an em funcao de an-1 e n.
 b) an apenas em funcao de n.



Re: [obm-l] Medida Positiva e Interior Vazio

2005-10-14 Por tôpico claudio\.buffara





De:
[EMAIL PROTECTED]




Para:
obm-l@mat.puc-rio.br




Cópia:





Data:
Fri, 14 Oct 2005 12:09:39 -0300




Assunto:
Re: [obm-l] Medida Positiva e Interior Vazio
 Se vc está pensando no exemplo X que vai embolotando o n-ésimo racional
 com intervalos abertos de raio eps/(2^(n+1)) (na verdade, o complementar
 desse X), acho que basta pegar esse épsilon irracional; isso garante que
 não teremos coisas do tipo (a,b) (b,c). 

Não tenho tanta certeza. Acho que, para cada eps  0, existe alguma enumeração de Q tal que, para todo n,
r_n + eps/2^(n+1)  raiz(2)ou r_n - eps/2^(n+1)  raiz(2).
ComoQ é denso, o embolotamento correspondente conteria algo do tipo (a,raiz(2)) união (raiz(2),b).

[]s,
Claudio.

 então qualquer intervalo aberto contendo um ponto z do complementar 
 de X
 irá conter pontos de X, o q pela sua estrutura implicaria que algum r_n
 + eps/(2^(n+1)) está nesse intervalo, e como com o eps irracional não caímos
 no caso (a,b) (b,c), acho que dá pra garantir que esse ponto é diferente
 de z.
 
 []s,
 Daniel
 
 ''-- Mensagem Original --
 ''Date: Fri, 14 Oct 2005 07:47:49 -0300
 ''Subject: Re:RES: RES: [obm-l] Medida Positiva e Interior Vazio
 ''From: "claudio.buffara" <[EMAIL PROTECTED]>
 ''To: "obm-l" 
 ''Reply-To: obm-l@mat.puc-rio.br
 ''
 ''
 ''OK. E se quisermos medida positiva, interior vazio, fechado e sem pontos
 ''isolados? Repare que, no exemplo abaixo, podemos ter dois intervalos
 abertos
 ''da forma (a,b) e (b,c), de modo que b seria um ponto isolado do complementar
 ''da união dos intervalos.
 ''Será que dá pra escolher, para cada racional r_n, um intervalo aberto
 I_n
 ''tal que isso nunca ocorra?
 ''
 ''[]s,
 ''Claudio.
 ''
 ''De:[EMAIL PROTECTED]
 ''
 ''Para:obm-l@mat.puc-rio.br
 ''
 ''Cópia:
 ''
 ''Data:Thu, 13 Oct 2005 17:23:02 -0300
 ''
 ''Assunto:RES: RES: [obm-l] Medida Positiva e Interior Vazio
 ''
 '' basta tomar o complementardaquele exemplo que vc deu.O complementar
 eh
 ''fechado, tem interior vazio e medida infinita
 '' Artur
 ''
 ''
 ''-Mensagem original-
 ''De: [EMAIL PROTECTED] [mailto:[EMAIL PROTECTED]
 nome
 ''de claudio.buffara
 ''Enviada em: quinta-feira, 13 de outubro de 2005 14:04
 ''Para: obm-l
 ''Assunto: Re:RES: [obm-l] Medida Positiva e Interior Vazio
 ''
 ''
 '' E se, além de medida positiva e interior vazio, exigirmos que o tal
 conjunto
 ''seja fechado?
 ''
 '' []s,
 '' Claudio.
 ''
 '' De:[EMAIL PROTECTED]
 ''
 '' Para:obm-l@mat.puc-rio.br
 ''
 '' Cópia:
 ''
 '' Data:Thu, 13 Oct 2005 12:13:18 -0300
 ''
 '' Assunto:RES: [obm-l] Medida Positiva e Interior Vazio
 ''
 ''  Na realidade, nos demos um exemplo ainda mais marcante: o de um
 conjunto
 ''aberto e denso em R mas com medida arbitrariamente proxima de zero.
 '' 
 ''  Um conjunto com medida infinita e interior vazio eh o dos irrracionais.
 ''Se quisermos medida finita e positiva, tomemos os irrracionais em [0,
 1],
 ''Tem medida 1.
 '' 
 ''  A funcao de Thomae eh um exemplo de funcao continua so nos irracionais,
 ''certo? f(x) = 0 se x for irracional, f(x) =1 /n se x = m/n for racional,
 ''m e n0 primos entre si. Agora, eu quero ver alguem dar um exemplo
 de funcao
 ''continua nos racionais e descontinua nos irracionais.
 '' 
 ''  Considremos agora f(x) = x/2 + (x^2)*(sen(1/x) se x0 e f(x) =
 0 se
 ''x = 0. Entao f'(0) = lim (x - 0) (x/2 + (x^2)*(sen(1/x)))/x = lim (x
 -
 ''0) 1/2 + x*sen(1/x) = 1/2  0.
 ''  Temos que 2*x*sen(1/x) = 0 quando x= 0 e que, em qualquer intervalo
 ''aberto do tipo (0, a), 1/2 + cos(1/x) passa infinitas vezes pelos valores
 ''-1/2 e 3/2. de modo que, em qualquer intervalo contendo a origem, f
 tem uma
 ''infinidade de maximos e minimos relativos. Logo, f nao eh monotonica
 em nenhum
 ''destes intervalos.
 '' 
 ''  Isto ilustra que f'(a) 0) nao eh condicao suficiente para que
 a seja
 ''ponto de crescimento de f. Dizemos que a eh ponto de crescimento de
 f se
 ''existir uma vizinhanca de a na qual f seja crescente.
 '' 
 ''  Artur
 '' 
 ''] -Mensagem original-
 ''De: [EMAIL PROTECTED] [mailto:[EMAIL PROTECTED]
 nome
 ''de claudio.buffara
 ''Enviada em: quarta-feira, 12 de outubro de 2005 22:53
 ''Para: obm-l
 ''Assunto: [obm-l] Medida Positiva e Interior Vazio
 ''
 ''
 ''  Oi, pessoal:
 '' 
 ''  Noutro dia o Artur pediu um exemplo de conjunto denso em R e de
 medida
 ''nula. Isso me lembrou de outro problema parecido:
 '' 
 ''  Dê um exemplo de subconjunto de R com medida positiva e interior
 vazio.
 '' 
 ''  Outros dois bonitinhos são:
 ''  Dê um exemplo de função real contínua nos irracionais e descontínua
 nos
 ''racionais.
 ''  e
 ''  Dê um exemplo de uma função real f derivável em todo ponto, tal
 que f'(0)
 '' 0 mas que não seja crescente em nenhum intervalo contendo a origem.
 '' 
 ''  No mais, alguém já descobriu por que um chicote estala quando é
 usado?
 '' 
 ''  []s,
 ''  Claudio.
 '' 
 
 
 
 =
 Instruções para entrar na lista, sair da lista e usar a 

Re: [obm-l] RECORRENCIA

2005-10-14 Por tôpico claudio\.buffara





De:
[EMAIL PROTECTED]




Para:
obm-l@mat.puc-rio.br




Cópia:





Data:
Sat, 15 Oct 2005 01:18:44 + (GMT)




Assunto:
Re: [obm-l] RECORRENCIA
 Claudio,
  como que vc partiu a(n-1)=5*a(n-1)+3^(n-1) e chegou na recorrencia
 a(n) - 8*a(n-1) + 15*a(n-2) = 0. Nao entendi os passos q vc fez!
Meu engano! Deveria ser a(n) = 5*a(n-1) + 3^(n-1).

[]s,
Claudio.
"claudio.buffara" [EMAIL PROTECTED] escreveu:

 Outra forma, chegando diretamente à recorrência, é a seguinte:
 
 Dada uma sequência com n-1 termos, teremos 3 possibilidades:
 
 1. A sequência obedece às condições do enunciado:
 Existem a(n-1) tais sequências e o n-ésimo termo pode ser escolhido de 5 maneiras distintas.
 Total = 5*a(n-1)
 
 2. A sequência não obedece às condições do enunciado:
 2a) A sequência não contém nenhum2 nem nenhum 0:
 Existem 3^(n-1) tais sequências e o n-ésimo termo tem que ser 2.
 Total = 3^(n-1).
 
 2b) A sequência não contém nenhum 2 mas contém algum 0:
 Não importa qual seja o n-ésimo termo, esta sequência não dará origem a uma seqûencia válida.
 Total = 0.
 
 Assim, 
 a(n-1) = 5*a(n-1) + 3^(n-1) ==
 
 a(n-1) = 5*a(n-2) + 3^(n-2) ==
 3^(n-1) = a(n) - 5*a(n-1) = 3*a(n-1) - 15*a(n-2) ==
 a(n) - 8*a(n-1) + 15*a(n-2) = 0
 
 Equação característica: t^2 - 8t + 15 = 0 ==
 raízes: t = 3 e t = 5 ==
 a(n) = P*3^(n-1) + Q*5^(n-1)
 
 Claramente, a(1) = 1e a(2) = 8==
 a(1) = P + Q = 1
 a(2) = 3P + 5Q = 8 ==
 P =-3/2e Q = 5/2 ==
 
 a(n) = (5^n - 3^n)/2
 
 []s,
 Claudio.
 


Re:[obm-l] Ajuda - Complexos - Trigonometria

2005-10-13 Por tôpico claudio\.buffara





De:
[EMAIL PROTECTED]




Para:
obm-l@mat.puc-rio.br




Cópia:





Data:
Thu, 13 Oct 2005 10:13:14 + (GMT)




Assunto:
[obm-l] Ajuda - Complexos - Trigonometria
 Olá Senhores !
 
 Estou com dificuldade para resolver um problema do
 livro do Morgado e do Manfredo Perdigão, o livro da
 coleção do IMPA, sobre complexos e trigonometria.
 
 Seja AnAm a distância entre os pontos An e Am. 
 Seja o polígono regular de n lados, inscrito em uma
 circunferência de raio 1. Demonstre que:
 
 A1A2.A1A3.A1A4. ... .A1An = n
 
 Cheguei a relações trigonométricas interessantes,
 mas não consegui desenvolvê-las.
 
 Obrigado desde já pela ajuda!
 
 Celso Faria de Souza
 

Esse é um clássico...

As raízes da equação z^n - 1 = 0 (raízes n-ésimas da unidade) são precisamente os vértices de um n-gono regular, centrado na origem do plano complexo einscrito num círculo de raio 1. Uma dessas raízes é 1. As outras são w, w^2, , w^(n-1), onde w = cis(2pi/n).

Agora, z^n - 1 se fatora de duas maneiras distintas:
z^n - 1 = (z - 1)(z - w)(z - w^2)...(z - w^(n-1))
e
z^n - 1 = (z - 1)(z^(n-1) + z^(n-2) + ... + z + 1)

Isso quer dizer que:
(z - w)(z - w^2)...(z - w^(n-1)) = z^(n-1) + z^(n-2) + ... + z + 1

Tomando valores absolutos e fazendo z = 1 na identidade acima, você obtém o resultado desejado.

Aliás, outra maneira de fatorar z^n - 1 que dá origem a resultados interessantes leva em conta que w^k e w^(n-k) são complexos conjugados e, portanto, raízes de um polinômio quadrático de coeficientes reais:
(z - w^k)(z - w^(n-k)) = z^2 - (w^k + w^(n-k))z + 1 =
z^2 - (2cos(2kpi/n))z + 1

Assim, se n é ímpar == n = 2m+1 ==
z^n - 1 = (z - 1)*PRODUTO(k=1...m) (z^2 - (2cos(2pi/n))z + 1)

Se n é par == n = 2m ==
z^n - 1 = (z - 1)*(z + 1)*PRODUTO(k=1...m-1) (z^2 - (2cos(2pi/n))z + 1)

Por exemplo, fazendo z = -i nessas identidades você obtem o valor (real) de um produto de cossenos.

[]s,
Claudio.




Re:RES: [obm-l] Medida Positiva e Interior Vazio

2005-10-13 Por tôpico claudio\.buffara
E se, além de medida positiva e interior vazio, exigirmos que o tal conjunto seja fechado?

[]s,
Claudio.





De:
[EMAIL PROTECTED]




Para:
obm-l@mat.puc-rio.br




Cópia:





Data:
Thu, 13 Oct 2005 12:13:18 -0300




Assunto:
RES: [obm-l] Medida Positiva e Interior Vazio

 Na realidade, nos demos um exemplo ainda mais marcante: o de um conjunto aberto e denso em R mas com medida arbitrariamente proxima de zero.
 
 Um conjunto com medida infinita e interior vazio eh o dos irrracionais. Se quisermos medida finita e positiva, tomemos os irrracionais em [0, 1], Tem medida 1.
 
 A funcao de Thomae eh um exemplo de funcao continua so nos irracionais, certo? f(x) = 0 se x for irracional, f(x) =1 /n se x = m/n forracional, me n0 primos entre si. Agora, eu quero ver alguem dar um exemplo de funcao continua nos racionais e descontinua nos irracionais.
 
 Considremos agora f(x) =x/2 + (x^2)*(sen(1/x) se x0 e f(x) = 0 se x = 0. Entao f'(0) = lim (x - 0) (x/2 + (x^2)*(sen(1/x)))/x = lim (x - 0) 1/2 + x*sen(1/x) = 1/2  0.
 Temos que2*x*sen(1/x) = 0 quando x= 0 e que, em qualquer intervalo aberto do tipo (0, a),1/2 + cos(1/x) passa infinitas vezes pelos valores -1/2 e 3/2. de modo que, em qualquer intervalo contendo a origem, f tem uma infinidade de maximos e minimos relativos. Logo, f nao eh monotonica em nenhum destes intervalos.
 
 Isto ilustra que f'(a) 0)nao eh condicao suficiente para que a seja ponto de crescimento de f. Dizemos quea ehponto de crescimento de f se existir uma vizinhanca de a na qual f seja crescente.
 
 Artur
 ]-Mensagem original-De: [EMAIL PROTECTED] [mailto:[EMAIL PROTECTED]Em nome de claudio.buffaraEnviada em: quarta-feira, 12 de outubro de 2005 22:53Para: obm-lAssunto: [obm-l] Medida Positiva e Interior Vazio

 Oi, pessoal:
 
 Noutro dia o Artur pediu um exemplo de conjunto denso em R e de medida nula. Isso me lembrou de outro problema parecido:
 
 Dê um exemplo de subconjunto de R com medida positiva e interior vazio.
 
 Outros dois bonitinhos são: 
 Dê um exemplo de função real contínua nos irracionais e descontínua nos racionais.
 e
 Dê um exemplo de uma função real f derivável em todo ponto, tal que f'(0)  0 mas que não seja crescente em nenhum intervalo contendo a origem.
 
 No mais, alguém já descobriu por que um chicote estala quando é usado?
 
 []s,
 Claudio.
 


Re:[obm-l] Ajuda - Complexos - Trigonometria

2005-10-13 Por tôpico claudio\.buffara
Também dá pra provar (e sem usar complexos) que a soma dos quadrados dos comprimentos de A1A2, A1A3, ..., A1An é igual a 2n.

[]s,
Claudio.





De:
[EMAIL PROTECTED]




Para:
"obm-l" obm-l@mat.puc-rio.br




Cópia:





Data:
Thu, 13 Oct 2005 09:25:03 -0300




Assunto:
Re:[obm-l] Ajuda - Complexos - Trigonometria
 




De:
[EMAIL PROTECTED]




Para:
obm-l@mat.puc-rio.br




Cópia:





Data:
Thu, 13 Oct 2005 10:13:14 + (GMT)




Assunto:
[obm-l] Ajuda - Complexos - Trigonometria
  Olá Senhores !
  
  Estou com dificuldade para resolver um problema do
  livro do Morgado e do Manfredo Perdigão, o livro da
  coleção do IMPA, sobre complexos e trigonometria.
  
  Seja AnAm a distância entre os pontos An e Am. 
  Seja o polígono regular de n lados, inscrito em uma
  circunferência de raio 1. Demonstre que:
  
  A1A2.A1A3.A1A4. ... .A1An = n
  
  Cheguei a relações trigonométricas interessantes,
  mas não consegui desenvolvê-las.
  
  Obrigado desde já pela ajuda!
  
  Celso Faria de Souza
  
 
 Esse é um clássico...
 
 As raízes da equação z^n - 1 = 0 (raízes n-ésimas da unidade) são precisamente os vértices de um n-gono regular, centrado na origem do plano complexo einscrito num círculo de raio 1. Uma dessas raízes é 1. As outras são w, w^2, , w^(n-1), onde w = cis(2pi/n).
 
 Agora, z^n - 1 se fatora de duas maneiras distintas:
 z^n - 1 = (z - 1)(z - w)(z - w^2)...(z - w^(n-1))
 e
 z^n - 1 = (z - 1)(z^(n-1) + z^(n-2) + ... + z + 1)
 
 Isso quer dizer que:
 (z - w)(z - w^2)...(z - w^(n-1)) = z^(n-1) + z^(n-2) + ... + z + 1
 
 Tomando valores absolutos e fazendo z = 1 na identidade acima, você obtém o resultado desejado.
 
 Aliás, outra maneira de fatorar z^n - 1 que dá origem a resultados interessantes leva em conta que w^k e w^(n-k) são complexos conjugados e, portanto, raízes de um polinômio quadrático de coeficientes reais:
 (z - w^k)(z - w^(n-k)) = z^2 - (w^k + w^(n-k))z + 1 =
 z^2 - (2cos(2kpi/n))z + 1
 
 Assim, se n é ímpar == n = 2m+1 ==
 z^n - 1 = (z - 1)*PRODUTO(k=1...m) (z^2 - (2cos(2pi/n))z + 1)
 
 Se n é par == n = 2m ==
 z^n - 1 = (z - 1)*(z + 1)*PRODUTO(k=1...m-1) (z^2 - (2cos(2pi/n))z + 1)
 
 Por exemplo, fazendo z = -i nessas identidades você obtem o valor (real) de um produto de cossenos.
 
 []s,
 Claudio.
 
 


Re:RES: [obm-l] Medida Positiva e Interior Vazio

2005-10-13 Por tôpico claudio\.buffara
Agora, eu quero ver alguem dar um exemplo de funcao continua nos racionais e descontinua nos irracionais.

Eu também.

É claro que você pode trapacear e definir f: R - R como:
f(x) = 0, se x é racional
f(x) = 1, se x é irracional algébrico
f(x) = 2, se x é transcendente

Nesse caso, a restrição de f a Q é contínua por ser constante, mas a restrição de f a R - Q é descontínua em todo ponto, pois o conjunto dos algébricos irracionais é denso em R - Q. Infelizmente, não é isso quevocê está pedindo...

O problema é que o conjunto dos pontos onde uma função real é contínua pode ser expresso como uma interseccção no máximo enumerável de abertos.
Infelizmente, Q não pode ser expresso como uma tal intersecção (teorema de Baire)

[]s,
Claudio.



Re: RES: [obm-l] Medida Positiva e Interior Vazio

2005-10-13 Por tôpico claudio\.buffara





De:
[EMAIL PROTECTED]




Para:
obm-l@mat.puc-rio.br




Cópia:





Data:
Thu, 13 Oct 2005 17:20:24 + (GMT)




Assunto:
Re: RES: [obm-l] Medida Positiva e Interior Vazio
 Olá Artur,
 
 Não sei se vale esta, mas considere f(x) = 1/(x-p)^2,
 com p um número irracional. O único ponto onde f(x)
 não é analítica é p.

De fato, f não está nem definida em p,já que não podemos dividir por 0.

Embora ela cresça indefinidamente
 nos racionais também, não atinge a singularidade. Isto
 é, se adotarmos como definição de continuidade que
 f(x) seja derivável, então 1/(x-p)^2 é continua nos
 racionais e descontinua no irracionais. 

Não entendi o que você quis dizer com isso. Supondo que estamos trabalhando com funções reais, o domínio máximo de f é R - {p}.
Neste domínio, f é contínua, derivável e, de fato, analítica em cada ponto.

 Também os
 limites de f(x) são iguais à esquerda e à direita de
 p. 

f é ilimitada em qualquer vizinhança furada de p.

Porém apesar de continua, f(x) também não é
 limitada nos racionais... 
 
 []´s Demétrio
 
 --- Artur Costa Steiner <[EMAIL PROTECTED]>
 escreveu:
  Agora, eu quero ver alguem
  dar um exemplo de funcao
  continua nos racionais e descontinua nos
  irracionais.
  
 
  
  ] -Mensagem original-
  De: [EMAIL PROTECTED]
  [mailto:[EMAIL PROTECTED] nome de
  claudio.buffara
  Enviada em: quarta-feira, 12 de outubro de 2005
  22:53
  Para: obm-l
  Assunto: [obm-l] Medida Positiva e Interior Vazio
  
  
  
  Oi, pessoal:
  
  Noutro dia o Artur pediu um exemplo de conjunto
  denso em R e de medida nula.
  Isso me lembrou de outro problema parecido:
  
  Dê um exemplo de subconjunto de R com medida
  positiva e interior vazio.
  
  Outros dois bonitinhos são: 
  Dê um exemplo de função real contínua nos
  irracionais e descontínua nos
  racionais.
  e
  Dê um exemplo de uma função real f derivável em todo
  ponto, tal que f'(0) 
  0 mas que não seja crescente em nenhum intervalo
  contendo a origem.
  
  No mais, alguém já descobriu por que um chicote
  estala quando é usado?
  
  []s,
  Claudio.
  
  
  
 
 
 
 
 
 
 
 
 
 
 
 ___ 
 Promoção Yahoo! Acesso Grátis: a cada hora navegada você acumula cupons e concorre a mais de 500 prêmios! Participe! http://yahoo.fbiz.com.br/
 =
 Instruções para entrar na lista, sair da lista e usar a lista em
 http://www.mat.puc-rio.br/~nicolau/olimp/obm-l.html
 =
 


Re: [obm-l] Medida Positiva e Interior Vazio

2005-10-13 Por tôpico claudio\.buffara
Oi, Daniel:

Ou seja, você está dizendo que se (R - X)é uma união enumerável de intervalos abertos e é denso em R, então X é no máximo enumerável?

Eu tenho certeza de que você conhece um contra-exemplo famoso pra essa afirmação.

[]s,
Claudio.






De:
[EMAIL PROTECTED]




Para:
obm-l@mat.puc-rio.br




Cópia:





Data:
Thu, 13 Oct 2005 15:44:57 -0300




Assunto:
Re: [obm-l] Medida Positiva e Interior Vazio
 ''E se, além de medida positiva e interior vazio, exigirmos que o tal
 conjunto
 ''seja fechado?
 
 Se entendi direito, vc quer um conjunto A na reta com interior vazio, medida
 positiva (m(A)  0) e que seja fechado. Neste caso, acho que tal conjunto
 não existe; vai abaixo a minha tentativa de mostrar isso por absurdo.
 
 Podemos supor que A está contido em [0,1], já que a interseção de A com
 cada intervalo do tipo [n, n+1) com n inteiro dá uma soma disjunta enumerável
 igual a A, e como m(A)  0, algum desses caras tem que ter medida positiva.
 
 Seja X = (0,1) inter Ac (Ac = complementar de A). Segue que X é aberto,
 e portanto X é decomposto como uma união enumerável disjunta de intervalos
 do tipo I_n = (a_n, b_n), com a_n  a_(n+1). Se y está em (0,1) e b_n 
 y  a_(n+1), pela definição de X e ordenação dos I_n temos necessariamente
 que y está em A, logo [b_n, a_(n+1)] está contido em A. Sendo o interior
 de A vazio, isso implica que b_n = a_(n+1). Da mesma forma, a_1 = 0, e do
 fato de que o fecho de X é [0,1], temos que todo ponto de A é igual a algum
 a_n ou b_n, e então A é enumerável, não podendo portanto ter medida positiva.
 
 []s,
 Daniel
 
 
 =
 Instruções para entrar na lista, sair da lista e usar a lista em
 http://www.mat.puc-rio.br/~nicolau/olimp/obm-l.html
 =
 


Re:[obm-l] Limite estranho

2005-10-13 Por tôpico claudio\.buffara





De:
[EMAIL PROTECTED]




Para:
obm-l@mat.puc-rio.br




Cópia:





Data:
Thu, 13 Oct 2005 18:00:41 -0300




Assunto:
[obm-l] Limite estranho
 Olá a todos
 
 Na minha lista de cálculo tem um limite assim:
 
 lim x--0 de: (x-tan[x])/x^3
 
 Como estudamos L'Hôpital dias antes dessa lista, acredito que seja util. 
 No entanto a resposta encontrada pelos meus colegas (eu também) difere 
 da resposta gráfica.
 
 Obrigado
 Maurizio
 
(x-tg(x))/x^3 = 
(x*cos(x)-sen(x))/(x^3*cos(x)) =
(x*(1-x^2/2+O(x^4)) - (x-x^3/6+O(x^5))/(x^3*(1+O(x^2)) =
(-x^3/3 + O(x^5))/(x^3 + O(x^5)) =
(-1/3 + O(x^2))/(1 + O(x^2)) 

Logo, o limite procurado é -1/3.

Uma forma deconjecturar isso é usar uma calculadora e por x = 0,01.

[]s,
Claudio.




Re: [obm-l] Medida Positiva e Interior Vazio

2005-10-13 Por tôpico claudio\.buffara
O contra-exemplo que eu tinha em mente erao conjunto de Cantor.

[]s,
Claudio.





De:
[EMAIL PROTECTED]




Para:
obm-l@mat.puc-rio.br




Cópia:





Data:
Thu, 13 Oct 2005 18:51:24 -0300




Assunto:
Re: [obm-l] Medida Positiva e Interior Vazio
 O erro crucial foi ignorar o fato de que a união dos fechos pode ser diferente
 do fecho da união!
 
 []s,
 Daniel
 
 ''Ou seja, você está dizendo que se (R - X) é uma união enumerável de
 ''intervalos abertos e é denso em R, então X é no máximo enumerável?
 ''
 ''Eu tenho certeza de que você conhece um contra-exemplo famoso pra essa
 ''afirmação.
 ''
 ''Olá, Cláudio
 ''
 ''Como sempre, tens razão... Um :) dos meus erros foi ter suposto uma
 ''ordenação dos intervalos que decompõem o aberto de forma a se ter um
 ''intervalo com extremidade menor que todos os outros.
 ''
 ''O contra-exemplo (que eu não conhecia, apesar de ser bem simples) fica
 por
 ''conta da mensagem do Artur "embolotando" os racionais.
 
 
 
 =
 Instruções para entrar na lista, sair da lista e usar a lista em
 http://www.mat.puc-rio.br/~nicolau/olimp/obm-l.html
 =
 


[obm-l] Re: [obm-l] Ajuda Polinômios.

2005-10-12 Por tôpico claudio\.buffara
Sim. Veja lá na minha msg original (ou então abaixo).

Aliás, a fórmula de Lagrange pode não constar do programa do ensino médio mas está certamente ao alcance de um alunonormal deste nível.

Dê uma olhada em http://mathworld.wolfram.com/LagrangeInterpolatingPolynomial.html

[]s,
Claudio.





De:
[EMAIL PROTECTED]




Para:
obm-l@mat.puc-rio.br




Cópia:





Data:
Wed, 12 Oct 2005 10:43:51 -0300




Assunto:
Re: [obm-l] Ajuda Polinômios.
 Claúdio,
 
 A fórmula de interpolação de lagrange está acima do meu nível de
 escolaridade. Resolúvel de outra forma?
 
 Abraço,
 Roger.
 
 
 Em 11/10/05, Claudio Buffara<[EMAIL PROTECTED]> escreveu:
  on 11.10.05 00:27, Roger Lebid at [EMAIL PROTECTED] wrote:
 
   Bem pessoal estou com dificuldade em três questões de polinômios, acho
   que está faltando criatividade...
  
   ___
  
   1) Determinar todos os polinômios p(x) satisfazendo a equação:
   (x-16)p(2x)=16(x-1)p(x) para todo x.
  
  Estou supondo que trabalhamos sobre o corpo dos complexos.
 
  Se p(x) satisfaz, entao, para qualquer k complexo, k*p(x) tambem satisfaz.
 
  Assim, podemos supor que p(x) eh monico de grau n.
  Comparando os termos de maior grau em cada membro, obteremos:
  2^n*x^(n+1) = 16*x^(n+1) == n = 4
 
  Assim, seja p(x) = x^4 + ax^3 + bx^2 + cx + d.
 
  (x-16)p(2x) = (x-16)(16x^4 + 8ax^3 + 4bx^2 + 2cx + d) =
  16(x^5 + (a/2-16)x^4 + (b/4-8a)x^3 + (c/8-4b)x^2 + (d/16-2c)x - d) =
 
  16(x-1)p(x) = 16(x-1)(x^4 + ax^3 + bx^2 + cx + d) =
  16(x^5 + (a-1)x^4 + (b-a)x^3 + (c-b)x^2 + (d-c)x - d)
 
  Igualando os coeficientes dos termos de mesmo grau, obtemos:
  a = -30, b = 280, c = -960, d = 1024
 
  Logo, p(x) = k*(x^4 - 30x^3 + 280x^2 - 960x + 1024), onde k eh um complexo
  qualquer.
 
 
   2)Se p(x) denota um polinômio de grau n tal que P(k) = k/ (k+1) , para
   k = 0,1,2,...,n, calcular o valor de P(n+1)
  
  Sem usar nenhuma criatividade, basta usar a formula de interpolacao de
  Lagrange...
 
  Por outro lado, a identidade (k+1)P(k) = k == (k+1)P(k) - k = 0 sugere que
  consideremos o polinomio Q(x) = (x + 1)*P(x) - x, cujas raizes sao:
  0, 1, 2, ..., n.
 
  Ou seja, Q(x) = Ax(x-1)(x-2)...(x-n), onde A = constante a ser determinada.
 
  Q(-1) = (-1 + 1)P(-1) - (-1) = 1 ==
  A*(-1)^(n+1)*(n+1)! = 1 ==
  A = (-1)^(n+1)/(n+1)!
 
  Assim, Q(n+1) = A*(n+1)! = (-1)^(n+1) ==
 
  (n+2)P(n+1) - (n+1) = (-1)^(n+1) ==
 
  P(n+1) = (n + 1 + (-1)^(n+1))/(n + 2).
 
 
 
  []s,
  Claudio.
 
 


[obm-l] Medida Positiva e Interior Vazio

2005-10-12 Por tôpico claudio\.buffara
Oi, pessoal:

Noutro dia o Artur pediu um exemplo de conjunto denso em R e de medida nula. Isso me lembrou de outro problema parecido:

Dê um exemplo de subconjunto de R com medida positiva e interior vazio.

Outros dois bonitinhos são: 
Dê um exemplo de função real contínua nos irracionais e descontínua nos racionais.
e
Dê um exemplo de uma função real f derivável em todo ponto, tal que f'(0)  0 mas que não seja crescente em nenhum intervalo contendo a origem.

No mais, alguém já descobriu por que um chicote estala quando é usado?

[]s,
Claudio.



Re: [obm-l] Ajuda Polinômios.

2005-10-11 Por tôpico Claudio Buffara
on 11.10.05 00:27, Roger Lebid at [EMAIL PROTECTED] wrote:

 Bem pessoal estou com dificuldade em três questões de polinômios, acho
 que está faltando criatividade...
 
 ___
 
 1) Determinar todos os polinômios p(x) satisfazendo a equação:
 (x-16)p(2x)=16(x-1)p(x) para todo x.

Estou supondo que trabalhamos sobre o corpo dos complexos.

Se p(x) satisfaz, entao, para qualquer k complexo, k*p(x) tambem satisfaz.
 
Assim, podemos supor que p(x) eh monico de grau n.
Comparando os termos de maior grau em cada membro, obteremos:
2^n*x^(n+1) = 16*x^(n+1) == n = 4

Assim, seja p(x) = x^4 + ax^3 + bx^2 + cx + d.

(x-16)p(2x) = (x-16)(16x^4 + 8ax^3 + 4bx^2 + 2cx + d) =
16(x^5 + (a/2-16)x^4 + (b/4-8a)x^3 + (c/8-4b)x^2 + (d/16-2c)x - d) =

16(x-1)p(x) = 16(x-1)(x^4 + ax^3 + bx^2 + cx + d) =
16(x^5 + (a-1)x^4 + (b-a)x^3 + (c-b)x^2 + (d-c)x - d)

Igualando os coeficientes dos termos de mesmo grau, obtemos:
a = -30,  b = 280,  c = -960,  d = 1024
  
Logo, p(x) = k*(x^4 - 30x^3 + 280x^2 - 960x + 1024), onde k eh um complexo
qualquer.


 2)Se p(x) denota um polinômio de grau n tal que P(k) = k/ (k+1) , para
 k = 0,1,2,...,n, calcular o valor de P(n+1)

Sem usar nenhuma criatividade, basta usar a formula de interpolacao de
Lagrange...

Por outro lado, a identidade (k+1)P(k) = k == (k+1)P(k) - k = 0 sugere que
consideremos o polinomio Q(x) = (x + 1)*P(x) - x, cujas raizes sao:
0, 1, 2, ..., n.

Ou seja, Q(x) = Ax(x-1)(x-2)...(x-n), onde A = constante a ser determinada.

Q(-1) = (-1 + 1)P(-1) - (-1) = 1 ==
A*(-1)^(n+1)*(n+1)! = 1 ==
A = (-1)^(n+1)/(n+1)!

Assim, Q(n+1) = A*(n+1)! = (-1)^(n+1) ==

(n+2)P(n+1) - (n+1) = (-1)^(n+1) ==

P(n+1) = (n + 1 + (-1)^(n+1))/(n + 2).



[]s,
Claudio.


=
Instruções para entrar na lista, sair da lista e usar a lista em
http://www.mat.puc-rio.br/~nicolau/olimp/obm-l.html
=


Re: [obm-l] Conjunto aberto e denso com medida eps

2005-10-11 Por tôpico Claudio Buffara
Dada uma enumeracao {r_n} dos racionais da reta real, tome, para cada n, um
intervalo aberto de comprimento eps/2^(n+1) e centro em r_n.
Ponha A = uniao destes intervalos.

[]s,
Claudio.

on 11.10.05 13:45, Artur Costa Steiner at [EMAIL PROTECTED] wrote:

 Boa tarde,
 
 Eu acho este problema interessante:
 
 Sendo m a medida de Lebesgue, mostre que, para todo eps0, existe um
 subconjunto A de R, aberto e denso em R, com m(A)  eps.
 
 O que eu acho interessante nesta conclusao eh que ela mostra que,
 contrariamente ao que talvez seja intuitivo, nao hah uma correspondencia
 entra a significancia de um conjunto sob os pontos de vista topologico e
 de medidas. Podemos encontrar conjuntos que sejam abertos e densos em R,
 logo topologicamente significantes, mas cujas medidas possam ser
 arbitrariamente proximas de zero (embora nunca iguais a a zero).
 
 Por outro lado, o complementar de A, A', tem medida infinita e, no entanto,
 eh um fechado com interior vazio, logo seu fecho tem interior vazio. Assim,
 topologicamente A' nao eh significante,  visto ser um conjunto que nao eh
 denso em lugar nehum. Entretanto, A' tem medida infinita.
 
 Um outro exemplo mais simples e interessante e o conjunto dos racionais, que
 eh denso em R mas tem medida nula. Logo, sob o ponto de vists de medidas, eh
 um conjunto insignificante.
 
 Lembrando, se A eh um subconjunto de R, entao, em [0, infinito], m(A) =
 infimo {Soma L(I_n) | In esta em C}, onde L(I_n) eh o comprimento do
 intervalo I_n e o infimo eh calculado considerando todas as coberturas
 enumeraveis de A compostas por intervalos abertos. A medida de um intervalo
 eh o seu comprimento.
 
 Artur
 =
 Instruções para entrar na lista, sair da lista e usar a lista em
 http://www.mat.puc-rio.br/~nicolau/olimp/obm-l.html
 =
 


=
Instruções para entrar na lista, sair da lista e usar a lista em
http://www.mat.puc-rio.br/~nicolau/olimp/obm-l.html
=


Re: [obm-l] Area de um quadrado

2005-10-11 Por tôpico Claudio Buffara
Title: Re: [obm-l] Area de um quadrado



on 11.10.05 16:48, Rejane at [EMAIL PROTECTED] wrote:

Boa tarde a todos.
 
Poderiam me ajudar?
Como se calcula a área de um quadrado inscrito em um semi círculo?
 
Obrigada.


Pitagoras: L^2 + (L/2)^2 = R^2 == Area = L^2 = 4R^2/5, onde R = raio do semi-circulo.

[]s,
Claudio.





[obm-l] REFERENDO SOBRE VENDA DE ARMAS (off-topic)

2005-10-10 Por tôpico claudio\.buffara
Sou só eu ou alguém mais acha que uma quantidade considerável de eleitores vai votar errado no referendo?
Eu digo isso porque a pergunta é se o eleitor é favorável à proibição.
Assim, se ele for a favor da venda de armas, então deve votar "não" e, se for contra, deve votar "sim". Como nem todo mundo lida bem com duplas negações, acho que pode haver confusão.
Será que a pergunta foi formulada assim de propósito ou os organizadores do referendo são meio mancos, mesmo???

Desculpem o off-topic mas acho o assunto relevante até porque muitas das discussões aqui na lista são sobre enunciados mal-formulados.

[]s,
Claudio.


Re: [obm-l] RECORRENCIA

2005-10-09 Por tôpico Claudio Buffara
Title: Re: [obm-l] RECORRENCIA



Voce realmente nao estah conseguindo resolver estes problemas ou soh os estah propondo para os participantes da lista por acha-los interessantes? Veja bem, ambas as alternativas sao validas. Eu soh quero saber...

[]s,
Claudio.

on 09.10.05 03:12, Danilo Nascimento at [EMAIL PROTECTED] wrote:

Seja an o numero  de sequencias de n elementos, todos pertencentes ao conjunto {0,1,2,3,4} tais que:
(i) há pelo menos um 2 na sequencia
(ii) se houver um 0 na sequencia, deve haver pelo menos um 2 antes dele.
Determine
a) an em funcao de an-1 e n.
b) an apenas em funcao de n.

 Promoção Yahoo! Acesso Grátis: a cada hora navegada você acumula cupons e concorre a mais de 500 prêmios! Participe! http://us.rd.yahoo.com/mail/br/taglines/*http://yahoo.fbiz.com.br/ 






Re: [obm-l] A LINGUAGEM DA CIÊNCIA!

2005-10-08 Por tôpico Claudio Buffara
Tah bom, tah bom! Mas como era uma questao de fisica eu pensei como um
fisico (ou pior, como um engenheiro).

Alias, sobre estas questoes conceituais de fisica, a que eu acho mais legal
eh a seguinte: Por que um chicote (do tipo usado por domadores de leoes ou
pelo Indiana Jones) estala quando eh usado?

[]s,
Claudio.

on 08.10.05 06:07, Nicolau C. Saldanha at [EMAIL PROTECTED] wrote:

 On Fri, Oct 07, 2005 at 03:48:56PM -0200, Claudio Buffara wrote:
 Assim como uma xícara de chá quente perde calor mais rapidamente do que uma
 xícara de chá morno, seria, então, correto dizer que a xícara de chá quente
 esfriará até a temperatura ambiente antes da outra.
 
 O mais razoavel eh supor que a taxa de resfriamento (d(Temperatura)/dt) eh
 proporcional a diferenca de temperatura entre o cha e o meio ambiente.
 Assim:
 
 dTcha/dt = d(Tcha - Tamb)/dt = -k*(Tcha - Tamb) ==
 Tcha = Tamb + (Tcha(inicial) - Tamb)*exp(-k*t)
 
 Dai eh facil concluir que o cha morno chegarah antes a temperatura ambiente.
 
 Na verdade, com este modelo, a conclusão seria que nenhuma das duas
 jamais chega à temperatura ambiente (apesar de se aproximarem muito).
 
 []s, N.
 
 
 =
 Instruções para entrar na lista, sair da lista e usar a lista em
 http://www.mat.puc-rio.br/~nicolau/olimp/obm-l.html
 =
 


=
Instruções para entrar na lista, sair da lista e usar a lista em
http://www.mat.puc-rio.br/~nicolau/olimp/obm-l.html
=


Re: [obm-l] equações diofantinas

2005-10-08 Por tôpico Claudio Buffara
Title: Re: [obm-l] equações diofantinas



on 07.10.05 15:48, William Mesquita at [EMAIL PROTECTED] wrote:



alguem poderia me alguma dica sobre como esolver essas equações diofantinas 


1/a + 1/b + 1/c = 1 


Suponhamos inicialmente que 0  a = b = c.

Nesse caso, a = 3, pois se a = 4, entao:
1/a + 1/b + 1/c = 1/a + 1/a + 1/a = 3/4.

a = 2 == 1/b + 1/c = 1/2 == (b,c) = (3,6) ou (4,4)
Nao ha outras solucoes pois se b  4, entao 1/b + 1/c  1/2.

a = 3 == 1/b + 1/c = 2/3 == (b,c) = (3,3)
Nao ha outras solucoes pois se b  3, entao 1/b + 1/c  2/3.

-

Se a  0  b = c, entao 1/b + 1/c = 1 - 1/a  1.
Mas 1/b + 1/c  1 == b = 1 == c = -a.
Assim, as unicas solucoes sao da forma:
(a,b,c) = (-n,1,n) com n inteiro positivo.

-

Se a = b  0  c, entao 1/c = 1 - 1/a - 1/b  1 ==
impossivel.

Assim, supondo que a = b = c, as solucoes (a,b,c) sao:
(2,3,6), (2,4,4), (3,3,3) e (-n,1,n) com n inteiro positivo.

As demais solucoes sao obtidas permutando a, b e c.


***

x^3 + 3 = 4y(y+1) ==

x^3 + 4 = 4y^2 + 4y + 1 = (2y + 1)^2 ==
x^3 = (2y + 1)^2 - 2^2 = (2y - 1)(2y + 3)

Mas mdc(2y - 1,2y + 3) = mdc(2y - 1,4) = 1 ==
2y - 1 e 2y + 3 sao ambos cubos perfeitos impares que diferem de 4 ==
contradicao, pois dois cubos perfeitos impares diferem de pelo menos 26 
(=3^3 - 1^3)

Logo, esta equacao nao tem solucao.


[]s,
Claudio.






Re: [obm-l] FUNCAO

2005-10-08 Por tôpico Claudio Buffara
Title: Re: [obm-l] FUNCAO



on 08.10.05 21:10, Danilo Nascimento at [EMAIL PROTECTED] wrote:

Seja f: R--R uma funcao tal que f(x+y)=f(x).f(y) para todos x, y pertencente a R e f nao é identicamente nula. Considere g(x) = (f(3x)-f(2x)) / (1+f(2x)f(3x)). Mostre que g é impar.

f(x) = f(x+0) = f(x)f(0) ==
f(x)(1 - f(0)) = 0, para todo x real ==
f(0) = 1, pois x pode ser escolhido tal que f(x)  0.

1 = f(0) = f(x + (-x)) = f(x)f(-x) ==
f(x)  0 para todo x real e f(-x) = 1/f(x)

f(2x) = f(x)^2 e f(3x) = f(x)^3 ==

g(x) = 
(f(x)^3 - f(x)^2)/(1 + f(x)^5) = 
f(x)^2*(f(x) - 1)/(1 + f(x)^5)

g(-x) = 
f(-x)^2*(f(-x) - 1)/(1 + f(-x)^5) = 
(1/f(x)^2)*(1/f(x) - 1)/(1 + 1/f(x)^5) =
(1/f(x)^3 - 1/f(x)^2)/(1 + 1/f(x)^5) =
(f(x)^2 - f(x)^3)/(f(x)^5 + 1) =
-g(x) ==

g eh impar.


[]s,
Claudio.





Re: [obm-l] A LINGUAGEM DA CIÊNCIA!

2005-10-07 Por tôpico Claudio Buffara
on 07.10.05 10:34, Jorge Luis Rodrigues e Silva Luis at
[EMAIL PROTECTED] wrote:

 Turma! Como campeão de artigos off's, me sinto na obrigação de defender esse
 casamento da Matemática com a Física. Elas são guias para o pensamento,
 mostrando as conexões entre os conceitos sobre a natureza. O método
 matemático e a experimentação levaram a ciência a um enorme sucesso. Eis
 abaixo uma oportunidade ímpar dos matemáticos participarem deste sucesso
 conjugal.
 
 Assim como uma xícara de chá quente perde calor mais rapidamente do que uma
 xícara de chá morno, seria, então, correto dizer que a xícara de chá quente
 esfriará até a temperatura ambiente antes da outra.
 
O mais razoavel eh supor que a taxa de resfriamento (d(Temperatura)/dt) eh
proporcional a diferenca de temperatura entre o cha e o meio ambiente.
Assim:

dTcha/dt = d(Tcha - Tamb)/dt = -k*(Tcha - Tamb) ==
Tcha = Tamb + (Tcha(inicial) - Tamb)*exp(-k*t)

Dai eh facil concluir que o cha morno chegarah antes a temperatura ambiente.

 Uma barcaça de rio, carregada com cascalho, aproxima-se de uma ponte baixa,
 sob a qual não pode passar. Dever-se-ia remover ou adicionar cascalho à
 barcaça?

Supono que o cascalho eh mais denso do que a agua, devemos adicionar
cascalho, pois se a densidade relativa eh d  1 entao, ao aumentarmos a
altura da pilha de cascalho em 1 metro, a barcaca afundarah d metros.
 
 No vácuo, uma moeda e uma pena caem igualmente, lado a lado. Seria correto
 dizer que iguais forças da gravidade atuam sobre a moeda e a pena quando
 estão no vácuo?

Nao. O que sao iguais sao as aceleracoes. A forca da gravidade serah tanto
maior quanto maior for a massa do objeto.
 
 Que equipe ganhará um cabo-de-guerra: aquela que puxa mais fortemente a
 corda ou aquela que empurra mais fortemente contra o solo?

Aquela que empurra o solo com mais forca. Supondo que a massa da corda eh
desprezivel face a massa total de cada time, a forca resultante na corda
serah sempre zero, ou seja, os times sempre puxarao a corda com a mesma
forca. Mas se o Time 1 empurrar o solo com mais forca do que o Time 2, o
solo exercerah uma forca (reacao) resultante sobre o sistema (Time 1 + Time
2 + corda) na direcao do Time 1, fazendo com o sistema se desloque nesta
direcao, o que ocasionarah a vitoria deste time.
 
 Quando um iôiô desce rolando até o fim do barbante, ele inverte sua rotação
 quando sobe de volta? Explique.

Boa essa. Infelizmente estou sem um ioio aqui pra fazer a experiencia.

 A propósito, por que o gelo muito frio fica grudento?

Eu diria que eh porque, ao entrar em contato com algum objeto com uma
temperatura maior do que 0 graus C (por exemplo, nossos dedos), uma fina
camada de gelo na superficie do bloco derrete mas, em seguida, re-congela e
adere a superficie mais quente. Mas eh soh chute...
 
[]s,
Claudio.


=
Instruções para entrar na lista, sair da lista e usar a lista em
http://www.mat.puc-rio.br/~nicolau/olimp/obm-l.html
=


Re:RES: RES: [obm-l]

2005-10-04 Por tôpico claudio\.buffara
Pois é.É só normalizar, pondo b_i = a_i/(k*p^(1/n)), que caímos no problema original.





De:
[EMAIL PROTECTED]




Para:
obm-l@mat.puc-rio.br




Cópia:





Data:
Tue, 4 Oct 2005 11:49:38 -0300




Assunto:
RES: RES: [obm-l]

 Na realidade, complica muito pouco. Pelo produto de Stevin e MA =MG dah facilmente pra ver que, tambem neste caso mais geral,o minimo ocorre quando os a_i sao iguias.
 
 Artur
 

-Mensagem original-De: [EMAIL PROTECTED] [mailto:[EMAIL PROTECTED]Em nome de Artur Costa SteinerEnviada em: terça-feira, 4 de outubro de 2005 11:18Para: obm-l@mat.puc-rio.brAssunto: RES: RES: [obm-l]
 Esta solucao ficou bem legal! bem mais interessante que usando calculo.
 
 Mas e tivessemos algo mais geral do tipo, 
 
 minimizar (k + a_1).(k +_a_n), k0
 
 dado que a_1 * a_2 *.a_n = p
 
 a_1,...a_n 0, entao acho que a solucao algebrica ia complicar, embora talvez ainda de pra sair pelo produto de Stevin e desigualdade MA = MG.A solucao otima continuria sendo com a_1 = a_n = p^(1/n).
 
 Neste caso ate que nao eh tao dificil analisar.Soh hah um ponto extremo, a funcao eh limitada inferiormente, ela e a as restricoes sao classe C^2
 
 Artur
 
 -Mensagem original-De: [EMAIL PROTECTED] [mailto:[EMAIL PROTECTED]Em nome de claudio.buffaraEnviada em: segunda-feira, 3 de outubro de 2005 20:03Para: obm-lAssunto: RE: RES: [obm-l]

 Talvez um enunciado mais claro pro problema original seja o seguinte:
 
 Se a_1, a_2, ..., a_n são reais positivos quaisquer cujo produto é 1, então:
 (1 + a_1)*(1 + a_2)*...*(1 + a_n)= 2^n
 e vale a igualdade se e somente se a_i = 1 para 1 = i = n.
 
 Agora, sabemos que se o produto dem números positivos for 1, então a soma desses números é =m com igualdade se e somente se todos os números são iguais (isso é simplesmente a desigualdade MA = MG).
 
 Expandindo o lado esquerdo, teremos:
 1 + S_1 + S_2 + ... + S_(n-1) + S_n, onde:
 S_k = soma dos produtos dos a_i tomados k a k.
 Assim, S_1 = a_1 + a_2 + ... + a_n,
 S_2 = a_1*a_2 + a_1*a_3 + ... + a_(n-1)*a_n
 ...
 S_n = a_1*a_2*...*a_n.
 
 É fácil ver que S_k possui Binom(n,k) parcelas, cujo produto é 1, de modo que S_k= Binom(n,k).
 
 Assim, o lado esquerdo é maior ou igual que:
 1 + Binom(n,1) + Binom(n,2) + ... + Binom(n,n) = 2^n.
 
 Finalmente, vale a igualdade ==
 S_1 = Binom(n,1) = n ==
 a_1 = ... = a_n.
 
 []s,
 Claudio.
 
 
 




De:
[EMAIL PROTECTED]




Para:
obm-l@mat.puc-rio.br




Cópia:





Data:
Mon, 3 Oct 2005 19:02:31 -0300




Assunto:
RE: RES: [obm-l]
  Tem razão, Artur... eu tava tão descontente com essa "solução" que nem exigi
  muito dela. Em todo caso, não sei quase nada deste assunto.
  
  []s,
  Daniel
  
  ''Esta solucao foi tambem a unica que me ocorreu. Soh que, na realidade,
  o
  ''problema nao se encerra no ponto em que vc parou. Os multiplicadores
  de
  ''Lagrande mostram de fato que y_1 = ... = y_n = 1 PODE, mas nao
  ''necessriamente TEM, que ser um ponto extremo. De modo geral, para se
  decidir
  ''se eh mesmo um ponto extremo e, se sendo de fato ponto extremo, eh
  maximo
  ''ou minimo relativo, temos que analisar condicoes de segunda ordem, no
  caso
  ''em que o problema, como este, tem funcao objetivo e restricoes com derivadas
  ''parciais de segunda ordem continuas (classe C^2). Além disto, precisamos
  ''garantir que eh minimo global, nao apenas local. Isto, de modo geral,
  exige
  ''condicoes de convexidade ou concavidade.
  ''Na programacao matematica hah um terorema que se aplica a casos como
  este,
  ''em que a funcao objetivo e as restricoes apresentam simetria. Nao me
  lembro
  ''dos detalhes, mas acho que nestes casos dah pra garantir que o ponto
  eh
  ''maximo ou minimo global.
  ''
  ''Artur
  
  
  =
  Instruções para entrar na lista, sair da lista e usar a lista em
  http://www.mat.puc-rio.br/~nicolau/olimp/obm-l.html
  =
  


Re:[obm-l] Funcao de Lipschitz

2005-10-04 Por tôpico claudio\.buffara





De:
[EMAIL PROTECTED]




Para:
"OBM-l (E-mail)" obm-l@mat.puc-rio.br




Cópia:





Data:
Tue, 4 Oct 2005 12:16:28 -0300




Assunto:
[obm-l] Funcao de Lipschitz
 Eu acho esses problemas sobre funcoes de Lipschitz bonitinhos:
 
 (1) - seja D um subconjunto de R^n e f:D = R^m Lipschitz em D. Mostre que
 (a) Existe uma menor constante de Lipschitz associada a f em D. 

Seja A = {K em R | |f(x) - f(y)| = K*|x - y| para todos x e y em D}.
É claro que A é limitado inferiormente (por 0, por exemplo) de modo que existe L = inf(A).

Se f for constante, então L = 0. Assim, suponhamos que f não é constante e, em particular, que D tem mais do que um elemento. Isso quer dizer que L  0.

Suponhamos que L não pertence a A, ou seja, que A = (L,+infinito).
Então existema eb em D tais que, para todo eps  0:
0  L*|b - a|  |f(b) - f(a)|= (L + eps)*|b - a|

Como eps é arbitrário, isso quer dizer que:
0  L*|b - a|  |f(b) - f(a)| = L*|b - a| == 
contradição ==
L pertence a A.


 (b) Se K eh esta menor constante, entao, para todo eps 0, existem x1 e  x2x1 em D tais
 que ||f(x2) - f(x1)|/|x2 - x1| - K|  eps 

Dado eps  0, existem x e y em D tais que x  y e:
(K - eps)*|x - y|  |f(x) - f(y)|  (K + eps)*|x - y| ==
K - eps  |f(x) - f(y)|/|x - y|  K + eps ==
||f(x) - f(y)|/|x - y| - K|  eps 


(c) Se K eh constante de Lipschitz
 de f em D e existirem x1x2 em D tais que:
|f(x2) - f(x1)| = K*|x2 - x1| ,
 entao K eh a menor constante de Lipschitz de f em D. A reciproca eh
 verdadeira?
 
K é constante de Lipschitz mas, para todo eps  0, teremos:
|f(x2) - f(x1)| = K*|x2 - x1|  (K - eps)*|x2 - x1| ==
K - eps não é constante de Lipschitz ==
K é a menor constante de Lipschitz de f em D.

A recíproca não vale.
Seja f:(1,+infinito) - R dada por f(x) = raiz(x).
Então, dados x y em (1,+infinito), teremos:
raiz(y) - raiz(x) = (y - x)/(raiz(y) + raiz(x))  (y - x)/2, 
de modo que f é Lipschitz com constante 1/2.
No entanto, não existem x e y distintos em (1,+infinito) tais que:
|raiz(y) - raiz(x)| = (1/2)*|y - x|, 
pois dividindo ambos os membros por |raiz(y) - raiz(x)|, obteremos:
raiz(y) + raiz(x) = 2, o que é impossível com x e y em (1,+infinito).



 (2) Sejam I um intervalo de R e f:I = R derivavel em I. Entao, f eh
 Lipschitz em I se, e somente se, f' for limitada em I, caso em que K
 =supremo {|f'(x)| | x estah em I} eh a menor constante de Lipschitz de f em
 I.
 
Se |f'(x)| = M para todo x em I, então, dados x  y em I, pelo TVM existirá z tal que x  z  y e |f(y) - f(x)| = |f'(z)|*|y - x| = M*|y - x| == f é Lipschitz em I com constante M

Reciprocamente, se f é Lipschitz em I com constante K, então, dado a em I, para todo x em I - {a} teremos- K = (f(x) - f(a))/(x - a) = K ==
-K = lim(x - a) (f(x) - f(a))/(x - a) = K (limites laterais se a for um dos extremos de I) == -K = f'(a) = K == |f'(a)| = K. Como a é qualquer, o resultado segue.

Seja K = supremo {|f'(x)| | x estah em I}.
Então, pelo TVM,é claro que f é Lipschitz com constante K.
Dado L com 0  L  K, existea em I tal que |f'(a)|  L.
Isso quer dizer que existe delta  0 tal que:
x pertence a I e 0  |x - a|  delta == |(f(x) - f(a))/(x - a)|  L
Ou seja, |f(x) - f(a)|  L*|x - a| == L não é constante de Lipschitz para f.

Acho que o mais interessante desse problema éque ele ilustrauma das propriedades mais importantes e úteis dos limites: a permanência das desigualdades.

[]s,
Claudio.

 
 Lembrando, f eh Lipschitz em D se existir uma constante K0 tal que |f(x2)
 - f(x1)| = K*|x2 - x1| para todos x1 e x2 de D. Eh imediatpo que se K for
 constante de Lipschitz, entao todo K'  K tambem eh.
 
 Artur 
 


RE: RES: [obm-l]

2005-10-03 Por tôpico claudio\.buffara
Talvez um enunciado mais claro pro problema original seja o seguinte:

Se a_1, a_2, ..., a_n são reais positivos quaisquer cujo produto é 1, então:
(1 + a_1)*(1 + a_2)*...*(1 + a_n)= 2^n
e vale a igualdade se e somente se a_i = 1 para 1 = i = n.

Agora, sabemos que se o produto dem números positivos for 1, então a soma desses números é =m com igualdade se e somente se todos os números são iguais (isso é simplesmente a desigualdade MA = MG).

Expandindo o lado esquerdo, teremos:
1 + S_1 + S_2 + ... + S_(n-1) + S_n, onde:
S_k = soma dos produtos dos a_i tomados k a k.
Assim, S_1 = a_1 + a_2 + ... + a_n,
S_2 = a_1*a_2 + a_1*a_3 + ... + a_(n-1)*a_n
...
S_n = a_1*a_2*...*a_n.

É fácil ver que S_k possui Binom(n,k) parcelas, cujo produto é 1, de modo que S_k= Binom(n,k).

Assim, o lado esquerdo é maior ou igual que:
1 + Binom(n,1) + Binom(n,2) + ... + Binom(n,n) = 2^n.

Finalmente, vale a igualdade ==
S_1 = Binom(n,1) = n ==
a_1 = ... = a_n.

[]s,
Claudio.







De:
[EMAIL PROTECTED]




Para:
obm-l@mat.puc-rio.br




Cópia:





Data:
Mon, 3 Oct 2005 19:02:31 -0300




Assunto:
RE: RES: [obm-l]
 Tem razão, Artur... eu tava tão descontente com essa "solução" que nem exigi
 muito dela. Em todo caso, não sei quase nada deste assunto.
 
 []s,
 Daniel
 
 ''Esta solucao foi tambem a unica que me ocorreu. Soh que, na realidade,
 o
 ''problema nao se encerra no ponto em que vc parou. Os multiplicadores
 de
 ''Lagrande mostram de fato que y_1 = ... = y_n = 1 PODE, mas nao
 ''necessriamente TEM, que ser um ponto extremo. De modo geral, para se
 decidir
 ''se eh mesmo um ponto extremo e, se sendo de fato ponto extremo, eh
 maximo
 ''ou minimo relativo, temos que analisar condicoes de segunda ordem, no
 caso
 ''em que o problema, como este, tem funcao objetivo e restricoes com derivadas
 ''parciais de segunda ordem continuas (classe C^2). Além disto, precisamos
 ''garantir que eh minimo global, nao apenas local. Isto, de modo geral,
 exige
 ''condicoes de convexidade ou concavidade.
 ''Na programacao matematica hah um terorema que se aplica a casos como
 este,
 ''em que a funcao objetivo e as restricoes apresentam simetria. Nao me
 lembro
 ''dos detalhes, mas acho que nestes casos dah pra garantir que o ponto
 eh
 ''maximo ou minimo global.
 ''
 ''Artur
 
 
 =
 Instruções para entrar na lista, sair da lista e usar a lista em
 http://www.mat.puc-rio.br/~nicolau/olimp/obm-l.html
 =
 


Re: [obm-l] f((x+y)/2) = (f(x) + f(y))/2

2005-09-30 Por tôpico Claudio Buffara
on 29.09.05 15:45, Artur Costa Steiner at [EMAIL PROTECTED] wrote:

 Oi a todos,
 
 Supomhamos que f seja seja definida em um intervalo aberto I e que satisfaca
 a f((x+y)/2) = (f(x) + f(y))/2 para todos x e y de I. Sabemos que se f for
 continua, entao f eh convexa em I. Sabemos tambem que se f for Lebesgue
 mensuravel, entao f eh continua e, portanto, convexa.
 
 Eu nao estou certo e ainda nao consegui chegar a uma conclusao, mas me
 parece que, se alem de satisfazer aa dada equacao funcional, f for limitada
 em I, entao f eh continua e, portanto, convexa. Alguem ja tentou demonstrar
 isto? (foi-me afirmado que isto eh verdade, mesmo que I seja ilimitado).
 
 Artur
 =

Oi, Artur:

No livro Analise Real - vol. 1 do Elon ha uma secao que trata exclusivamente
de funcoes convexas e prova a maioria dos resultados basicos sobre o
assunto.

[]s,
Claudio.

=
Instruções para entrar na lista, sair da lista e usar a lista em
http://www.mat.puc-rio.br/~nicolau/olimp/obm-l.html
=


Re: [obm-l] sistema de congruencias

2005-09-29 Por tôpico Claudio Buffara
on 28.09.05 21:48, Adroaldo Munhoz at [EMAIL PROTECTED] wrote:

 Olá pessoal,
 
 Como eu resolvo o sistema de congruências abaixo:
 
 x==0 (mod 5)
 x==6 (mod 7)
 x==7 (mod 9)
 x==8 (mod 11)
 
 Abraços,
 
 Aldo

x == 8 (mod 11) ==
x = 8 + 11a ==

x == 7 (mod 9) ==
8 + 11a == 7 (mod 9) ==
2a == 8 (mod 9) ==
a == 4 (mod 9) ==
x = 8 + 11(4 + 9b) = 52 + 99b

x == 6 (mod 7)
52 + 99b == 6 (mod 7) ==
b == 3 (mod 7) ==
x = 52 + 99(3 + 7c) = 349 + 693c

x == 0 (mod 5) ==
349 + 693c == 0 (mod 5) ==
3c == 1 (mod 5) ==
c == 2 (mod 5) ==
x = 349 + 693(2 + 5d) = 1735 + 3465d ==

x == 1735 (mod 3465)

Ou entao use o teorema chines dos restos - veja qualquer livro de teoria dos
numeros.

[]s,
Claudio.


=
Instruções para entrar na lista, sair da lista e usar a lista em
http://www.mat.puc-rio.br/~nicolau/olimp/obm-l.html
=


Re: [obm-l] Duvidas

2005-09-29 Por tôpico Claudio Buffara
Title: Re: [obm-l] Duvidas



Sejam:
r = raio do circulo menor;
d = distancia do vertice do angulo ao centro do circulo menor.

Semelhanca de triangulos retangulos:
cateto oposto a x / hipotenusa = r/d = R/(R+r+d) = sen(x)

Usando razoes e proporcoes:
r/d = (R-r)/(R+r)

Logo:
r = R*(1 - sen(x))/(1 + sen(x))

[]s,
Claudio.

on 29.09.05 08:20, matduvidas48 at [EMAIL PROTECTED] wrote:

 
 
1.Dois círculos são tangentes entre si e aos lados de um ângulo dado 2x. Conhecendo o raio R do maior , calcular o raio do círculo manor.
 
 
(essa queatão é do livro da SBM -Trigonomtria e numeros complexos).
 
Tentei fazer mais , não saiu,alguem poderia me ajudar.
 
Agradeço desde de já.
 
 
Ary Queiroz







Re:[obm-l] Problemas de Teoria de Numeros

2005-09-28 Por tôpico claudio\.buffara





De:
[EMAIL PROTECTED]




Para:
obm-l@mat.puc-rio.br




Cópia:





Data:
Wed, 28 Sep 2005 07:46:35 -0300




Assunto:
[obm-l] Problemas de Teoria de Numeros
 Pessoal, estou com alguns problemas de Teoria de Números que não sei 
 como resolver.
 
 1. Provar que para p primo (p-1)!=p-1 (mod 1+2+3+...+(p-1))

módulo = p(p-1)/2
Obviamente, (p-1)! == p-1 ( == 0 ) (mod (p-1)/2)
T. de Wilson == (p-1)! == p-1 (mod p)
Logo, (p-1)! == p-1 (mod p(p-1)/2).

 2. Encontrar o máximo divisor comum de (p-1)!-1 e p!, com p primo.

p divide ambos e, além disso, p^2 não divide p!.
Qualquer primo maior do que p não divide p!
Qualquer primo menor do quep não divide (p-1)! - 1.
Logo, mdc = p.

 3. Mostrar que para n=4 o resto da divisão por 12 de 1!+2!+3!+...+n! é 9.

Para n = 4, n! é divisível por 12.
Logo, Soma == 1! + 2!+ 3! == 9 (mod 12).

 4. Mostrar que para todo n inteiro 3n^2-1 nunca é um quadrado.

Um quadrado só pode ser == 0 ou 1 (mod 3), pois:
k == 0, 1, 2 (mod 3) == k^2 == 0, 1, 1 (mod 3), respectivamente.
3n^2 - 1 == 2 (mod 3). Logo, não pode ser quadrado.
3n^2 - m^2 = 1 ==

 5. Mostrar que 5n^3+7n^5=0 (mod 12) para todo n.

Usando pequeno Fermat e propriedades das congruências, teremos:
Mod 3: 5n^3 + 7n^5 == 2n + n = 3n == 0
Mod 4: 5n^3 + 7n^5 == n^3 - n^5 = -n^3(n - 1)(n + 1) == 0, pois se n for par, então 8 | n^3 e se n for ímpar então n-1 e n+1 serão pares.

 6. Seja f(x)=a_0+a_1x+...+a_nx^n um polinômio com coeficientes inteiros 
 onde a_n0 e n=1. Mostrar que f(x) é composto para infinitos valores da 
 variável x.

Suponhamos que f(c) = p = primo, para algum inteiro c (se um tal c não existir, então acabou!).
f(x) - f(c) = (x - c)*g(x), para um certo g(x) ==
f(x) = (x - c)*g(x) + p.
Além disso, como a_n  0, g(x)  0 para x suficientemente grande.

Tome x = t*p - c, com t inteiro.
Então, f(t*p - c) = p*(t*g(t*p - c) + 1).

Para todo t suficientemente grande, t*g(t*p - c) + 1  1 ==
f(t*p - c) = múltiplo de p = composto.

 7. Mostrar que para a e b inteiros, com (a, b)=1 temos a^fi(b)+b^fi(a)=1 
 (mod ab)
 
T. de Euler == a^fi(b) == 1 (mod a) e b^fi(a) == 1 (mod b).
Logo, (a^fi(b) - 1)(b^fi(a) - 1) == 0 (mod ab) ==
a^fi(b)*b^fi(a) - (a^fi(b) + b^fi(a)) + 1 == 0 (mod ab) ==
Mas a^fi(b)*b^fi(a) == 0 (mod ab), donde segue o resultado desejado.


[]s,
Claudio.



Re: [obm-l] ajuda em diofantina

2005-09-28 Por tôpico claudio\.buffara
Reduzindo a 1a. equação mod 13, teremos:
x^13 - x == 1 (mod 13).
Mas o pequeno teorema de Fermat implica que x^13 - x == 0 (mod 13) para todo x inteiro.
Logo, a congruência não tem solução e, portanto, com mais razão ainda, a equação diofantina não tem solução.

Reduzindo a 2a. equação mod7 teremos:
x^2==3 (mod 5)
Mas se n == 0, 1, 2, 3, 4, 5, 6 (mod 7), então:
n^2 == 0, 1, 4, 2, 2, 4, 1 (mod 7).
Ou seja, nenhum quadrado é == 3 (mod 7).
Mesma conclusão.

[]s,
Claudio.





De:
[EMAIL PROTECTED]




Para:
obm-l@mat.puc-rio.br




Cópia:





Data:
Wed, 28 Sep 2005 12:47:10 -0300 (ART)




Assunto:
Re: [obm-l] ajuda em diofantina
 Sim, é x ao quadrado.Paulo Melo [EMAIL PROTECTED] escreveu:
q q significa x*2x elevado ao quadrado???--- nilton rr <[EMAIL PROTECTED]>escreveu: Companheiros, continuo aguardando ajuda, obrigado  Mostre q não tem soluções inteiras as seguintes equações:  a)x*13+12x+13y*5 = 1 b) x*2-14y*3 = 3  - Novo Yahoo! Messenger com voz: ligações, Yahoo! Avatars, novos emoticons e muito mais. Instaleagora!___ Novo Yahoo! Messenger com voz: ligações, Yahoo! Avatars, novos emoticons e muito mais. Instale agora! www.yahoo.com.br/messenger/=Instruções para entrar na lista, sair da lista e usar a lista emhttp://www.mat.puc-rio.br/~nicolau/olimp/obm-l.html=


Novo Yahoo! Messenger com voz: ligações, Yahoo! Avatars, novos emoticons e muito mais. Instale agora!


Re: [obm-l] Problemas de Teoria de Numeros

2005-09-28 Por tôpico claudio\.buffara
Eu erreiquando disse que "p divide ambos". Supondo p ímpar, teremos:
p divide p! mas não divide (p-1)! - 1, pois o teorema de Wilson diz que p divide (p-1)! + 1 e não que p divide (p-1)! - 1.
Conclusão: conforme o resto do meu argumento original, nenhum primo divide ambos, o que implica que o mdc é 1.
Se p = 2, o mdc é 2.

No mais, a divide a^fi(b) e b divide b^fi(a), pois fi(x) =1 para todo x inteiro positivo. Logo ab divide a^fi(b)*b^fi(a).

[]s e desculpe o deslize na 2a. questão.
Claudio.





De:
[EMAIL PROTECTED]




Para:
obm-l@mat.puc-rio.br




Cópia:





Data:
Wed, 28 Sep 2005 11:04:12 -0300




Assunto:
Re: [obm-l] Problemas de Teoria de Numeros
 Eu não entendi porque o Cláudio Buffara deu a solução abaixo pra questão 
 "Encontrar o máximo divisor comum de (p-1)!-1 e p!, com p primo" e a 
 resposta do Marcelo Rufino deu diferente? Tem alguma razão? Onde está o 
 erro?
 
 -- Resposta do Cláudio Buffara
 p divide ambos e, além disso, p^2 não divide p!.
 Qualquer primo maior do que p não divide p!
 Qualquer primo menor do que p não divide (p-1)! - 1.
 Logo, mdc = p.
 
 -- Resposta do Marcelo Rufino
 Seja d = mdc [(p - 1)! - 1, p!] = d | (p - 1)! - 1 e d | p! = 
 d | p[(p - 1)! - 1] - p! = d | - p = d = 1 ou d = p
 Entretanto, pelo teorema de Wilson, temos que (p - 1)! = - 1 (mpd. p), 
 ou seja, (p - 1)! - 1 = - 2 (mod. p), ou seja, p não divide (p - 1)! - 1.
 Logo, a única possibilidade é d = 1.
 
 Abraços,
 Aldo
 
 Marcelo Rufino wrote:
 
  As soluções de algumas das questões seguem abaixo.
 
  Pessoal, estou com alguns problemas de Teoria de Números que não sei 
  como resolver.
 
  1. Provar que para p primo (p-1)!=p-1 (mod 1+2+3+...+(p-1))
  2. Encontrar o máximo divisor comum de (p-1)!-1 e p!, com p primo.
 
 
  Seja d = mdc [(p - 1)! - 1, p!] = d | (p - 1)! - 1 e d | p! 
  = d | p[(p - 1)! - 1] - p! = d | - p = d = 1 ou d = p
  Entretanto, pelo teorema de Wilson, temos que (p - 1)! = - 1 (mpd. p), 
  ou seja, (p - 1)! - 1 = - 2 (mod. p), ou seja, p não divide (p - 1)! - 1.
  Logo, a única possibilidade é d = 1.
 
  3. Mostrar que para n=4 o resto da divisão por 12 de 1!+2!+3!+...+n! 
  é 9.
 
 
  Como para n = 4 temos que 12 | n!, então o resto da divisão de 
  1!+2!+3!+...+n! por 12 é igual ao resto da divisão de 1! + 2! + 3! = 9 
  por 12, que vale 9.
 
  4. Mostrar que para todo n inteiro 3n^2-1 nunca é um quadrado.
 
 
  Observe que:
  (3x)^2 = 9x^2 = 3k,
  (3x + 1)^2 = 9x^2 + 6x + 1 = 3(3x^2 + 2x) + 1 = 3k + 1,
  (3x + 2)^2 = 9x^2 + 12x + 4 = 3(3x^2 + 4x + 1) + 3 = 3k + 1
  Logo, todo quadrado perfeito deixa resto o ou 1 na divisão por 3. Como 
  3n^2 - 1 deixa resto 2 na divisão por 3 então não pode ser quadrado 
  perfeito.
 
  5. Mostrar que 5n^3+7n^5=0 (mod 12) para todo n.
  6. Seja f(x)=a_0+a_1x+...+a_nx^n um polinômio com coeficientes 
  inteiros onde a_n0 e n=1. Mostrar que f(x) é composto para 
  infinitos valores da variável x.
  7. Mostrar que para a e b inteiros, com (a, b)=1 temos 
  a^fi(b)+b^fi(a)=1 (mod ab)
 
 
  Até mais,
  Marcelo Rufino
 
 
 
 
  Será que alguém pode me ajudar a resolvê-los?
 
  Obrigado,
 
  Aldo.
 
  = 
 
  Instruções para entrar na lista, sair da lista e usar a lista em
  http://www.mat.puc-rio.br/~nicolau/olimp/obm-l.html
  = 
 
 
  =
  Instruções para entrar na lista, sair da lista e usar a lista em
  http://www.mat.puc-rio.br/~nicolau/olimp/obm-l.html
  =
 
 =
 Instruções para entrar na lista, sair da lista e usar a lista em
 http://www.mat.puc-rio.br/~nicolau/olimp/obm-l.html
 =
 


Re: [obm-l] periodo

2005-09-22 Por tôpico Claudio Buffara
Title: Re: [obm-l] periodo



Suponha que seja e que o periodo fundamental eh p.
Entao, f(p) = f(0) == 
cos(raiz(p)) = 1 e p eh o menor real positivo com esta propriedade ==
raiz(p) = 2*pi ==
p = 4*pi^2

Mas f(2p) = f(p) = f(0) = 1 ==
cos(raiz(8*pi^2)) = 1 ==
cos(2*pi*raiz(2)) = 1 ==
contradicao pois cos(x) = 1 == x eh multiplo inteiro de 2*pi.

Conclusao: f nao eh periodica.

[]s,
Claudio.

on 22.09.05 11:32, Danilo Nascimento at [EMAIL PROTECTED] wrote:

Demonstrar que a funcao f(x) = cos sqrt(x) nao é periodica.

 Novo Yahoo! Messenger com voz: ligações, Yahoo! Avatars, novos emoticons e muito mais. Instale agora! http://us.rd.yahoo.com/mail/br/taglines/*http://br.messenger.yahoo.com/ 






Re: [obm-l] Não Resisto...

2005-09-16 Por tôpico Claudio Buffara
on 16.09.05 07:54, edmilson motta at [EMAIL PROTECTED] wrote:

 A discussão entre Nicolau e Gugu sobre o problema da
 mentira me fez lembrar uma das lendas sobre a origem
 do problema do 3x+1.
 
 Ele teria sido criado pelos soviéticos para atrapalhar
 a pesquisa matemática nos Estados Unidos...
 
 Abraços, Ed.
 
 
 
Eu ateh tenho uma opinao sobre o problema mas prefiro seguir a maxima:
em briga de elefante, lugar de macaco eh na arvore...

[]s,
Claudio.


=
Instruções para entrar na lista, sair da lista e usar a lista em
http://www.mat.puc-rio.br/~nicolau/olimp/obm-l.html
=


Re: [obm-l] desigualdade

2005-09-15 Por tôpico Claudio Buffara
Title: Re: [obm-l] desigualdade



Usando a serie (1/2)*ln((1+x)/(1-x)) = SOMA(n=1) x^(2n-1)/(2n-1) com x = 1/3, obtemos:
log(2)  2*(1/3 + 1/(3*3^3) + 1/(5*3^5) + 1/(7*3^7) + 1/(9*3^9))  0,693146 ==
log(2)^5  0,16

Por outro lado, (2/5)^2 = 4/25 = 0,16.

Logo, log(2)^5  (2/5)^2 == log(2)  (2/5)^(2/5).

No entanto, a aproximacao eh muito boa.
A diferenca eh de apenas 0,0234 (aproximadamente).

Contudo, eu ainda gostaria de ver uma solucao mais engenhosa...

[]s,
Claudio.


on 09.09.05 21:19, Júnior at [EMAIL PROTECTED] wrote:

A questao diz: Mostrar que ln 2  (2/5)^(2/5).
Por extenso: Mostrar que log neperiano de 2 é maior que dois quintos elevado a dois quintos.
É isso.






Re: [obm-l] ajuda

2005-08-26 Por tôpico Claudio Buffara
Title: Re: [obm-l] ajuda



on 26.08.05 00:00, Marcus Aurélio at [EMAIL PROTECTED] wrote:

Demonstre que sendo m inteiro e positivo a parte inteira de (2+3^1/2)^m e sempre um número ímpar. 


Seja x(m) = (2+raiz(3))^m + (2-raiz(3))^m

x(m) eh solucao da recorrencia:
x(m) = 4*x(m-1) - x(m-2) com x(0) = 2 e x(1) = 4.
(use inducao, por exemplo)

Eh facil ver que x(m) eh sempre par e que 0  (2-raiz(3))^m  1.

Como (2+raiz(3))^m = x(m) - (2-raiz(3))^m = (PAR) - (INTEIRO ENTRE 0 E 1), temos que (2+raiz(3))^m = (IMPAR) + (INTEIRO ENTRE 0 E 1).

[]s,
Claudio. 







Re:[obm-l] criterio de Eisenstein

2005-08-23 Por tôpico claudio\.buffara





De:
[EMAIL PROTECTED]




Para:
obm-l@mat.puc-rio.br




Cópia:





Data:
Tue, 23 Aug 2005 02:22:07 + (GMT)




Assunto:
[obm-l] criterio de Eisenstein
 No livro do Arnaldo Garcia e Yves Lequain ( elementos de álgebra ), na página 74 , foi proposto o seguinte exercício: Sejam D um domínio fatorial e f( X ) = a(n)X^n + ... + a(0) pertencente a D[X] um polinômio de grau n( ou igual) 1. Suponha que exista um um elento primo p pertencente a D tal que p não divida a a(0) , p divida 
 a(i) para todo i ( ou igual ) 1 e p^2 não divida a(n). Mostre que f(X) não é o produto de dois fatores de grau (ou igual) 1 em D[X]. Alguem poderia provar esta proposição que o livro do Arnaldo chama de " dual " do Critério de Eisenstein ? Eu poderia usar este "dual " para mostrar que f(X) é irredutível ou não em Z[X] ?
UseEisenstein no polinômio g(X) = X^n*f(1/X).
[]s,
Claudio.


Re: RES: RES: [obm-l] Problema do casal de filhos

2005-08-23 Por tôpico claudio\.buffara





De:
[EMAIL PROTECTED]




Para:
obm-l@mat.puc-rio.br




Cópia:





Data:
Tue, 23 Aug 2005 10:05:18 -0300




Assunto:
Re: RES: RES: [obm-l] Problema do casal de filhos
 On Mon, Aug 22, 2005 at 09:31:55PM -0300, Luiz Viola wrote:
  Intuitivamente pra mim é 1/2. Acho que para a maioria das pessoas a quem
  eu propus o problema também responderam 1/2. O livro porém apresenta a
  resposta 1/3, tal como propuseram a solução aqui na lista... eu não
  consigo aceitar 1/3...nem fazendo força...
 
 O livro dá alguma explicação? Seria interessante se você pudesse transcrever
 enunciado e resolução para que pudéssemos saber exatamente de que estamos
 falando. Pelo que você escreveu até agora, não acho implausível que o livro
 esteja simplesmente errado.
 
 []s, N.
 

Isso me faz lembrar o problema das três caixas fechadas.
A primeiradelas contém duas moedas de ouro, a segunda, uma de ouro e uma de prata, e a terceira, duas de prata.
Tomamos uma caixa ao acaso e dela retiramos uma moeda.
Dado que a moeda retirada é de ouro, qual a probabilidade de que a outra moeda desta caixa também seja de ouro?

A resposta é 2/3, apesar de muita gente achar que é 1/2.

[]s,
Claudio.




Re:[obm-l] Soma de serie

2005-08-23 Por tôpico claudio\.buffara
Esse é interessante pois essa série é condicionalmente convergente.

É razoável supor que n varia de 2 a infinito.

Expandindo o somando em frações parciais, a série fica:
S = (1/3)*SOMA(n=2) (-1)^n*(5/(n-1)+ 1/(n+2))

ou, equivalentemente (trocando n por n+1):
(1/3)*SOMA(n=1) (-1)^(n+1)*(5/n+ 1/(n+3)) =
(1/3)*((5+1/4) - (5/2+1/5) + (5/3+1/6)- (5/4+1/7) + ... )

Como as séries:
S1 = SOMA(n=1) (-1)^(n+1)*5/n 
e 
S2 = SOMA(n=1) (-1)^(n+1)/(n+3)
são ambas convergentes, nós podemos somá-las separadamente, obtendo:
S1 = 5*log(2) 
e 
S2 = log(2) - 1 + 1/2 - 1/3 = log(2) - 5/18.

Assim, S = (S1 + S2)/3 = (4/3)*log(2)+ 5/18

[]s,
Claudio.





De:
[EMAIL PROTECTED]




Para:
obm-l@mat.puc-rio.br




Cópia:





Data:
Tue, 23 Aug 2005 17:20:02 -0300 (ART)




Assunto:
[obm-l] Soma de serie
 Pessoal , o exercicio abaixo eu tirei do livro do Paulo Boulos "sequencias e series de numeros e funções" . Ele pede pra calcular 
 
  Somatorio a_n onde 
 
 
  a_n =[(-1)^n (2n+3)]/[(n-1)(n+2)]
 
 Aguem consegue fazer?
 
 Obs: No exercicio ele não diz qual o indice do somatorio.
 
 
  Abs.
 


Yahoo! Acesso Grátis: Internet rápida e grátis. Instale o discador agora!


Re:[obm-l] convergencia da sequencia das derivadas

2005-08-16 Por tôpico claudio\.buffara





De:
[EMAIL PROTECTED]




Para:
"OBM-l (E-mail)" obm-l@mat.puc-rio.br




Cópia:





Data:
Tue, 16 Aug 2005 11:36:41 -0300




Assunto:
[obm-l] convergencia da sequencia das derivadas
 Bom dia a todos
 
 Seja f_n uma sequencia de funcoes definidas e diferenciaveis em um intervalo
 I de R. Suponhamos que a sequencia das derivadas f'_n convirja uniformemente
 em I para uma funcao g. Hah um teorema que diz que, se a sequencia de numero
 reais f_n(u) convergir para algum u de I, entao f_n converge uniformemente
 em I para uma funcao f tal que f' = g em I. Esta ultima condicao eh
 realmente essencial? 

Sim. Suponha que f_n:I - Ré dada por:
f_n(x) = x + (-1)^n.
Cada f_n é diferenciável e (f_n') converge uniformemente para a função constante e igual a 1. No entanto, (f_n) não converge. 
Repare que, qualquer que seja u em I, f_n(u) não converge.

Se soubermos que f'_n converge uniformemente em I, já
 podemos entao fazer alguma inferencia quanto aa convergencia das primitivas?

Não, conforme o exemplo acima.

 Se adicionarmos a hipotese de as f'_n sao continuas, temos entao alguma
 conclusao interessante, alem de que g eh continua?

Não que eu saiba. É claro que f_n' contínua == f_n' integrável. Mas continuamos a precisar da convergência de (f_n(u)) para algum u.

 Eu acho que hah um teorema que se refere ao caso em que as f'_n sao
 Lipschitz, mas nao sei qual eh.

Me parece que a condição de (f_n(u)) ser convergente para algum u permanece necessária.

[]s,
Claudio.



Re:[obm-l] limite de uma sequencia

2005-08-09 Por tôpico claudio\.buffara
O problema é achar lim x(n), onde:
x(n) = p*x(n-1) + (1-p)*x(n-2) com 0  p  1.
x(1) = a
x(2) = b

Equação característica: t^2 - p*t + p - 1 = 0 ==
t =1 ou t = p - 1

x(n) = A + B*(p - 1)^(n-1)

Como-1  p - 1 0, temos que lim x(n) = A.

x(1) = A + B = a
x(2) = A + B*(p - 1) = b ==

A = a - (a - b)/(2 - p) e B = (a - b)/(2 - p)

Ou seja, lim x(n) =a - (a - b)/(2 - p)

Fazendo p = p1/(p1 + p2), teremos:

lim x(n) = a - (a - b)*(p1 + p2)/(p1 + 2*p2).

[]s,
Claudio.





De:
[EMAIL PROTECTED]




Para:
"OBM" obm-l@mat.puc-rio.br




Cópia:





Data:
Mon, 8 Aug 2005 20:38:11 -0700 (PDT)




Assunto:
[obm-l] limite de uma sequencia
 Eu encontrei o problema de determinar o limite da
 sequencia de reais dada por:
 
 x(1) = a, x(2) = b, x(n) = (p1*x(n-2) + p2*x(n-1))/(p1
 + p2) para n=3, com p1, p2 0. Assim, a partir de n
 =3, cada termo da seq. eh a media ponderada dos 2
 termos anteriores com relacao aos pesos p1 e p2. 
 
 Eu cheguei a uma solucao, mas com um processo um tanto
 trabalhoso. Inicialmente, nao eh dificil mostrar que,
 para a=0 e b =1, x_n eh uma sequencia de Cauchy, logo
 convergente. Para isto, verificamos que, |x(n) -
 x(n-1)| = (max(p1,p2)/(p1 + p2))^(n-2) para n=2. Com
 alguma algebra, levando em conta o limite de series
 geometricas de razao 1, podemos mostrar que x(n) e'
 uma seq. de Cauchy.
 
 Depois, verificamos por inducao, num processo
 algebrico um tanto trabalhoso, que a subsequencia x(2n
 +1 ) e' uma serie geometrica de razao  1. Logo esta
 serie converge para o limite de x(n). Definindo-se r =
 p2/(p1 + p2) e s = p1/(p1 + p2), concluimos que, se
 a=0 e b=1, entao x(n) -- L(0,1) = r/(1 - s^2). E aih
 eh facil concluir que, para quaisquer a e b, x(n) --
 L(a,b) = (b-a)L(0,1) + a.
 
 Eu creio que hah um processo simples para chegarmos ao
 limite, mas nao consegui ver. Uma saida natural seria
 observando que, sendo L o limite, entao L tem que
 satisfazer a L = (p1*L + p2*L)/(p1 + p2), mas isto nos
 leva a que L = L e nao agrega qualquer informacao. 
 
 Se em vez da media ponderada dos 2 termos anteriores
 tivessemos a dos k termos anteriores, entao o processo
 que achei levaria a um trabalho algebrico realmente
 insano. 
 
 Artur
 
 
 
 
 Start your day with Yahoo! - make it your home page 
 http://www.yahoo.com/r/hs 
 
 =
 Instruções para entrar na lista, sair da lista e usar a lista em
 http://www.mat.puc-rio.br/~nicolau/olimp/obm-l.html
 =
 


Re: [obm-l] sistemas lineares

2005-07-17 Por tôpico claudio\.buffara
Oi, Michele e Wagner:

Nao resisto e vou me intrometer na discussao. Quando eu estava no 2o. grau, aprendi a regra de Cramer pra resolver sistemas lineares mas soh vim a aprender escalonamento quando cursei algebra linear na faculdade (engenharia), apesar deste segundo metodo ser muito mais natural, intuitivo, eficiente e eficaz (no sentido de sempre determinar o numero de solucoes do sistema - 0, 1 ou infinitas). 

Pra mim esta eh uma aberracao do curriculo oficial de matematica do 2o. grauno Brasil.

[]s,
Claudio.





De:
[EMAIL PROTECTED]




Para:
obm-l@mat.puc-rio.br




Cópia:





Data:
Sat, 16 Jul 2005 17:11:26 -0300




Assunto:
Re: [obm-l] sistemas lineares
 Olá, Michele!
 
 Esta é uma questão importante. O problema é que o método falha em certos 
 sistemas, sem aviso prévio.
 Veja o sistema x+y+z=1; 2x+2y+2z=2; 3x+3y+3z=4 que é obviamente 
 impossível. Discutindo com esse método, todos os determinantes são nulos 
 e o sistema deveria apresentar infinitas soluções. Desafio então, 
 alguém, a me mostrar uma só. Existem muitos sistemas menos "visuais" que 
 este no qual o método falha também. Então, melhor que arriscar, é ter um 
 método seguro que acerte em 100% dos casos, como Rouché-Capelli ou 
 escalonamento.
 
 Um abraço,
 
 Guilherme.
 
 
 Michele Calefe wrote:
 
  Eduardo, mas quando o sistema tem o número de incógnitas igual ao 
  número de equações, e, o determinante é zero, dá pra dizer que se 
  todos os Dx, Dy,...forem nulos, o sistema é SPI? Além disso, se pelo 
  menos um deles é diferente de zero o sistema é SI? Por que não faz 
  sentido discutir dessa maneira?
  michele
 
  */Eduardo Wagner <[EMAIL PROTECTED]>/* escreveu:
 
  MIchele:
 
  A regra de Cramer eh um metodo que permite
  explicitar cada incognita de um sistema linear com
  mesmo numero de equacoes e incognitas quando o
  determinante do sistema eh diferente de zero.
  Tem interesse teorico mas, na pratica eh terrivelmente
  ineficiente.
  A regra de Cramer nao serve para discutir sistemas.
  A melhor forma de discutir um sistema linear com m
  equacoes e n incognitas eh o escalonamento.
 
  Abraco.
 
  W.
 
  --
  From: Michele Calefe <[EMAIL PROTECTED]>
  To: obm-l@mat.puc-rio.br
  Subject: [obm-l] sistemas lineares
  Date: Fri, Jul 15, 2005, 3:52 PM
 
 
  Pessoal, eu gostaria de saber se é possível *discutir* um
  sistema linear utilizando a regra de Cramer. Sei que não é
  possível encontrar a solução do SPI, mas, é possível afirmar
  quando o sistema é SI ou SPI?
  
  obrigada,
  
  michele
  __
  Converse com seus amigos em tempo real com o Yahoo! Messenger
  _http://br.download.yahoo.com/messenger/_
 
  
  Yahoo! Acesso Grátis 
  


Re:[obm-l] Outra serie divergente

2005-07-01 Por tôpico claudio\.buffara
Oi, Artur:

Pondo s_n = a_1 + a_2 + ... + a_n, sabemos que se SOMA a_n diverge, entao SOMA (a_n/s_n) tambem diverge.

Como s_n - +infinito, existe n_0 tal que n  n_0 == 
a_1 + a_2 + ... + a_(n-1) = s_(n-1)  1 ==
s_n  1+ a_n.
Assim, n  n_0 == a_n/(1 + a_n)  a_n/s_n.
Como SOMA (a_n/s_n) diverge, SOMA a_n(1+a_n) tambem diverge.

[]s,
Claudio.





De:
[EMAIL PROTECTED]




Para:
obm-l@mat.puc-rio.br




Cópia:





Data:
Fri, 1 Jul 2005 19:01:00 -0700 (PDT)




Assunto:
[obm-l] Outra serie divergente
 Boa noite amigos
 
 Eu gostaria de alguma sugestao para provar o seguinte
 (talvez haja uma saida simples):
 
 Seja a_n uma sequencia de numeros positivos tal que
 Soma(n=1) a_n divirja. Entao, Soma(n=1)
 (a_n)/(1+a_n) tambem diverge. 
 
 Se lim a_n 0, entao eh facil ver que lim
 (a_n)/(1+a_n) 0, de modo que a serie diverge. Mas se
 lim a_n = 0 ou nao existir, entao a conclusao nao eh
 tao simples, acho que por ai nao eh uma boa saida.
 
 Abracos
 Artur
 
 __
 Do You Yahoo!?
 Tired of spam? Yahoo! Mail has the best spam protection around 
 http://mail.yahoo.com 
 =
 Instruções para entrar na lista, sair da lista e usar a lista em
 http://www.mat.puc-rio.br/~nicolau/olimp/obm-l.html
 =
 


Re: [obm-l] divisor

2005-06-23 Por tôpico claudio\.buffara
 Aklias, sera que da para fatorar o polinomio
 a^33-a^19-a^17-1 ?
 
Certamente. 
Isso eh igual a (a+ 1)*f(a), onde f(a) é mônico de grau 32.
Aliás, isso dá uma solução mais natural para o problema original, com a = 2, pois mostra que além de ser ímpar, a expressão é divisível por 3.

[]s,
Claudio.



Re: [obm-l] integrais - funcoes analiticas

2005-06-23 Por tôpico claudio\.buffara
Sugestão: Considere a integral do campo de vetores F: R^2 - {0} - R^2 dado por ( -y/(x^2+y^2) , x/(x^2+y^2) ) sobre o caminho g:[0,2pi] - R^2 dado por g(t) = (acos(t),bsen(t))

Como g é de classe C^infinito, a integral será igual a:
Integral(0...2pi) F(g(t)),g'(t)dt, onde  ,  é o produto interno usual.

Fazendo z = x + iy, expressando dz/z em função de x e y e olhando pra parte imaginária desta diferencial, você vai ver que o campo F não foi tirado da cartola. No mais, você está certo em afirmar que Integral(C) dz/z = i*2pi, onde C é qualquer curva fechada contendo a origem em seu interior.

[]s,
Claudio.





De:
[EMAIL PROTECTED]




Para:
obm-l@mat.puc-rio.br




Cópia:





Data:
Thu, 23 Jun 2005 15:20:46 -0300 (ART)




Assunto:
Re: [obm-l] integrais - funcoes analiticas
 
 Prezado Fábio
 
 Acredito que z = a.cost + i.b.sent ???...
 
 Parece, então, tratar-se de uma aplicação do Teorema
 de Green (já que pede para calcular a integral de
 linha de duas formas).
 
 []s
 
 Wilner
 
 --- Fabio Niski <[EMAIL PROTECTED]>escreveu:
 
  Olá gente!
  Topei com este problema
  "Sejam a,b pert R, a  0, b  0 e consideremos a
  elipse
  g : t pert [0,2pi] - acost + isent pert C. Calcular
  de duas formas 
  diferentes a integral Int_linha[sobre g]dz/z e
  deduzir que
  Int[0, 2pi] (dt/(acost)^2 + (bsent)^2) = 2pi/ab"
  
  obs: Int_linha é integral de linha se nao ficou
  claro.
  
  Bom, fique claro que no curso nao vimos
  singularidades, series de 
  Laurent e residuos. Se a unica maneira de resolver
  este problema for 
  lançando mao destas ferramentes por favor alguem me
  avise.
  
  Eu começei assim:
  Considere: g[b] a circunferencia de centro na origem
  e raio b orientada 
  no sentido antihorario, V o complementar de uma
  disco fechado centrado 
  na origem com raio estritamente menor que b e a
  funcao f, dada por f(z) 
  = 1/z.
  Temos evidentemente que V é aberto e f é holomorfa
  em V.
  Como g e g[b] são V-homologicas vale o teorema de
  Cauchy
  e portanto
  Int_linha[sobre g]dz/z = Int_linha[sobre g[b]dz/z
  =
  Int[0,2pi]((b*i*e^it)/b*e^it))dt
  = 2pi*i
  Bom, nem sei se esta resultado esta correto, mas
  apartir dai eu nao 
  tenho nenhuma ideia para continuar. Agradeco
  qualquer ajuda/sugestao.
  Obrigado
  
  Niski
  
 
 =
  Instruções para entrar na lista, sair da lista e
  usar a lista em
  http://www.mat.puc-rio.br/~nicolau/olimp/obm-l.html
 
 =
  
 
 
 
 
 
 
 ___ 
 Yahoo! Acesso Grátis - Internet rápida e grátis. 
 Instale o discador agora! http://br.acesso.yahoo.com/
 =
 Instruções para entrar na lista, sair da lista e usar a lista em
 http://www.mat.puc-rio.br/~nicolau/olimp/obm-l.html
 =
 


RE: [obm-l] Caso de divisibilidade

2005-06-22 Por tôpico claudio\.buffara
Eu supuz que p é primo. Se não for, então o teorema não vale.





De:
[EMAIL PROTECTED]




Para:
obm-l@mat.puc-rio.br




Cópia:





Data:
Tue, 21 Jun 2005 21:59:26 -0700 (PDT)




Assunto:
RE: [obm-l] Caso de divisibilidade
 
 Cláudio, Daniel, outros,
 
 Consegui encontrar vários contra-exemplos para esse
 problema. Sendo comb(a,b) o número de combinações de a
 elementos tomados b a b, ou comb(a,b)=a!/((a-b)!b!),
 pede-se mostrar que comb(a^c,b)=0(mod a), para c=2,
 a^cb. Entretanto:
 
 comb(6^2,4) = 3 (mod 6)
 comb(22^2,4) = 11 (mod 22)
 comb(6^3,8) = 3 (mod 6)
 comb(6^2,9) = 4 (mod 6)
 comb(12^2,9) = 4 (mod 12)
 comb(10^2,25) = 4 (mod 10)
 comb(6^3,27) = 2 (mod 6)
 comb(33^2,121) = 9 (mod 33)
 comb(21^3,343) = 6 (mod 21)
 
 Pode ser que eu tenha errado alguma coisa na hora de
 programar o computador para fazer as contas, mas pelo
 menos o primeiro exemplo eu conferi na mão. Eu não
 achei esses números ao acaso. Em todos eles, sendo a =
 p*q, com p e q primos, eu fiz b = p^c e escolhi um q
 que desse problema.
 
 Abraços,
 Maurício
 
 
  Interessante! Dei uma olhada no livro que estou
 estudando e ele menciona essa fórmula (...)
 
   Um jeito mais fácil é usar a velha e, espero,
 conhecida fórmula para o expoente de p em n!, igual a
   [n/p] + [n/p^2] + [n/p^3] + 
   O expoente de p no numerador de Binom(p^m,k) (1 =
 k = p^m - 1) é:
   [p^m/p] + [p^m/p^2] + ... + [p^m/p^(m-1)] +
   (...)
   A partir dessas duas desigualdades é fácil
 concluir que o expoente de p no numerador é
 estritamente maior do que o expoente de p no
 denominador, de modo que p divide Binom(p^m,k).
   
   []s,
   Claudio.
 
Oi, Maurício,
é possível resolver essa como aplicação imediata
 do teorema de lucas, que é o seguinte:
(...)
Mas eu ainda gostaria de ver uma prova mais
   elementar deste fato...
   
[]s,
Daniel
   
 Oi, pessoal,

 Estou em cima desse exercício de teoria dos
 números faz tempo e não cheguei a nada, alguém tem
 alguma dica?
 (...)
 
 
 __
 Do You Yahoo!?
 Tired of spam? Yahoo! Mail has the best spam protection around 
 http://mail.yahoo.com 
 =
 Instruções para entrar na lista, sair da lista e usar a lista em
 http://www.mat.puc-rio.br/~nicolau/olimp/obm-l.html
 =
 


RE: [obm-l] Caso de divisibilidade

2005-06-19 Por tôpico claudio\.buffara
Um jeito mais fcil  usar a velha e, espero, conhecida frmula para o expoente de p em n!, igual a [n/p] + [n/p^2] + [n/p^3] + 

O expoente de pno numerador de Binom(p^m,k) (1 = k = p^m - 1) :
[p^m/p] + [p^m/p^2] + ... + [p^m/p^(m-1)] + [p^m/p^m] = 
p^(m-1) + p^(m-2) + ... + p + 1.

O expoente de p no denominador de Binom(p^m,k)  a soma de:
[k/p] + [k/p^2] + [k/p^3] + 
e
[(p^m-k)/p] + [(p^m-k)/p^2] + [(p^m-k)/p^3] + ...

Mas: 
[k/p^j] + [(p^m - k)/p^j] = [(k + (p^m-k))/p^j] = [p^m/p^j] = p^(m-j). 

Alm disso, para k = 1, teremos:
[1/p^j] + [(p^m - 1)/p^j] = p^(m-j) - 1  p^(m-j) = [p^m/p^j]

A partir dessas duas desigualdades  fcil concluir que o expoente de p no numerador  estritamente maior do que o expoente de p no denominador, de modo que p divide Binom(p^m,k).

[]s,
Claudio.





De:
[EMAIL PROTECTED]




Para:
obm-l@mat.puc-rio.br




Cpia:





Data:
Fri, 17 Jun 2005 17:48:08 -0300




Assunto:
RE: [obm-l] Caso de divisibilidade
 Oi, Maurcio,
  possvel resolver essa como aplicao imediata do teorema de lucas, que
  o seguinte:
 
 Se m = a_k*p^k + a_(k-1)*p^(k-1) + ... + a_1*p + a_0 e n = b_k*p^k + ...
 + b_0 (representao na base p), denotando B(m,n) a combinao de m elementos
 tomados n a n, vale
 
 B(m,n) == B(a_0,b_0)*B(a_1,b_1)*...*B(a_k,b_k)(mod p),
 
 onde B(x,y) = 0 se y  x.
 
 Mas eu ainda gostaria de ver uma prova mais elementar deste fato...
 
 []s,
 Daniel
 
 '' Oi, pessoal,
 ''
 '' Estou em cima desse exerccio de teoria dos nmeros
 ''faz tempo e no cheguei a nada, algum tem alguma
 ''dica?
 ''
 '' Mostrar que o nmero de combinaes de p^a (p
 ''elevado a a) elementos tomados k a k  divisivel por
 ''p, supondo p^ak (acho que tambm  necessrio que
 ''a1). Formulei isso assim:
 ''
 '' p^a!/(k!(p^a-k)!) = 0 (mod p)
 ''
 ''
 '' Abraos,
 '' Maurcio
 '' 
 ''
 ''
 ''
 '' 
 '' 
 ''Yahoo! Sports 
 ''Rekindle the Rivalries. Sign up for Fantasy Football 
 ''http://football.fantasysports.yahoo.com
 ''=
 ''Instrues para entrar na lista, sair da lista e usar a lista em
 ''http://www.mat.puc-rio.br/~nicolau/olimp/obm-l.html
 ''=
 
 
 
 =
 Instrues para entrar na lista, sair da lista e usar a lista em
 http://www.mat.puc-rio.br/~nicolau/olimp/obm-l.html
 =
 


Re: [obm-l] Olimpiadas Universitarias (ainda com duvidas) - parte 2

2005-06-10 Por tôpico Claudio Buffara
Title: Re: [obm-l] Olimpiadas Universitarias (ainda com duvidas) - parte 2



O site tem um arquivo de provas de olimpiadas passadas. As questoes das olimpiadas universitarias sao a melhor dica do que voce deve estudar, mas eh razoavel que voce se concentre em analise real, algebra linear (os livros do Elon sao a referencia padrao), algebra abstrata (Herstein ou, se voce for arrojado, Dummit  Foote) e teoria dos grafos (tem um livro da colecao Schaum que eh introdutorio mas bem completo).
Conhecimentos basicos de probabilidades, topologia, variaveis complexas, curvas algebricas planas e teoria de Galois tambem ajudam, pois alguns dos problemas que aparecem nas olimpiadas sao versoes simplificadas e/ou casos particulares de teoremas destas areas.

O quanto estudar por dia depende da sua disponibilidade de tempo e capacidade de manter a concentracao. 

Espero que isso ajude.


[]s,
Claudio.

on 10.06.05 00:31, Daniel Madeira Araujo at [EMAIL PROTECTED] wrote:

Wagner,
 
 Obrigado pela dica. Já tinha antes olhado esse site. O que preciso saber a mais são orientações de que assuntos estudar e quanto tempo devo estudar por dia, além de livros para estudar. O site que você forneceu não dá essas orientações. Obrigado pela atenção dispensada.
 
 Daniel


Quebra-cuca
__
Converse com seus amigos em tempo real com o Yahoo! Messenger 
http://br.download.yahoo.com/messenger/ 






Matematica Elementar (era: [obm-l] Os Teoremas de Sylow)

2005-06-03 Por tôpico Claudio Buffara
Oi, Paulo:

Muito obrigado pelas explicacoes. Vao ser muito uteis pra mim, especialmente
agora que estou justamente estudando os teoremas de Sylow.

Se, por resultado elementar, voce quer dizer um resultado que depende apenas
de conceitos combinatorios e aritmeticos simples, eu concordo com voce: os
teoremas de Sylow sao, de fato, elementares.

No entanto, pra mim, um resultado verdadeiramente elementar eh aquele que
jah estah na massa do sangue, como se diz por ai. Por exemplo, no meu caso
seria algo como o teorema de Pitagoras ou as propriedades do trinomio de 2o.
grau. Por que sao elementares? Por causa da enorme familiaridade que eu
tenho com eles, jah tendo visto, desde ha muito tempo, inumeras
demonstracoes e aplicacoes. E mesmo assim, tenho certeza de que ha problemas
envolvendo estes dois topicos que eu nao conseguiria resolver.

Alias, com essa definicao, tudo em matematica eh potencialmente elementar.
Eh soh uma questao de costume, de muita reflexao sobre o significado de cada
definicao e teorema, e de uma boa dose de pratica na aplicacao destes (veja
bem, nao me refiro a centenas de exercicios mecanicos e repetitivos, mas a
um punhado de exemplos e problemas especialmente escolhidos que ilustrem as
aplicacoes fundamentais, as extensoes e as limitacoes de cada resultado e
que forcem o estudante a pensar).

Tenho certeza de que, se conseguir continuar estudando matematica como
quero, daqui a um ou dois anos os teoremas de Sylow vao ser tao elementares
pra mim quanto (-b +/- raiz(b^2 - 4ac))/(2a).

[]s,
Claudio.
 
on 03.06.05 08:53, Paulo Santa Rita at [EMAIL PROTECTED] wrote:

 Ola Pessoal,
 
 As demonstracoes dos teoremas abaixo podem ser encontradas em muitos livros
 de algebra. Os pre-requisitos para entender os teoremas sao os conceitos
 inicias de qualquer curso sobre grupos, tais como grupo normal, isomorfismos
 etc.
 
 Os Teoremas de Sylow
...

=
Instruções para entrar na lista, sair da lista e usar a lista em
http://www.mat.puc-rio.br/~nicolau/olimp/obm-l.html
=


Re:[obm-l] Aplicacao do Teorema Chines

2005-06-01 Por tôpico claudio\.buffara
Bem, os teoremas de Sylow e a teoria de Galois fazem parte da ementa dasdisciplinas de álgebra do programa de mestrado do IMPA. Obviamente, nós dois temos idéias um pouco distintas sobre o que é elementar, o que, sem dúvidas, deve-se a minha falta de conhecimentos matemáticos.

[]s,
Claudio.





De:
[EMAIL PROTECTED]




Para:
obm-l@mat.puc-rio.br




Cópia:





Data:
Wed, 01 Jun 2005 11:38:49 +




Assunto:
Re:[obm-l] Aplicacao do Teorema Chines
 Oi Claudio e demais colegas
 desta lista ... OBM-L,
 
 Quanto falei "metodos elementares" estava pensando "ate Teoria de Galois", 
 pois é para esta teoria que convergem quase todos os conceitos que estudamos 
 em Algebra e que a precedem a apresentacao da Teoria de Galois. Ela é como 
 um ponto de aglutinacao, a partir do qual o que nao e elementar nasce.
 
 O problema e verdadeiramente simples, senao eu nao teria colocado nos termos 
 que coloquei. Vou dar uma ideia e voce, preenchendo os detalhes ( por favor 
 ), vai ver isso :
 
 Seja G de ordem 255. Dado que 255=3*5*17, e facil mostrar - usando o 3 
 teorema de Sylow - que ha apenas um 17subgrupo de Sylow. Logo ele e normal. 
 Chame ele de H. Tome um 3subgrupo de Sylow E e um 5subgrupo de Sylow F.
 
 Claramente que HE e subgrupo. Aplique Sylow aqui e mostre que E e unico em 
 HE. Logo E e normal em HE. Usando o fato que o normalizador de E e o maior 
 subgrupo no qual E e normal e que o indice do normalizador e o total de 
 3subgrupos, conclua que E e unico em G. Mesmo raciocinio para F. Segue que 
 existem pois apenas 1 3subgrupo de Sylow, 1 5subrupo de Sylow e 1 17subgrupo 
 de Sylow. Dado que 3,5 e 17 sao primos, todos eles sao ciclicos e isomorfos 
 respectivamente a (Z/3Z), (Z/5Z) e (Z/17Z). Assim, G e o produto direto 
 (Z/3Z)*(Z/5Z)*(Z/17Z), pois o grupo e o produto direto dos seus p-subrupos 
 de Sylow.
 
 Agora, pelo Teorema Chines dos Restos ( pois 3, 5 e 17 sao dois a dois 
 primos entre si ) :
 (Z/3Z)*(Z/5Z)*(Z/17Z) isomorfo a (Z/3*5*17Z). Assim, G e isomorfo a 
 (Z/3*5*17Z)=(Z/255Z). E acabou.
 
 Um Abraco a todos
 Paulo Santa Rita
 4,0832,010605
 
 
 From: "claudio.buffara" <[EMAIL PROTECTED]>
 Reply-To: obm-l@mat.puc-rio.br
 To: "obm-l" 
 Subject: Re:[obm-l] Aplicacao do Teorema Chines
 Date: Wed, 1 Jun 2005 01:47:26 -0300
 Data:Tue, 31 May 2005 11:07:40 +
 
 Oi, Paulo:
 
 Como é que se prova isso usando apenas resultados elementares (ou seja, sem 
 usar os teoremas de Sylow ou algo equivalente)?
 
 Mesmo a demonstração de que existe um único grupo de ordem 15 fica muito 
 complicada se formos usar apenas métodos elementares.
 
 []s,
 Claudio.
 
 _
 MSN Messenger: converse online com seus amigos . 
 http://messenger.msn.com.br
 
 =
 Instruções para entrar na lista, sair da lista e usar a lista em
 http://www.mat.puc-rio.br/~nicolau/olimp/obm-l.html
 =
 


Re: [obm-l] complexos : problema do Rudin

2005-05-25 Por tôpico claudio\.buffara





De:
[EMAIL PROTECTED]




Para:
obm-l@mat.puc-rio.br




Cópia:





Data:
Mon, 23 May 2005 16:10:27 -0300




Assunto:
Re: [obm-l] complexos : problema do Rudin
 Fabio Niski wrote:
 
  Fabio Niski wrote:
  
  Pessoal, este é o exercicio 5 do Capitulo 10 do Real and Complex 
  Analysis :
 
  Suponha que b é um numero complexo, |b| != 1. Calcule
  Integral[0 até 2pi](dt/(1-2b*cos(t) + b^2))
  integrando [(z - b)^-1]*{[z-(1/b)]^-1} sobre o circulo unitario.
 
  Alguem saberia como resolver? Poderia postar aqui?
  Obrigado.
  
  
  
  Ignorem! Eu acabei de conseguir.
 
 Alias, agora estou na duvida.
 Pela minha resolucao se o valor absoluto de b for menor do que 1,
 eu cheguei em:
 
 Integral[0 até 2pi](dt/(1-2b*cos(t) + b^2)) = -2*pi*(b^2 - 1)
 
 Testando no Mathematica, eu vi que para valores de b com modulo muito 
 proximo a zero, o meu resultado parece estar correto, mas quando eu tomo 
 b = 0,9 + 0i por exemplo, o Mathematica me diz que a integral vale
 aprox 33.0694 , enquanto pela minha formula chego em aprox 1.19381.
 
 E agora? Quem é que esta certo?
 

Pra b = 0,9, o integrando fica 1/(1,81 - 1,80*cos(t))
Fazendo uma soma de Riemann com subintervalosmedindo 2pi/1000 numa planilha Excel, eu achei que a integral vale aproximadamente 33,0694.
Ou seja, o Mathematica está certo.

Além disso, repare que se b = 0,9, então 1 - b^2 = 0,19.
Repare também que33,0694*0,19 = 6,2832 = 2*3,1416.

Ou seja, há uma alta probabilidade de que a integral para b qualquer (com módulo  1) valha 2*pi/(1 - b^2). Pelo menos pra b = 0, o resultado bate exatamente.

Quem disse que matemática não é uma ciência experimental?

[]s,
Claudio.



Re: [obm-l] a(n+1) = x^a(n)

2005-05-22 Por tôpico Claudio Buffara
on 19.05.05 22:53, Claudio Buffara at [EMAIL PROTECTED] wrote:

 OI, Demetrio:
 
 Segue abaixo uma solucao detalhada para o problema de se determinar os
 valores de x tais que a sequencia (a(n)) dada por a(1) = x e a(n+1) = x^a(n)
 converge. O caso 0  x  1 foi feito pelo Marcio Cohen.
 
Apenas uma retificacao: checando com mais cuidado os meus arquivos,
verifiquei que a solucao enviada pelo Marcio Cohen eh, de fato, de autoria
do Gugu. Desculpem o deslize. Nao foi intencional.

[]s,
Claudio.

=
Instruções para entrar na lista, sair da lista e usar a lista em
http://www.mat.puc-rio.br/~nicolau/olimp/obm-l.html
=


Re: [obm-l] grupo abeliano

2005-05-22 Por tôpico Claudio Buffara
on 22.05.05 15:20, Chicao Valadares at [EMAIL PROTECTED] wrote:

 
 Minha ferrugem em relaçao ao assunto nao esta deixando
 fazer esse aqui: como provo se no grupo temos (xy)^3 =
 x^3y^3, tal grupo é abeliano??
 
 
Acho que isso soh eh verdade em geral se a ordem de G nao for um multiplo de
3.

Nesse caso, teremos:
(xy)^3 = x^3y^3  e  (yx)^2 = x^2y^2 == (xy)^2 = y^2x^2.

Assim: 
xy = (xy)^3*(xy)^(-2) = x^3y^3x^(-2)y^(-2) ==
e = x^2y^3x^(-2)y^(-3) ==
y^3x^2 = x^2y^3

Se |G| = 3m+1, entao x^(3m) = x^(-1) para todo x em G. Logo:
y^(3m)x^2 = x^2y^(3m) ==
y^(-1)x^2 = x^2y^(-1) ==
yx^2 = y^2x^2y^(-1) ==
yx^2 = (xy)^2y^(-1) ==
yx^2 = xyxyy^(-1) ==
yx^2 = xyx ==
yx = xy.

Se |G| = 3m+2, entao x^(3m) = x^(-2) para todo x em G. Logo:
y^(3m)x^2 = x^2y^(3m) ==
y^(-2)x^2 = x^2y^(-2) ==
x^2 = y^2x^2y^(-2) ==
x^2 = (xy)^2y^(-2) ==
x^2 = xyxyy^(-2) ==
x = yxy^(-1) ==
xy = yx.

***

Infelizmente, vou ficar devendo o exemplo de um grupo nao abeliano cuja
ordem eh divisivel por 3. Mas com certeza o Nicolau ou o Gugu vao arranjar
algum.


[]s,
Claudio.


=
Instruções para entrar na lista, sair da lista e usar a lista em
http://www.mat.puc-rio.br/~nicolau/olimp/obm-l.html
=


[obm-l] a(n+1) = x^a(n)

2005-05-19 Por tôpico Claudio Buffara
OI, Demetrio:

Segue abaixo uma solucao detalhada para o problema de se determinar os
valores de x tais que a sequencia (a(n)) dada por a(1) = x e a(n+1) = x^a(n)
converge. O caso 0  x  1 foi feito pelo Marcio Cohen.

Repare que o meu enunciado na minha mensagem anterior estava errado: de
fato, a sequencia converge apenas com x em [e^(-e),e^(1/e)].

Talvez seja interessante ver o que acontece quando 0  x  e^(-e). Nesse
caso, a sequencia eh limitada (0  a_n  1, para todo n) e, portanto, possui
alguma subsequencia convergente. Assim, dado x, quais os valores de
aderencia de a_n?

[]s,
Claudio.

***

Se x = 1 entao a_n eh obviamente constante e igual a 1.

Assim, suponhamos que x  1:
1) (a_n) é crescente.
Dem:
a_1 = x  1
a_2 = x^x  x^1 = x = a_1
Suponhamos que a_n = x^a_(n-1)  a_(n-1).
Teremos, então que a_(n+1) = x^a_n = x^(x^a_(n-1))  x^a_(n-1) = a_n.
Logo, por indução, o resultado está provado.

2) Se 1  x = e^(1/e), então, para todo n,  a_n = e.
Dem:
a_1 = x = e^(1/e)  e
a_2 = x^x = (e^(1/e))^e = e^(e/e) = e^1 = e
Suponhamos que a_n = e.
Então, a_(n+1) = x^a_n = (e^(1/e))^a_n = e^(e/e) = e^1 = e.
Logo, por indução, o resultado está provado.

Assim, se 1  x = e^(1/e), então a_n é crescente e limitada. Logo,
converge.

Em particular, se x = e^(1/e), teremos que lim a_n = e.

3) Se x  e^(1/e), então (a_n) é crescente e ilimitada.
Dem:
Já provamos acima que (a_n) é crescente.
Suponhamos que exista K  0 tal que, para todo n,  a_n  K.
Nesse caso, deverá existir L = lim a_n.
Assim, L = x^L  (e^(1/e))^L = e^(L/e) ==
ln(L)  L/e ==
contradição, pois para todo x  0, ln(x) = x/e ==
não existe lim a_n ==
como (a_n) é crescente, não pode ser limitada.

***

Considere o caso 0x1. Entao, x^x  x (pois a funcao h(y)=a^y eh
decrescente para a em (0,1)), i.e a2  a1.
  Do mesmo modo, x^(x^x)  x^x, i.e, a_3  a_2  (agora usando a = x^x).
  Com a = x^(x^x), vc conclui que a_4  a_3.
  Sendo composta de duas funcoes decrescentes, a funcao h(y) = x^(x^y) eh
crescente, e portanto a_1  a_3 (y=x e y=0) e em geral:
  a_1  a_3  a_5  .  a_6  a_4  a_2
A sequencia (a_n) tem portanto dois valores de aderencia, que
vamos chamar de A1 e A2. Ela converge sse esses valores forem iguais.
Observe q a subsequencia impar e par que satisfazem: a_n+2 = x^(x^a_n). Os
valores de aderencia satisfazem portanto Ai = x^(x^Ai), A1 = x^(A2) e A2
= x^(A1).
Considere entao a funcao h(y) = x^(x^y) - y.
Essa equacao sempre tem ao menos uma  raiz (que vou chamar de L'), que eh a
raiz de  g(y)=x^y-y (fazendo o grafico, como x  1, x^y eh decrescente e y
eh crescente, logo ha no maximo uma raiz. Analisando os valores em y=0 e y=1
garantimos que essa raiz de fato existe). Note que a_n, caso seja
convergente, vai para L'.
h'(y) = x^(x^y) * (x^y) * (lnx)^2 - 1
h''(y) = x^(x^y) * (x^y) * (lnx)^3 * ( 1 + (x^y)*lnx )
Como h'' tem no maximo uma raiz em (0,1) (soh o ultimo parentesis acima pode
se anular), vemos que h tem no maximo 3 raizes em (0,1) (pelo tvi, 4 raizes
de h implicam ao menos 3 de h', ..)
   1o caso: h'(L')  0. Desenhando o grafico, vc ve que h tem tres raizes
L1L'L2 nesse caso (mais formalmente, h(0)=x  0, h(L'-eps)0, h(L'+eps)0
e h(1)=x^(x) - x  0).
   Alem disso, nesse caso a seq. diverge, pois ou ela comeca de a_1 = x =
L1 e nesse caso a subsequencia impar fica limitada por L1 (inducao) nao
podendo nunca chegar em L', ou entao, se L1  x, como h(0)0 e
h(x)=x^(x^x)-x  0, temos ao menos duas raizes em (0,x), e portanto L' x,
de modo que (a_n) nao pode convergir para L' (pois a subseq. impar eh
crescente).
   2o caso: h'(L') = 0. Nesse caso, é impossível haver 3 raízes L1  L' 
L2 (pois isso traria ao menos 2 raízes em (0,L') (ja que h(0)*h(L'- eps)0)
e mais duas em (L', 1), totalizando mais de 3 raizes.  Portanto, nesse caso
a serie converge (pois se ela divergisse, a equacao teria como solucao pelo
menos os dois valores de aderencia distintos A1  A2, alem do ponto fixo L',
sendo que L' != A1,A2 (se L'=A1, entao A2 = x^(A1)=x^(L')=L'=A1).
   Logo, a_n converge sse h'(L') = 0 sse (L')^2 * (lnx)^2 = 1 sse (ln
L')^2 = 1 sse e^-1 = L' = e sse L' = e^-1 (pois claramente L'1e).
Agora, x = L' ^(1/L'), e a funcao f(y) = y^(1/y) eh crescente para ye
(f'(y) = f(y)*(1-lny)/y^2), logo L'= 1/e sse x = (1/e)^e = e^(-e).



=
Instruções para entrar na lista, sair da lista e usar a lista em
http://www.mat.puc-rio.br/~nicolau/olimp/obm-l.html
=


Re: [obm-l] Transcendentes - forma definitiva.

2005-05-19 Por tôpico Claudio Buffara
Eu sei. Acabei de mandar uma msg com o enunciado correto e a solucao.

[]s,
Claudio.

on 19.05.05 20:01, Carlos Gustavo Tamm de Araujo Moreira at [EMAIL PROTECTED]
wrote:

 Oi Cl?udio,
 Isso n?o ? exatamente verdade n?o. A seq??ncia a(n) converge se e somente
 se e^(-e) = x = e^(1/e). Se 0xe^(-e), a seq??ncia a(n) tem dois valores
 de ader?ncia em (0,1). O caso 0x1 da' um pouco mais de trabalho que o aso
 x = 1, mas tamb?m ? legal.
 Abra?os,
 Gugu
 
 
 Esse tamb=E9m =E9 um belo problema:
 
 Prove que se a(1) =3D x  0 e a(n+1) =3D x^a(n), para n =3D 1, ent=E3o a=
 sequ=EAncia ((a(n)) converge se e somente se x =3D e^(1/e).
 
 []s,
 Claudio.
 
 De:[EMAIL PROTECTED]
 
 Para:obm-l@mat.puc-rio.br
 
 C=F3pia:
 
 Data:Wed, 18 May 2005 00:53:25 +
 
 Assunto:Re:[obm-l] Transcendentes - forma definitiva.
 
 Oi Claudio e demais colegas
 desta lista ... OBM-L,
 
 Resposta correta.
 
 Em linhas gerais, a historia do problema e a seguinte : alguem resolveu=
 um 
 problema mostrando que haviam duas respostas possiveis, uma das quais
 deveria ser falsa. Uma estudante reclamou querendo saber a opcao corret=
 a. Eu 
 invoquei o teorema do Gelfond e identifiquei a resposta correta :
 
 Gelfond =3D raiz(2)^raiz(2) e transcendente =3D e irracional
 
 E entao resolvi construir explicitamente uma sequencia de numeros
 transcendentes que tinha como limite um numero natural. Aqui entrou o G=
 UGU, 
 reclamando que mesmo nao sabendo provar a transcendencia, nao haviam
 hipoteses suficientes para postular tal transcendencia. A reclamacao de=
 le, 
 correta e justificavel, era implicitamente a proposicao de um problema =
 : 
 este problema abaixo, onde voce ensaia uma solucao ...
 
 Voce faz a observacao basica e fundamental : fixando a base, o restan=
 te ( 
 o expoente ) tende para o mesmo limite. Dai, na sua linguagem, r=3Dt^r.=
 Daqui 
 sai tranquilo o resto. Note que se voce faz uma tal passagem perante al=
 gumas 
 assembleias que amam o detalhe, muito provavelmente voce sera linchado =
 e 
 execrado... Eu sempre achei notavel a capacidade de algumas pessoas de =
 
 essencializar
 o trivial e trivializar o essencial. Mas elas existem. E sao muitas !
 
 Note tambem que o pulo logico final precisa ser conectado com o e as =
 
 demais hipoteses.
 
 Quer descobrir algo que vai lhe surpreender ? Mantenha do lado esquerdo=
 do 
 cerebro o numero e=3D2,71... Com o lado direito estude as sequencias da=
 forma 
 X^X^X^... QUE CONVERGEM. Procure descobrir algo equivalente a um raio =
 de 
 convergencia.
 
 E com os melhores votos
 de paz profunda, sou
 
 Paulo Santa Rita
 3,2154,170505
 
 From: claudio.buffara
 Reply-To: obm-l@mat.puc-rio.br
 To: obm-l 
 Subject: Re:[obm-l] Transcendentes - forma definitiva.
 Date: Tue, 17 May 2005 13:47:18 -0300
 
 Ola Pessoal desta
 lista ... OBM-L,
 
 Esse problema e antigo, bonito e foi proposto aqui nesta lista - se=
 nao 
 me
 falha a memoria - pelo Prof Carlos Gustavo (GUGU), em uma forma men=
 os 
 geral.
 Peco desculpas a todos por tantas correcoes.
 
 Seja T um transcendente da forma i^i, onde i e um irracional algebr=
 ico.
 Definimos a sequencia :
 
 A(1) =3D T
 A(N+1) =3D T^A(N)
 
 Se LIM A(N)=3Dr, r racional, Considere a afirmacao : Existe N, N
 suficientemente grande, tal que A(N) e algebrico. Voce consegue pr=
 ovar 
 ou
 refutar esta afirmacao ? Note que nao e possivel aplicar, DIRETAMEN=
 TE, o
 teorema de Gelfond.
 
 Um Abraco a Todos
 Paulo Santa Rita
 3,1242,170505
 
 
 Oi, Paulo:
 
 Se lim A(n) existe e =E9 igual ao racional r, ent=E3o lim A(n+1) =3D r=
 .
 Portanto, teremos: r =3D t^r =3D=3D
 t =3D r^(1/r) =3D alg=E9brico =3D=3D
 contradi=E7=E3o, pois estamos supondo que t =E9 transcendente.
 
 Logo, ou lim A(n) n=E3o existe ou existe mas =E9 irracional.
 
 Assim, a senten=E7a:
 lim A(n) =E9 racional =3D=3D A(N) =E9 alg=E9brico para algum N sufici=
 entemente 
 grande
 =E9 verdadeira, j=E1 que a sua premissa =E9 falsa.
 
 Era isso o que voc=EA tinha em mente?
 
 []s,
 Claudio.
 
 
 --_=__=_XaM3_.1116385718.2A.615916.42.31897.52.42.007.1250663696
 Content-Type: text/html; charset=iso-8859-1
 Content-Transfer-Encoding: quoted-printable
 
 DIVEsse tamb=E9m =E9 um belo problema:/DIV
 DIVnbsp;/DIV
 DIVProve que se a(1) =3D x gt; 0 e a(n+1) =3D x^a(n), para n gt;=3D 1=
 , ent=E3o a sequ=EAncia ((a(n)) converge se e somente se x lt;=3D e^(1/e=
 )./DIV
 DIVnbsp;/DIV
 DIV[]s,/DIV
 DIVClaudio./DIV
 DIVnbsp;/DIV
 DIV
 TABLE cellSpacing=3D0 cellPadding=3D4 width=3D100% bgColor=3D#f0f0f0 b=
 order=3D0
 TBODY
 TR
 TD width=3D70 bgColor=3D#bde9fdFONT face=3DVerdana,Arial,'Trebuchet M=
 S' size=3D2BDe:/B/FONT/TD
 TDFONT face=3DVerdana,Arial,'Trebuchet MS' size=3D2[EMAIL PROTECTED]
 puc-rio.br/FONT/TD/TR/TBODY/TABLE/DIV
 DIV
 TABLE cellSpacing=3D0 cellPadding=3D4 width=3D100% bgColor=3D#f0f0f0 b=
 order=3D0
 TBODY
 TR
 TD width=3D70 bgColor=3D#bde9fdFONT face=3DVerdana,Arial,'Trebuchet M=
 S' size=3D2BPara:/B/FONT/TD
 TDFONT face=3DVerdana,Arial,'Trebuchet MS' size=3D2[EMAIL PROTECTED]
 

Re: [obm-l] Construcao Geometrica

2005-05-19 Por tôpico Claudio Buffara
Oi, Wilner:

Qual a justificativa pra essa construcao?
Ela nao me parece obvia.

[]s,
Claudio.

on 19.05.05 20:08, Eduardo Wilner at [EMAIL PROTECTED] wrote:

 
 Olá Claudio
 
 Espero que seja permitido usar lápis para traçar as
 retas,pois seria incomodo e, mesmo, antiprofissional
 fazê-lo com a grafite do compasso...
 Desculpe a brincadeira e vamos ao triângulo. Um
 ponto P', arbitrário na reta que contém BA (pode ser
 no lado ou no prolongamento de B para A).
 Denominemos r = |BP'| e transportemos esse
 comprimento para uma paralela ao lado AC passando por
 P', a partir de B', interseção desta paralela com BC,
 até E no prolongamento de B' para P'. Construamos uma
 parlela, p, ao lado BC passando por E.
 Compasso com ponta seca em P' traçamos um arco de
 raio r interceptando (ou seria intersectando?) p em
 E', no lado voltado para o triângulo. A reta BE'
 inerceptaAC no ponto Q e o ponto P sai fàcil (arco
 com centro em  Q e raio |QC| inercepta AB em P).
 
 []'s
 Wilner
 
 
 
 --- Claudio Buffara [EMAIL PROTECTED]
 escreveu:
 Dado um triangulo ABC, construa, com regua e
 compasso, um ponto P no lado AB
 e um ponto Q no lado AC tais que |BP| = |PQ| = |QC|.
 
 []s,
 Claudio.
 
 


=
Instruções para entrar na lista, sair da lista e usar a lista em
http://www.mat.puc-rio.br/~nicolau/olimp/obm-l.html
=


[obm-l] Construcao Geometrica

2005-05-15 Por tôpico Claudio Buffara
Dado um triangulo ABC, construa, com regua e compasso, um ponto P no lado AB
e um ponto Q no lado AC tais que |BP| = |PQ| = |QC|.

[]s,
Claudio.

=
Instruções para entrar na lista, sair da lista e usar a lista em
http://www.mat.puc-rio.br/~nicolau/olimp/obm-l.html
=


Re: [obm-l] Probleminha dos trancedentes.

2005-05-14 Por tôpico Claudio Buffara
on 14.05.05 08:43, Ronaldo Luiz Alonso at [EMAIL PROTECTED]
wrote:

 Olá... como vão..
 
 Primeiramente obrigado pelas respostas relacionadas as questões anterires
 sobre cosseno.
 
 Estou com duas curiosidades. ( por gentileza, se puderem me respondam!)
 1)Se um número não é raiz de nenhum polinomio esse número é chamado de
 transcedente.( está correto?) Então como eu provo que um número é ou não é
 
 Até onde eu sei está.
 
Na verdade o polinomio tem que ter coeficientes inteiros, senao dado
qualquer numero complexo qualquer a, ele eh raiz de p(x) = x - a.
Um numero complexo que eh raiz de um polinomio com coeficientes inteiros eh
chamado de numero algebrico. Todos os demais sao transcendentes.

 trancedente a partir deste raciocinio, ou de outro? ( não consegui
 enguergar
 nenhuma saida!)
 
 Lioville provou que pi era um número transcendente mostrando que *se*
 ele era raiz
 de um polinômio, esse polinômio tinha que ter grau *infinito*.  Para ver a
 prova de Lioville
 consulte um bom livro de álgebra.

Em geral essas provas de transcendencia sao dificeis e usam bastante
analise, uma vez que os numeros transcendentes sao definidos, em geral, por
meio de limites. No caso do pi, em algum ponto de demonstracao entram senos,
cossenos, derivadas e integrais.
 
 
 2)Gauss afirmou que um polinomio de grau n possue n raízes!De onde esse
 cara
 tirou isso???
 
 Dah uma olhada no livo de Alcides Lins Neto do IMPA - Análise
 Complexa.
 Nas primeiras 10 páginas você irá entender o que são as n raízes de um
 número complexo:
 São vértices de um polígono regular com n lados inscrito na
 circunferência cujo raio
 é a raiz do módulo do número complexo no plano complexo.
 Simplificadamente isso ocorre porque todo número complexo z pode
 ser escrito
 como z = |z| e^{i t) = |z|(cos p + i sen p) ==
 z^{1/n} =  |z|^{1/n} (cos p + i sen p)^{1/n} ==
 z^{1/n} =  |z|^{1/n} (cos ( p+2k*pi)/n + i sen ( p+k*pi)/n )
 
 veja que cos (p + 2*k*pi) = cos p, para todo k \in Z
 e por isso ao dividir por n temo cos(p) por  n devemos na realidade
 dividir
 cos (p+2*k*pi)  por n.
 Para ver isso note que cos p + i sen p = e^{ip} tem módulo 1
 e que (simplificadamente)
 e^{ip/n} = cos (p/n) + i sen (p/n).   Agora voltando à linha
 de cima...
 
 []s  Ronaldo L. Alonso
 

O que Gauss provou foi o que chamamos hoje de teorema fundamental da
algebra, que diz que todo polinomio com coeficientes complexos possui pelo
menos uma raiz complexa. Usano isso e inducao, voce prova que um polinomio
complexo de grau n tem exatamente n raizes (algums das quais podem ser
repetidas).

[]s,
Claudio.

 Perguntei aos meus professores e eles disseram que éssa foi a tese de
 doutorado dele, logo é muito complexa para entender. Mas mesmo assim
 queria
 pelo menos uma luz de onde ele comessou essa tese.!!!
 
 Mútissimo obrigado.
 
 Filipe Louly Quinan Junqueira
 


=
Instruções para entrar na lista, sair da lista e usar a lista em
http://www.mat.puc-rio.br/~nicolau/olimp/obm-l.html
=


Re: [obm-l] Problemas de Algebra

2005-05-13 Por tôpico Claudio Buffara
O das matrizes tudo bem, mas esse exemplo com dois elementos foi chato!

Muito obrigado.

[]s,
Claudio.

on 13.05.05 00:19, Carlos Gustavo Tamm de Araujo Moreira at [EMAIL PROTECTED]
wrote:

 
 Oi, pessoal:
 
 Preciso de ajuda nos seguintes problemas sobre grupos do Herstein - Topics
 in Algebra:
 
 Secao 2.4:
 
 13) De um exemplo de um conjunto S, fechado em relacao a uma operacao
 associativa * e tal que:
 i) Existe e em S, tal que a*e = a, para todo a em S;
 ii) Para todo a em S, existe y(a) em S tal que y(a)*a = e;
 iii) S nao eh um grupo.
 
 Um exemplo: S={e,a}, e.e=e, a.e=a, e.a=e, a.a=a. Mais geralmente,
 S={matrizes 2x2 com segunda coluna nula e a(1,1) nao nulo; e=(1,0; 0,0)}.
 
 
 
 Secao 2.6:
 
 8) De um exemplo de um grupo G, um subgrupo H, e um elemento a de G tais que
 aHa^(-1) estah propriamente contido em H.
 
 Um tal H, se existir, tem que ser necessariamente infinito, alem de
 nao-abeliano. Eu imagino que deva haver algum grupo de matrizes com esta
 propriedade, mas nao consegui pensar em nenhum.
 
 Esse eu achei mais dif?cil: acho que podemos tomar um grupo gerado por
 elementos a e x(n), com n inteiro, que s? satisfazem as rela??es
 a.x(n).a^(-1)=x(n+1), e H gerado pelos x(n) com n natural (aHa^(-1) vai ser
 gerado pelos x(n+1) com n natural). Talvez haja exemplos mais simples e
 naturais...  
 
 
 []s,
 Claudio.
 


=
Instruções para entrar na lista, sair da lista e usar a lista em
http://www.mat.puc-rio.br/~nicolau/olimp/obm-l.html
=


[obm-l] Dominos

2005-05-13 Por tôpico Claudio Buffara
Title: Dominos



Aqui vai uma versao mais simples do problema, onde ha apenas 10 dominos com numeros de 0 a 4:

Usando a ideia abaixo, temos que calcular o numero de sequencias de 10 numeros dentre {0,1,2,3,4} tais que dois numeros quaisquer sao vizinhos exatamente uma vez (o 10o. termo da sequencia eh considerado vizinho do 1o.)
Essa condicao garante que cada numero vai aparecer exatamente duas vezes. Aquela primeira condicao do primeiro termo ser igual ao ultima nao eh valida - foi mancada minha.

Fixe um numero qualquer, digamos o 0.

Entao, teremos dois tipos de sequencia:
a 0 b c 0 d _ _ _ _
ou
a 0 b _ _ c 0 d _ _
onde {a,b,c,d} = {1,2,3,4}.
Ou seja, os dois zeros distam 3 ou 5 um do outro.

Consideremos o primeiro tipo: a 0 b c 0 d x y z w

* a, b, c, d podem ser escolhidos de 4! = 24 maneiras.
Pra facilitar a analise, fixemos uma dessas escolhas. Digamos:
1 0 2 3 0 4 x y z w.

* Temos 3 possibilidades para x: 1, 2 ou 3.
Caso 1: x = 1 == 1 0 2 3 0 4 1 y z w
y soh pode ser 2 ou 3 e, uma vez escolhido y, z e w ficam unicamente determinados.
Por exemplo, se y = 2, entao necessariamente z = 4 e w = 3.
Logo, fixando x = 1 temos 2 possibilidades para a sequencia (y = 2 ou y = 3).

Caso 2: x = 2 == 1 0 2 3 0 4 2 y z w
y soh pode ser 1 e z pode ser 3 ou 4.
Se z = 3 entao w = 4 e se z = 4 entao w = 3.
Logo, fixando x = 2 temos 2 possibilidades para a sequencia (z = 3 ou z = 4)

Caso 3: x = 3.
Esse caso eh analogo ao anterior, resultando em mais duas possibilidades para a sequencia.

*Em suma, existem 4!*(2+2+2) = 144 sequencias do primeiro tipo.

Uma analise semelhante mostra que existem 96 sequencias do segundo tipo (a menos de algum erro de conta).

Logo, temos um total de 144 + 96 = 240 sequencias, ou seja, 240 configuracoes possiveis para um jogo fechado de dominos usando apenas os 10 dominos que contem os numeros de 0 a 4.

Se voce quiser o numero de maneiras em que o jogo pode proceder de forma a usar todas as 10 pecas, voce precisa multiplicar 240 por 10*2^8, pois temos 10 possibilidades para a primeira peca colocada e, depois disso, 2 possibilidades para a peca seguinte (uma de cada lado da sequencia de dominos na mesa) ateh a nona. A decima fica unicamente determinada. Outra maneira de ver isso eh rodar o jogo de tras pra frente e ir retirando as pecas.

***

Pra 21 dominos com os numeros 0, 1, 2, 3, 4, 5, 6, acho que dah pra fazer do mesmo jeito, mas com muito mais sub-casos.

[]s,
Claudio.

on 12.05.05 20:29, Claudio Buffara at [EMAIL PROTECTED] wrote:

Acho que nao eh por falta de interesse que as pessoas nao estao respondendo.
Deve ser porque o problema eh dificil mesmo.

Uma ideia eh, ao inves de olhar para os dominos, olhe para os pares de numeros identicos em sequencia.
Por exemplo, uma dada sequencia de dominos pode ser:
...[2,3][3,6][6,0][0,1][1,3][3,4][4,5][5,3][3,0]... 
A sequencia correspondente de pares de numeros identicos eh:
...(2,2)(3,3)(6,6)(0,0)(1,1)(3,3)(4,4)(5,5)(3,3)(0,0)...

Assim, ao inves de calcular o numero de sequencias de 21 dominos voce poderia olhar para o numero de sequencias de 21 numeros do conjunto A = {0,1,2,3,4,5,6} tais que:
1) elas comecam e terminam com o mesmo numero;
2) cada numero aparece exatamente 3 vezes;
3) dois numeros quaisquer (distintos) de A aparecem exatamente uma vez como vizinhos.

Essas condicoes devem ser redundantes, mas nao faz mal.

Talvez seja mais facil desse jeito...

[]s,
Claudio.

on 12.05.05 18:05, eritotutor at [EMAIL PROTECTED] wrote:

 boa noite pessoal ...
 
Estou há algum tempo para fazer a quetão abaixo, entretanto, não sei q abordagem tomar para analisá-la
Ficaria grato se vcs pudessem me ajudar
 
 Considere um jogo de dominó sem as peças com valores iguais dos dois lados (restarão portanto 21 peças). De quantas formas diferentes é possível fechar o jogo, em uma partida com dois jogadores?
 
 Desde já agradeço...
 
 []s











[obm-l] Dominos (correcao)

2005-05-13 Por tôpico Claudio Buffara
Hoje tah mais complicado que o de habito

Problema: Dados 10 dominos distintos da forma {a,b} com 0 = a  b = 4, de
quantas formas podemos arranja-los em sequencia de modo que dois dominos
vizinhos tenham um numero em comum (e o 10o. domino tenha um numero em comum
com o 1o.)?

Problema equivalente: De quantas maneiras podemos arranjar 10 elementos do
conjunto {0,1,2,3,4} em sequencia de modo que dois numeros quaisquer sejam
vizinhos exatamente uma vez (o 10o. termo da sequencia eh considerado
vizinho do 1o.)

Solucao proposta para o segundo problema:

Fixe um elemento qualquer de {0,1,2,3,4}. Digamos, o 0.

Teremos 3 tipos de sequencia:
(1)  a 0 b c 0 d x y z w
ou
(2)  a 0 b x c 0 d y z w
ou
(3)  a 0 b x y c 0 d z w

onde {a,b,c,d} = {x,y,z,w} = {1,2,3,4}

Repare que sequencias do tipo a 0 b x y z c 0 d w jah estao incluidas dentre
as de tipo (2) e sequencias do tipo a 0 b x y z w c 0 d dentre as de tipo
(1).

Em cada sequencia de cada tipo, a, b, c e d poderao ser escolhidos de 4! =
24 maneiras distintas.

No tipo (1): a 0 b c 0 d x y z w
x pertence a {a, b, c}
x = a == w = b  ou  w = c
x = b == w = c  ou  w = d
x = c == w = b  ou  w = d
Em cada caso, y e z ficam unicamente determinados.
Por exemplo, se x = b e w = d, entao y = a e z = c necessariamente.
Assim, fixados a, b, c, d, teremos 3*2 = 6 possibilidades para x, y, z, w.

No tipo (2): a 0 b x c 0 d y z w
x pertence a {a, d}
x = a == w = d == {y,z} = {b,c}
x = d ==  y = a == {z,w} = {b,c}
Assim, fixados a, b, c, d, teremos 2*2 = 4 possibilidades para x, y, z, w.

No tipo(3): a 0 b x y c 0 d z w
{x,y} = {a,d}  e  {z,w} = {b,c}
Assim, fixados a, b, c, d, teremos 2*2 = 4 possibilidades para x, y, z, w.

Logo, fixados a, b, c, d, teremos 6 + 4 + 4 = 14 sequencias de todos os
tipos.

Total = 24*14 = 336.

Logo, existem 336 configuracoes diferentes para um jogo fechado de 10
dominos, cada um com 2 numeros distintos do conjunto {0,1,2,3,4}.

Cada configuracao pode ser obtida de 10*2^8 = 2560 maneiras (ou seja,
existem 2560 ordens distintas de colocacao dos dominos na mesa para se obter
cada configuracao)

Assim, com estes 10 dominos, um jogo fechado pode ser obtido de 860160
formas distintas

Espero que agora esteja certo.

[]s,
Claudio.


=
Instruções para entrar na lista, sair da lista e usar a lista em
http://www.mat.puc-rio.br/~nicolau/olimp/obm-l.html
=


Re: [obm-l] Problemas de Algebra

2005-05-13 Por tôpico Claudio Buffara
O chato (entenda-se embaracoso) foi eu nao ter pensado nele.
Eu negligenciei qualquer exemplo com matrizes pois em qualquer semi-grupo
multiplicativo de matrizes que contem I (matriz identidade), I serah o
elemento identidade e, nesse caso, se BA = I entao AB = I.
Eu esqueci do fato crucial de que existem semi-grupos (como o seu) onde o
elemento identidade nao eh necessariamente a matriz identidade.

[]s,
Claudio.

on 13.05.05 13:35, Carlos Gustavo Tamm de Araujo Moreira at [EMAIL PROTECTED]
wrote:

 Oi Claudio,
 De fato esse exemplo com dois elementos corresponde a esse exemplo das
 matrizes sobre Z/2Z (em geral corresponde as matrizes como essas com
 a(1,1)=1 e a(1,2) em {0,1}).
 Abracos,
 Gugu
 
 
 O das matrizes tudo bem, mas esse exemplo com dois elementos foi chato!
 
 Muito obrigado.
 
 []s,
 Claudio.
 
 on 13.05.05 00:19, Carlos Gustavo Tamm de Araujo Moreira at [EMAIL PROTECTED]
 wrote:
 
 
 Oi, pessoal:
 
 Preciso de ajuda nos seguintes problemas sobre grupos do Herstein - Topics
 in Algebra:
 
 Secao 2.4:
 
 13) De um exemplo de um conjunto S, fechado em relacao a uma operacao
 associativa * e tal que:
 i) Existe e em S, tal que a*e = a, para todo a em S;
 ii) Para todo a em S, existe y(a) em S tal que y(a)*a = e;
 iii) S nao eh um grupo.
 
 Um exemplo: S={e,a}, e.e=e, a.e=a, e.a=e, a.a=a. Mais geralmente,
 S={matrizes 2x2 com segunda coluna nula e a(1,1) nao nulo; e=(1,0; 0,0)}.
 
 
 
 Secao 2.6:
 
 8) De um exemplo de um grupo G, um subgrupo H, e um elemento a de G tais
 que
 aHa^(-1) estah propriamente contido em H.
 
 Um tal H, se existir, tem que ser necessariamente infinito, alem de
 nao-abeliano. Eu imagino que deva haver algum grupo de matrizes com esta
 propriedade, mas nao consegui pensar em nenhum.
 
 Esse eu achei mais dif?cil: acho que podemos tomar um grupo gerado por
 elementos a e x(n), com n inteiro, que s? satisfazem as rela??es
 a.x(n).a^(-1)=x(n+1), e H gerado pelos x(n) com n natural (aHa^(-1) vai ser
 gerado pelos x(n+1) com n natural). Talvez haja exemplos mais simples e
 naturais...  
 
 
 []s,
 Claudio.
 
 

=
Instruções para entrar na lista, sair da lista e usar a lista em
http://www.mat.puc-rio.br/~nicolau/olimp/obm-l.html
=


Re: [obm-l] Problemas de Algebra

2005-05-13 Por tôpico Claudio Buffara
on 13.05.05 13:48, Carlos Gustavo Tamm de Araujo Moreira at [EMAIL PROTECTED]
wrote:

 Oi Claudio,
 Qual e' esse problema 26 da secao 2.5 ?

Provar que se um grupo abeliano possui subgrupos de ordens m e n, entao ele
possui um subgrupo cuja ordem eh mmc(m,n).

Isso eh facil de provar (usando apenas o teorema de Lagrange e a formula
|HK| = |H|*|K|/|H inter K|, onde H e K sao subgrupos) se m e n sao primos
entre si ou se os subgrupos de ordens m e n sao ciclicos.

Eu conheco uma demonstracao do caso geral que usa o teorema de Sylow:
se p^k || |G| entao G tem um subgrupo de ordem p^k.
Assim, se |H| = m e |K| = n, entao para cada potencia de primo p^k tal que
p^k || mmc(m,n), tome o subgrupo A_p de H ou K, conforme o caso, tal que
|A_p| = p^k. O subgrupo desejado eh Produto(p | mmc(m,n)) A_p.

Mas o que o livro pede eh uma demonstracao que nao use nenhum resultado mais
profundo do que os dois que eu mencionei acima e nem mesmo alguma construcao
de grupo quociente.

 Gostei muito do exemplo do Nicolau. Eu pensei em alguns outros depois de
 responder a mensagem, por exemplo, um grupo G gerado por a e b com b de ordem
 2 e sem outras relacoes. O conjunto H dos elementos cuja representacao
 simplificada e' uma palavra formada pelas letras a, a^(-1) e b tal que o
 numero de a's nas suas k primeiras letras e' sempre maior ou igual ao numero
 de a^(-1)'s, para todo k, e' um subgrupo

Entao, pelo que eu entendi, ababa^(-1) estah em H mas (ababa^(-1)) =
aba^(-1)ba(-1) nao estah. Eh isso mesmo?

 e aHa^(-1) esta' estritamente
 contido em H - senao b=axa^(-1) para algum x em H, mas x tem que ser
 a^(-1)ba, que nao esta' em H.

 Outro exemplo e' G={bijecoes crescentes de R
 em R} e H={f(x) em G tal que lim (x-+oo) f(x)/x existe e e' racional
 positivo}. Se a=a(x)=x^3, afa^(-1)(x)=f(x^(1/3))^3, e, se f(x)/x tende a c
 racional positivo entao afa^(-1)(x)/x tende a c^3, que tambem e' racional
 positivo. Por outro lado, f(x)=2x pertence a H, mas se afa^(-1)(x)=2x entao
 f(y)=2^(1/3).y, que nao pertence a H. Eu tentei um pouco achar um exemplo
 natural com G={bijecoes de N}, mas so' consegui versoes artificiais de
 exemplos anteriores...

Esse eh bem legal!

[]s,
Claudio.

=
Instruções para entrar na lista, sair da lista e usar a lista em
http://www.mat.puc-rio.br/~nicolau/olimp/obm-l.html
=


[obm-l] Construcao Geometrica

2005-05-13 Por tôpico Claudio Buffara
Dado um triangulo ABC, construa, com regua e compasso, um ponto P no lado AB
e um ponto Q no lado AC tais que |BP| = |PQ| = |QC|.

[]s,
Claudio.

=
Instruções para entrar na lista, sair da lista e usar a lista em
http://www.mat.puc-rio.br/~nicolau/olimp/obm-l.html
=


Re: [obm-l] Teorema do Valor Intermedi ário e Função Monótona

2005-05-12 Por tôpico Claudio Buffara
Title: Re: [obm-l] Teorema do Valor Intermediário e Função Monótona



Seja f: [0,1] - [0,1] dada por:
f(x) = x se x eh racional
f(x) = 1-x se x eh irracional.
f satisfaz as suas condicoes pois eh uma bijecao mas estah longe de ser monotona.
Alem disso, eh descontinua em cada ponto de [0,1] exceto 1/2.

[]s,
Claudio.

on 12.05.05 08:48, Bruno Pereira Dias at [EMAIL PROTECTED] wrote:

Olá pessoal,

Em minha aula de Cálculo surgiu uma questão que o professor não conseguiu responder:

Suponha que f seja uma função que satisfaça a conclusão do Teorema do Valor Intermediário num intervalo [a,b], mas f toma cada valor entre f(a) e f(b) apenas uma única vez. Prove ou dê um contra-exemplo que f é monótona em [a,b].

Agradeço qualquer ajuda,

Bruno






[obm-l] Problemas de Algebra

2005-05-12 Por tôpico Claudio Buffara
Oi, pessoal:

Preciso de ajuda nos seguintes problemas sobre grupos do Herstein - Topics
in Algebra:

Secao 2.4:

13) De um exemplo de um conjunto S, fechado em relacao a uma operacao
associativa * e tal que:
i) Existe e em S, tal que a*e = a, para todo a em S;
ii) Para todo a em S, existe y(a) em S tal que y(a)*a = e;
iii) S nao eh um grupo.

Secao 2.6:

8) De um exemplo de um grupo G, um subgrupo H, e um elemento a de G tais que
aHa^(-1) estah propriamente contido em H.

Um tal H, se existir, tem que ser necessariamente infinito, alem de
nao-abeliano. Eu imagino que deva haver algum grupo de matrizes com esta
propriedade, mas nao consegui pensar em nenhum.

[]s,
Claudio.

=
Instruções para entrar na lista, sair da lista e usar a lista em
http://www.mat.puc-rio.br/~nicolau/olimp/obm-l.html
=


Re: [obm-l] Teorema do Valor Intermedi ário e Função Monótona

2005-05-12 Por tôpico Claudio Buffara
on 12.05.05 11:41, Nicolau C. Saldanha at [EMAIL PROTECTED] wrote:

 On Thu, May 12, 2005 at 11:30:33AM -0300, Claudio Buffara wrote:
 Seja f: [0,1] - [0,1] dada por:
 f(x) = x se x eh racional
 f(x) = 1-x se x eh irracional.
 f satisfaz as suas condicoes pois eh uma bijecao mas estah longe de ser
 monotona.
 Alem disso, eh descontinua em cada ponto de [0,1] exceto 1/2.
 
 Da forma como eu interpretei o enunciado, f não satisfaz a conclusão do TFI
 (e portanto não satisfaz ax condições do problema).
 
 Por exemplo, temos f(1/10) = 1/10, f(2/10) = 2/10, 1/10  pi - 3  2/10
 mas não existe x entre 1/10 e 2/10 tal que f(x) = pi-3.
 
 []s, N.

Tah certo. O TVI diz que para todo y entre f(c) e f(d) existe x
*** entre c e d ***,
com f(x) = y. Assim, f bijetiva nao basta.

Obrigado pela correcao.

[]s,
Claudio.


=
Instruções para entrar na lista, sair da lista e usar a lista em
http://www.mat.puc-rio.br/~nicolau/olimp/obm-l.html
=


Re: [obm-l] Problemas de Algebra

2005-05-12 Por tôpico Claudio Buffara
on 12.05.05 14:41, [EMAIL PROTECTED] at [EMAIL PROTECTED] wrote:

 Claudio Buffara ([EMAIL PROTECTED]) escreveu:
 
 Oi, pessoal:
 
 Preciso de ajuda nos seguintes problemas sobre grupos do Herstein - Topics
 in Algebra:
 
 Secao 2.4:
 
 13) De um exemplo de um conjunto S, fechado em relacao a uma operacao
 associativa * e tal que:
 i) Existe e em S, tal que a*e = a, para todo a em S;
 ii) Para todo a em S, existe y(a) em S tal que y(a)*a = e;
 iii) S nao eh um grupo.
 
 Oi, Cláudio
 Dei uma olhada no meu Hernstein:
 
 Veja o problema 12:
 12) Seja G um conjnto não vazio fechado com relação a um produto
 associativo, que além disso satisfaz
 a) Existe e em G tal que a*e = a para todo a em G
 b) Dado a em G, existe um elemento y(a) tal que a*y(a) = e.
 Demonstrar que G é um grupo com relação a este produto.
 
 Aí o problema 13 é assim:
 13) Demonstrar, através de um exemplo, que a conclusão do Problema 12 é
 falsa se admitirmos, ao invés:
 a) Existe um e em G tal que a*e = a para todo a em G
 b) Dado a em G, existe um elemento y(a) em G tal que a*y(a) = e.
 
Eh importante a ordem dos produtos.
No 12 eh:  a*e = a   e   a*y(a) = e.
No 13 eh:  a*e = a   e   y(a)*a = e.

Se no seu livro estiver como voce disse, entao eh erro da traducao.

 Se vc disser que G é fechado para um produto associativo, então o enunciado
 do 13 é idêntico ao do 12, a menos que vc pense o (a) e (b) do 13 como itens
 separados, isto é, dar exemplo de quando somente (a) vale e depois quando
 somente (b) vale.

Se soh o (a) valer, entao tome G = (N,+).
Nao tem como soh valer o (b), senao quem eh e?

[]s,
Claudio.

 Ou então trata-se de um erro do livro, possivelmente na tradução (já
 encontrei vários no meu exemplar)
 
 []s,
 Daniel
 


=
Instruções para entrar na lista, sair da lista e usar a lista em
http://www.mat.puc-rio.br/~nicolau/olimp/obm-l.html
=


Re: [obm-l] questão de analise dificil

2005-05-12 Por tôpico Claudio Buffara
Title: Re: [obm-l] questão de analise dificil



Acho que nao eh por falta de interesse que as pessoas nao estao respondendo.
Deve ser porque o problema eh dificil mesmo.

Uma ideia eh, ao inves de olhar para os dominos, olhe para os pares de numeros identicos em sequencia.
Por exemplo, uma dada sequencia de dominos pode ser:
...[2,3][3,6][6,0][0,1][1,3][3,4][4,5][5,3][3,0]... 
A sequencia correspondente de pares de numeros identicos eh:
...(2,2)(3,3)(6,6)(0,0)(1,1)(3,3)(4,4)(5,5)(3,3)(0,0)...

Assim, ao inves de calcular o numero de sequencias de 21 dominos voce poderia olhar para o numero de sequencias de 21 numeros do conjunto A = {0,1,2,3,4,5,6} tais que:
1) elas comecam e terminam com o mesmo numero;
2) cada numero aparece exatamente 3 vezes;
3) dois numeros quaisquer (distintos) de A aparecem exatamente uma vez como vizinhos.

Essas condicoes devem ser redundantes, mas nao faz mal.

Talvez seja mais facil desse jeito...

[]s,
Claudio.

on 12.05.05 18:05, eritotutor at [EMAIL PROTECTED] wrote:

 boa noite pessoal ...
 
 Estou há algum tempo para fazer a quetão abaixo, entretanto, não sei q abordagem tomar para analisá-la
 Ficaria grato se vcs pudessem me ajudar
 
 Considere um jogo de dominó sem as peças com valores iguais dos dois lados (restarão portanto 21 peças). De quantas formas diferentes é possível fechar o jogo, em uma partida com dois jogadores?
 
 Desde já agradeço...
 
 []s
 
 







Re: [obm-l] identidade

2005-05-12 Por tôpico Claudio Buffara
on 12.05.05 17:28, Fabio Niski at [EMAIL PROTECTED] wrote:

 Ola pessoal.
 Como posso chegar na seguinte igualdade (operando apenas com os termos
 do lado esquerdo)
 
 
 [(z^m-w^m)/(z-w)]-[m*w^(m-1)]=(z-w)*Soma[1=k=m-1](k*w^(k-1)*z^(m-k-1)
 
 (supondo m = 2, e só pra ficar claro; Soma = Somatorio para k indo de 1
 até m-1)
 
 Eu tentei fazer desenvolvendo z^m - w^m, mas sem sucesso no final.
 Alguem tem alguma sugestão?

Aquele k*w^(k-1) do lado direito sugere uma derivada.

[]s,
Claudio.


=
Instruções para entrar na lista, sair da lista e usar a lista em
http://www.mat.puc-rio.br/~nicolau/olimp/obm-l.html
=


<    3   4   5   6   7   8   9   10   11   12   >